GMAT考試寶典 -- 作文範文全集

 109題argument範文.. 13

1. the following appeared as part of an annual report sent to stockholders by olympic foods, a processor of frozen foods. 13

2. the following appeared in a memorandum from the business department of the apogee company. 14

3. the following appeared in a memorandum issued by a large city’s council on the arts. 14

4. the following appeared in a report presented for discussion at a meeting of the directors of a company that manufactures parts for heavy machinery. 15

5. the following appeared in an announcement issued by the publisher of the mercury, a weekly newspaper. 16

6. the following appeared as part of an article in a magazine devoted to regional life. 17

7. the following appeared in the health section of a magazine on trends and lifestyles. 18

8. the following appeared in the editorial section of a corporate newsletter. 18

9. the following appeared in the opinion column of a financial magazine. 19

10. the following appeared in the editorial section of a local newspaper. 21

11. the following appeared in the editorial section of a local newspaper. 22

12. the following appeared as part of a promotional campaign to sell advertising space in the daily gazette to grocery stores in the marston area. 23

13. the following appeared as part of a campaign to sell advertising time on a local radio station to local businesses. 24

14. the following appeared as part of a newspaper editorial. 25

15. the following appeared as a part of an advertisement for adams, who is seeking reelection as governor. 26

16. the following appeared as part of an article in the education section of a waymarsh city newspaper. 27

17. the following appeared in an article in a consumer-products magazine. 28

18. the following is an excerpt from a memo written by the head of a governmental department. 29

19. the following appeared as part of an article in the travel section of a newspaper. 30

20. the following appeared in an article in a health and fitness magazine. 31

21. the following appeared as part of an editorial in an industry newsletter. 32

22. the following appeared in the editorial section of a newspaper. 33

23. the following appeared in a speech delivered by a member of the city council. 34

24. the following appeared in a memo from the customer service division to the manager of mammon savings and loan. 35

25. the following appeared as part of an article in a magazine on lifestyles. 36

26. the following appeared in a memorandum from a member of a financial management and consulting firm. 37

27. the following appeared in a newspaper editorial. 38

28. the following appeared in the editorial section of a local newspaper. 39

29. the following was excerpted from the speech of a spokesperson for synthetic farm products, inc. 40

30. the following appeared in a newspaper story giving advice about investments. 41

31. the following appeared as part of the business plan of an investment and financial consulting firm. 42

32. the following appeared in the editorial section of a west cambria newspaper. 43

33. the following is part of a business plan being discussed at a board meeting of the perks company. 44

34. the following appeared as part of a plan proposed by an executive of the easy credit company to the president. 44

35. the following appeared as part of a recommendation from the financial planning office to the administration of fern valley university. 45

36. the following appeared in an article in a college departmental newsletter 46

37. the following appeared as part of an article in the business section of a local newspaper. 47

38. the following appeared in the editorial section of a campus newspaper. 48

39. the following appeared in an avia airlines departmental memorandum. 49

40. the following appeared as part of an article in a weekly newsmagazine. 50

41. the following appeared as part of an article in a trade publication. 51

42. the following appeared in the opinion section of a national newsmagazine. 52

43. the following appeared in an article in the health section of a newspaper. 53

44. the following is part of a business plan created by the management of the megamart grocery store. 54

45. the following appeared as part of a column in a popular entertainment magazine. 54

46. the following appeared in a memorandum from the directors of a security and safety consulting service. 55

47. the following appeared as part of an article in the business section of a local newspaper. 56

48. the following appeared in the editorial section of a local newspaper. 57

49. the following appeared in the editorial section of a local newspaper. 57

50. the following appeared as part of a business plan recommended by the new manager of a musical rock group called zapped. 58

51. the following appeared in a magazine article on trends and lifestyles. 59

52. the following editorial appeared in the elm city paper. 60

53. the following appeared as part of an editorial in a weekly newsmagazine. 61

54. the following appeared in an excelsior company memorandum. 62

55. the following appeared as part of an article in a health club trade publication. 63

56. the following appeared as part of an article in a popular arts and leisure magazine. 63

57. the following is from a campaign by big boards, inc., to convince companies in river city that their sales will increase if they use big boards billboards for advertising their locally manufactured products. 64

58. the following appeared as part of an article on government funding of environmental regulatory agencies. 65

59. the following appeared as part of an article in a popular science magazine. 66

60. the following appeared as part of a recommendation by one of the directors of the beta company. 67

61. the following appeared in the letters-to-the-editor section of a local newspaper. 68

62. the following appeared as part of an article in the business section of a local newspaper. 68

63. the following appeared in a memorandum from the director of human resources to the executive officers of company x. 69

64. the following appeared in a memorandum from the vice president of road food, an international chain of fast-food restaurants. 70

65. the following appeared in the promotional literature for cerberus dog food. 71

66. the following appeared in an article in a travel magazine. 72

67. the following appeared in a memorandum to the planning department of an investment firm. 73

68. the following appeared in a memorandum from a company’s marketing department. 74

69. the following appeared in a memorandum from the president of a company that makes (舊題有修飾詞glabrous) shampoo. 75

70. the following appeared as part of a recommendation from the business manager of a department store. 75

71. the following appeared in a letter to the editor of a regional newspaper. 76

72. the following appeared as part of an editorial in a campus newspaper. 77

73. the following appeared as part of a memorandum from a government agency. 78

74. the following appeared as part of an article in an entertainment magazine. 79

75. the following appeared in a letter to the editor of a popular science and technology magazine. 79

76. the following appeared in the editorial section of a local newspaper. 80

77. the following appeared in the editorial section of a local newspaper. 81

78. the following appeared in the editorial section of a monthly business newsmagazine. 82

79. the following appeared as part of a company memorandum. 83

80. the following appeared in the editorial section of a daily newspaper. 84

81. the following appeared in the editorial section of a newspaper in the country of west cambria. 84

82. the following appeared as part of a memorandum from the vice president of nostrum, a large pharmaceutical corporation. 85

83. the following appeared as part of an article on trends in television. 86

84. the following appeared as part of an article in the business section of a daily newspaper. 87

85. the following appeared as part of an article in a photography magazine. 88

86. the following appeared as part of a letter to the editor of a local newspaper. 89

87. the following appeared in an ad for a book titled how to write a screenplay for a movie. 90

88. the following appeared in a memorandum from the electrowares company’s marketing department. 91

89. the following is taken from an editorial in a local newspaper. 92

90. the following appeared as part of an article in a local newspaper. 93

91. the following appeared in a proposal from the development office at platonic university. 94

92. the following appeared as part of an article in the business section of a local newspaper. 94

93. the following appeared in a memorandum from the manager of kmtv, a television station. 95

94. the following appeared as part of an article in a computer magazine. 96

95. the following was excerpted from an article in a farming trade publication. 97

96. the following appeared in a letter to prospective students from the admissions office at plateau college. 98

97. the following appeared in a memorandum sent by a vice-president of the nadir company to the company’s human resources department. 99

98. the following appeared as part of an article in a trade magazine for breweries. 100

99. the following appeared in an editorial from a newspaper serving the town of saluda. 100

100. the following appeared as part of an article in the book section of a newspaper. 101

101. the following appeared as an editorial in a magazine concerned with educational issues. 102

102. the following appeared as part of a business plan created by the management of the take heart fitness center. 103

103. the following appeared in a letter from a staff member in the office of admissions at argent university. 104

104. the following appeared as part of a memorandum from the loan department of the frostbite national bank. 105

105. the following appeared as part of a letter to the editor of a local newspaper. 106

106. the following appeared in a memo to the saluda town council from the town’s business manager. 107

107. the following appeared in a memorandum written by the assistant manager of a store that sells gourmet food items from various countries. 108

108. the following appeared in a memorandum from the director of research and development at ready-to-ware, a software engineering firm. 108

109. the following appeared in a memorandum from the vice-president of the dolci candy company. 109

115題issue範文.. 111

1. in some countries, television and radio programs are carefully censored for offensive language and behavior. in other countries, there is little or no censorship. 111

2. “it is unrealistic to expect individual nations to make, independently, the sacrifices necessary to conserve energy. international leadership and worldwide cooperation are essential if we expect to protect the world’s energy resources for future generations.” 111

3. “corporations and other businesses should try to eliminate the many ranks and salary grades that classify employees according to their experience and expertise. a ‘flat’ organizational structure is more likely to encourage collegiality and cooperation among employees.” 112

4. “of all the manifestations* of power, restraint in the use of that power impresses people most.” 113

5. “all groups and organizations should function as teams in which everyone makes decisions and shares responsibilities and duties. giving one person central authority and responsibility for a project or task is not an effective way to get work done.” 114

6. “there is only one definition of success — to be able to spend your life in your own way.” 114

7. “the best way to give advice to other people is to find out what they want and then advise them how to attain it.” 115

8. “for hundreds of years, the monetary system of most countries has been based on the exchange of metal coins and printed pieces of paper. however, because of recent developments in technology, the international community should consider replacing the entire system of coins and paper with a system of electronic accounts of credits and debits.” 116

9. “employees should keep their private lives and personal activities as separate as possible from the workplace.” 117

10. “in any enterprise, the process of making or doing something is ultimately more important than the final product.” 117

11. “when someone achieves greatness in any field — such as the arts, science, politics, or business — that person’s achievements are more important than any of his or her personal faults.” 118

12. “education has become the main provider of individual opportunity in our society. just as property and money once were the keys to success, education has now become the element that most ensures success in life.” 119

13. “responsibility for preserving the natural environment ultimately belongs to each individual person, not to government.” 120

14. “organizations should be structured in a clear hierarchy in which the people at each level, from top to bottom, are held accountable for completing a particular component of the work. any other organizational structure goes against human nature and will ultimately prove fruitless.” 120

15. “nations should cooperate to develop regulations that limit children’s access to adult material on the internet.” *  121

16. “public buildings reveal much about the attitudes and values of the society that builds them. today’s new schools, courthouses, airports, and libraries, for example, reflect the attitudes and values of today’s society.” 122

17. “some people believe that the best approach to effective time management is to make detailed daily and long-term plans and then to adhere to them. however, this highly structured approach to work is counterproductive. time management needs to be flexible so that employees can respond to unexpected problems as they arise.” 123

18. “if the primary duty and concern of a corporation is to make money, then conflict is inevitable when the corporation must also acknowledge a duty to serve society.” 124

19. some employers who recruit recent college graduates for entry-level jobs evaluate applicants only on their performance in business courses such as accounting, marketing, and economics. however, other employers also expect applicants to have a broad background in such courses as history, literature, and philosophy. 124

20. “in this age of automation, many people complain that humans are becoming subservient to machines. but, in fact, machines are continually improving our lives.” 126

21. “job security and salary should be based on employee performance, not on years of service. rewarding employees primarily for years of service discourages people from maintaining consistently high levels of productivity.” 127

22. “clearly, government has a responsibility to support the arts. however, if that support is going to produce anything of value, government must place no restrictions on the art that is produced.” 128

23. “schools should be responsible only for teaching academic skills and not for teaching ethical and social values.” 128

24. “a powerful business leader has far more opportunity to influence the course of a community or a nation than does any government official.” 129

25. “the best strategy for managing a business, or any enterprise, is to find the most capable people and give them as much authority as possible.” 130

26. “location has traditionally been one of the most important determinants of a business’s success. the importance of location is not likely to change, no matter how advanced the development of computer communications and others kinds of technology becomes.” 131

27. “a company’s long-term success is primarily dependent on the job satisfaction and the job security felt by the company’s employees.” 131

28. “because businesses use high-quality advertising to sell low-quality products, schools should give students extensive training in how to make informed decisions before making purchases.” 132

29. “too many people think only about getting results. the key to success, however, is to focus on the specific task at hand and not to worry about results.” 133

30. “companies benefit when they discourage employees from working extra hours or taking work home. when employees spend their leisure time without ‘producing’ something for the job, they will be more focused and effective when they return to work.” 134

31. “financial gain should be the most important factor in choosing a career.” 134

32. “you can tell the ideas of a nation by its advertisements.” 135

33. “people are likely to accept as a leader only someone who has demonstrated an ability to perform the same tasks that he or she expects others to perform.” 136

34. “all citizens should be required to perform a specified amount of public service. such service would benefit not only the country as a whole but also the individual participants.” 136

35. “business relations are infected through and through with the disease of short-sighted motives. we are so concerned with immediate results and short-term goals that we fail to look beyond them.” 137

36. “businesses and other organizations have overemphasized the importance of working as a team. clearly, in any human group, it is the strong individual, the person with the most commitment and energy, who gets things done.” 138

37. “since science and technology are becoming more and more essential to modern society, schools should devote more time to teaching science and technology and less to teaching the arts and humanities.” 138

38. “courtesy is rapidly disappearing from everyday interactions, and as a result, we are all the poorer for it.” 139

39. “it is difficult for people to achieve professional success without sacrificing important aspects of a fulfilling personal life.” 140

40. “with the increasing emphasis on a global economy and international cooperation, people need to understand that their role as citizens of the world is more important than their role as citizens of a particular country.” 141

41. “the best way to preserve the natural environment is to impose penalties—whether fines, imprisonment, or other punishments—on those who are most responsible for polluting or otherwise damaging it.” 141

42. “scientists are continually redefining the standards for what is beneficial or harmful to the environment. since these standards keep shifting, companies should resist changing their products and processes in response to each new recommendation until those recommendations become government regulations.” 142

43. “the most important reason for studying history is not that knowledge of history can make us better people or a better society but that it can provide clues to solving the societal problems that we face today.” 143

44. “all companies should invest heavily in advertising because high-quality advertising can sell almost any product or service.” 144

45. “the most effective way for a businessperson to maximize profits over a long period of time is to follow the highest standards of ethics.” 145

46. businesses are as likely as are governments to establish large bureaucracies, but bureaucracy is far more damaging to a business than it is to a government. 146

47. the primary responsibility for preventing environmental damage belongs to government, not to individuals or private industry. 146

48. in matching job candidates with job openings, managers must consider not only such variables as previous work experience and educational background but also personality traits and work habits, which are more difficult to judge. 147

49. “ask most older people to identify the key to success, and they are likely to reply ‘hard work.’ yet, i would tell people starting off in a career that work in itself is not the key. in fact, you have to approach work cautiously—too much or too little can be self-defeating.” 148

50. how far should a supervisor go in criticizing the performance of a subordinate? some highly successful managers have been known to rely on verbal abuse and intimidation. 149

51. “the presence of a competitor is always beneficial to a company. competition forces a company to change itself in ways that improve its practices.” 149

52. “successful individuals typically set their next goal somewhat—but not too much—above their last achievement. in this way, they steadily raise their level of aspiration.” 150

53. “the term ‘user-friendly’ is usually applied to the trouble-free way that computer software moves people from screen to screen, function to function. however, the term can also refer to a government office, a library, public transportation, or anything designed to provide information or services in an easy, friendly way. just as all societies have many striking examples of user-friendly services, so do they abound in examples of user-unfriendly systems.” identify a system or service that you have found to be either “user-friendly” or “user-unfriendly.” 151

54. “popular entertainment is overly influenced by commercial interests. superficiality, obscenity, and violence characterize films and television today because those qualities are commercially successful.” 152

55. “never tell people how to do things. tell them what to do, and they will surprise you with their ingenuity.” 153

56. “the secret of business is to know something that nobody else knows.” 153

57. “everywhere, it seems, there are clear and positive signs that people are becoming more respectful of one another’s differences.” 154

58. “what is the final objective of business? it is to make the obtaining of a living—the obtaining of food, clothing, shelter, and a minimum of luxuries—so mechanical and so little time-consuming that people shall have time for other things.” 155

59. “juvenile crime is a serious social problem, and businesses must become more involved in helping to prevent it.” 156

60. “employers should have no right to obtain information about their employees’ health or other aspects of their personal lives without the employees’ permission.” 156

61. “even at its best, a government is a tremendous burden to business, though a necessary one.” 157

62. “what education fails to teach us is to see the human community as one. rather than focus on the unique differences that separate one nation from another, education should focus on the similarities among all people and places on earth.” 158

63. “as government bureaucracy increases, citizens become more and more separated from their government.” 159

64. “the goal of business should not be to make as big a profit as possible. instead, business should also concern itself with the wellbeing (n. 福利) of the public.” 159

65. “the rise of multinational corporations is leading to global homogeneity*. because people everywhere are beginning to want the same products and services, regional differences are rapidly disappearing.” 160

66. “manufacturers are responsible for ensuring that their products are safe. if a product injures someone, for whatever reason, the manufacturer should be held legally and financially accountable for the injury.” 161

67. “work greatly influences people’s personal lives—their special interests, their leisure activities, even their appearance away from the workplace.” 162

68. “since the physical work environment affects employee productivity and morale, the employees themselves should have the right to decide how their workplace is designed.” 162

69. “the most important quality in an employee is not specific knowledge or technical competence. instead, it is the ability to work well with other employees.” 163

70. “so long as no laws are broken, there is nothing unethical about doing whatever you need to do to promote existing products or to create new products.” 164

71. “commercialism has become too widespread. it has even crept into schools and places of worship. every nation should place limits on what kinds of products, if any, can be sold at certain events or places.” 164

72. “companies should not try to improve employees’ performance by giving incentives—for example, awards or gifts. these incentives encourage negative kinds of behavior instead of encouraging a genuine interest in doing the work well.” 165

73. people often give the following advice: “be yourself. follow your instincts and behave in a way that feels natural.” 166

74. “the people we remember best are the ones who broke the rules.” 167

75. “there are essentially two forces that motivate people: self-interest and fear.” 167

76. “for a leader there is nothing more difficult, and therefore more important, than to be able to make decisions.” 168

77. although “genius” is difficult to define, one of the qualities of genius is the ability to transcend traditional modes of thought and create new ones. 169

78. most people would agree that buildings represent a valuable record of any society’s past, but controversy arises when old buildings stand on ground that modern planners feel could be better used for modern purposes. 169

79. “the ability to deal with people is as purchasable a commodity as sugar or coffee, and it is worth more than any other commodity under the sun.” 170

80. “as individuals, people save too little and borrow too much.” 171

81. “no one can possibly achieve any real and lasting success or ‘get rich’ in business by conforming to conventional practices or ways of thinking.” 172

82. “business and government must do more, much more, to meet the needs and goals of women in the workplace.” 172

83. “we shape our buildings and afterwards our buildings shape us.” 173

84. “a business should not be held responsible for providing customers with complete information about its products or services; customers should have the responsibility of gathering information about the products or services they may want to buy.” 174

85. “advertising is the most influential and therefore the most important artistic achievement of the twentieth century.” 175

86. “whether promoting a product, an event, or a person, an advertising campaign is most effective when it appeals to emotion rather than to reason.” 175

87. “as technologies and the demand for certain services change, many workers will lose their jobs. the responsibility for those people to adjust to such change should belong to the individual worker, not to government or to business.” 176

88. “each generation must accept blame not only for the hateful words and actions of some of its members but also for the failure of other members to speak out against those words and actions.” 177

89. “the study of history is largely a waste of time because it prevents us from focusing on the challenges of the present.” 177

90. “people often complain that products are not made to last. they feel that making products that wear out fairly quickly wastes both natural and human resources. what they fail to see, however, is that such manufacturing practices keep costs down for the consumer and stimulate demand.” 178

91. “government should establish regulations to reduce or eliminate any suspected health hazards in the environment, even when the scientific studies of these health hazards are incomplete or contradictory.” 180

92. “employees should show loyalty to their company by fully supporting the company’s managers and policies, even when the employees believe that the managers and policies are misguided.” 181

93. “to be successful, companies should trust their workers and give them as much freedom as possible. any company that tries to control employees’ behavior through a strict system of rewards and punishments will soon find that such controls have a negative effect on employee morale and, consequently, on the company’s success.” 181

94. “if parents want to prepare their children to succeed in life, teaching the children self-discipline is more important than teaching them self-esteem.” 182

95. “companies are never justified in employing young children, even if the child’s family would benefit from the income.” 183

96. “in order to understand a society, we must examine the contents of its museums and the subjects of its memorials. what a society chooses to preserve, display, and commemorate is the truest indicator of what the society values.” 184

97. “in business, more than in any other social arena, men and women have learned how to share power effectively.” 184

98. “in order to accommodate the increasing number of undergraduate students, college and universities should offer most courses through distance learning, such as videotaped instruction that can be accessed through the internet or cable television. requiring students to appear at a designated time and place is no longer an effective or efficient way of teaching most undergraduate courses.” 185

99. “if a nation is to ensure its own economic success, it must maintain a highly competitive educational system in which students compete among themselves and against students from other countries.” 186

100. “in order to force companies to improve policies and practices considered unethical or harmful, society should rely primarily on consumer action—such as refusal to buy products—rather than legislative action.” 187

101. “the automobile has caused more problems than it has solved. most societies would probably be much better off if the automobile had never been invented.” 188

102. “an advanced degree may help someone get a particular job. once a person begins working, however, the advanced degree and the formal education it represents are rarely relevant to success on the job.” 189

103. “most people today place too much emphasis on satisfying their immediate desires. the overall quality of life would be greatly improved if we all focused instead on meeting our long-term needs.” 189

104. “the value of any nation should be measured more by its scientific and artistic achievements than by its business successes.” 190

105. “all archeological treasures should remain in the country in which they were originally discovered. these works should not be exported, even if museums in other parts of the world are better able to preserve and display them.” 191

106. “the most effective way for managers to assign work is to divide complex tasks into their simpler component parts. this way, each worker completes a small portion of the task but contributes to the whole.” 192

107. “people are overwhelmed by the increasing amount of information available on the computer. therefore, the immediate goal of the information technology industry should be to help people learn how to obtain the information they need efficiently and wisely.” 192

108. “employees should not have full access to their own personnel files. if, for example, employees were allowed to see certain confidential materials, the people supplying that information would not be likely to express their opinions candidly.” 193

109. “all personnel evaluations at a company should be multi-directional — that is, people at every level of the organization should review not only those working ‘under’ them but also those working ‘over’ them.” 194

110. “the most effective business leaders are those who maintain the highest ethical standards.” 195

111. “because of recent advancements in business and technology, the overall quality of life in most societies has never been better than at the present time.” 196

112. “in most fields—including education, politics, and business—the prevailing philosophy never stays in place very long. this pattern of constantly shifting from one theoretical position to another is an inevitable reflection of human nature: people soon tire of the status quo.” 196

113. “it is essential that the nations of the world increase spending on the building of space stations and on the exploration of other planets, even if that means spending less on other government programs.” 197

114. “technology ultimately separates and alienates people more than it serves to bring them together.” 198

134. “although many people object to advertisements and solicitations that intrude into their lives through such means as the telephone, the internet, and television, companies and organizations must have the right to contact potential customers and donors whenever and however they wish.” 199

109題argument範文1. the following appeared as part of an annual report sent to stockholders by olympic foods, a processor of frozen foods.“over time, the costs of processing go down because as organizations learn how to do things better, they become more efficient. in color film processing, for example, the cost of a 3-by-5-inch print fell from 50 cents for five-day service in 1970 to 20 cents for one-day service in 1984. the same principle applies to the processing of food. and since olympic foods will soon celebrate its twenty-fifth birthday, we can expect that our long experience will enable us to minimize costs and thus maximize profits.”

discuss how well reasoned you find this argument. in your discussion be sure to analyze the line of reasoning and the use of evidence in the argument. for example, you may need to consider what questionable assumptions underlie the thinking and what alternative explanations or counterexamples might weaken the conclusion. you can also discuss what sort of evidence would strengthen or refute the argument, what changes in the argument would make it more logically sound, and what, if anything, would help you better evaluate its conclusion.

citing facts drawn from the color-film processing industry that indicate a downward trend in the costs of film processing over a 24-year period, the author argues that olympic foods will likewise be able to minimize costs and thus maximize profits in the future. in support of this conclusion the author cites the general principle that “as organizations learn how to do things better, they become more efficient.” this principle, coupled with the fact that olympic foods has had 25 years of experience in the food processing industry leads to the author’s rosy prediction. this argument is unconvincing because it suffers from two critical flaws.

first, the author’s forecast of minimal costs and maximum profits rests on the gratuitous assumption that olympic foods’ “long experience” has taught it how to do things better. there is, however, no guarantee that this is the case. nor does the author cite any evidence to support this assumption. just as likely, olympic foods has learned nothing from its 25 years in the food-processing business. lacking this assumption, the expectation of increased efficiency is entirely unfounded.

second, it is highly doubtful that the facts drawn from the color-film processing industry are applicable to the food processing industry. differences between the two industries clearly outweigh the similarities, thus making the analogy highly less than valid. for example, problems of spoilage, contamination, and timely transportation all affect the food industry but are virtually absent in the film-processing industry. problems such as these might present insurmountable obstacles that prevent lowering food-processing costs in the future.

as it stands the author’s argument is not compelling. to strengthen the conclusion that olympic foods will enjoy minimal costs and maximum profits in the future, the author would have to provide evidence that the company has learned how to do things better as a result of its 25 years of experience. supporting examples drawn from industries more similar to the food-processing industry would further substantiate the author’s view.

2. the following appeared in a memorandum from the business department of the apogee company.“when the apogee company had all its operations in one location, it was more profitable than it is today. therefore, the apogee company should close down its field offices (n. 外地辦事處) and conduct all its operations from a single location. such centralization would improve profitability by cutting costs and helping the company maintain better supervision of all employees.”

discuss how well reasoned... etc.

in this argument the author concludes that the apogee company should close down field offices and conduct all its operations from a single, centralized location because the company had been more profitable in the past when all its operations were in one location. for a couple of reasons, this argument is not very convincing.

first, the author assumes that centralization would improve profitability by cutting costs and streamlining supervision of employees. this assumption is never supported with any data or projections. moreover, the assumption fails to take into account cost increases and inefficiency that could result from centralization. for instance, company representatives would have to travel to do business in areas formerly served by a field office, creating travel costs and loss of critical time. in short, this assumption must be supported with a thorough cost-benefit analysis of centralization versus other possible cost-cutting and/or profit-enhancing strategies.

second, the only reason offered by the author is the claim that apogee was more profitable when it had operated from a single, centralized location. but is centralization the only difference relevant to greater past profitability? it is entirely possible that management has become lax regarding any number of factors that can affect the bottom line (帳本底線) such as inferior products, careless product pricing, inefficient production, poor employee expense account monitoring, ineffective advertising, sloppy buying policies and other wasteful spending. unless the author can rule out other factors relevant to diminishing profits, this argument commits the fallacy of assuming that just because one event (decreasing profits) follows another (decentralization), the second event has been caused by the first.

in conclusion, this is a weak argument. to strengthen the conclusion that apogee should close field offices and centralize, this author must provide a thorough cost-benefit analysis of available alternatives and rule out factors other than decentralization that might be affecting current profits negatively.

3. the following appeared in a memorandum issued by a large city’s council on the arts.“in a recent citywide poll, fifteen percent more residents said that they watch television programs about the visual arts than was the case in a poll conducted five years ago. during these past five years, the number of people visiting our city’s art museums has increased by a similar percentage. since the corporate funding that supports public television, where most of the visual arts programs appear, is now being threatened with severe cuts, we can expect that attendance at our city’s art museums will also start to decrease. thus some of the city’s funds for supporting the arts should be reallocated to public television.”

discuss how well reasoned... etc.

in this argument the author concludes that the city should allocate some of its arts funding to public television. the conclusion is based on two facts: (1) attendance at the city’s art museum has increased proportionally with the increases in visual-arts program viewing on public television, and (2) public television is being threatened by severe cuts in corporate funding. while this argument is somewhat convincing, a few concerns need to be addressed.

to begin with, the argument depends on the assumption that increased exposure to the visual arts on television, mainly public television, has caused a similar increase in local art-museum attendance. however, just because increased art-museum attendance can be statistically correlated with similar increases in television viewing of visual-arts programs, this does not necessarily mean that the increased television viewing of arts is the cause of the rise in museum attendance.

moreover, perhaps there are other factors relevant to increased interest in the local art museum; for instance, maybe a new director had procured more interesting, exciting acquisitions and exhibits during the period when museum attendance increased, in addition, the author could be overlooking a common cause of both increases. it is possible that some larger social or cultural phenomenon is responsible for greater public interest in both television arts programming and municipal art museums.

to be fair, however, we must recognize that the author’s assumption is a special case of a more general one that television viewing affects people’s attitudes and behavior. common sense and observation tell me that this is indeed the case. after all, advertisers spend billions of dollars on television ad time because they trust this assumption as well.

in conclusion, i am somewhat persuaded by this author’s line of reasoning. the argument would be strengthened if the author were to consider and rule out other significant factors that might have caused the increase in visits to the local art museum.

4. the following appeared in a report presented for discussion at a meeting of the directors of a company that manufactures parts for heavy machinery.“the falling revenues that the company is experiencing coincide with delays in manufacturing. these delays, in turn, are due in large part to poor planning in purchasing metals. consider further that the manager of the department that handles purchasing of raw materials has an excellent background in general business, psychology, and sociology, but knows little about the properties of metals. the company should, therefore, move the purchasing manager to the sales department and bring in a scientist from the research division to be manager of the purchasing department.”

discuss how well reasoned... etc.

in response to a coincidence between falling revenues and delays in manufacturing, the report recommends replacing the manager of the purchasing department. the grounds for this action are twofold. first, the delays are traced to poor planning in purchasing metals. second, the purchasing manager’s lack of knowledge of the properties of metals is thought to be the cause of the poor planning. it is further recommended that the position of the purchasing manager be filled by a scientist from the research division and that the current purchasing manager be reassigned to the sales department. in support of this latter recommendation, the report states that the current purchasing manager’s background in general business, psychology, and sociology equip him for this new assignment. the recommendations advanced in the report are questionable for two reasons.

to begin with, the report fails to establish a causal connection between the falling revenues of the company and the delays in manufacturing. the mere fact that falling revenues coincide with delays in manufacturing is insufficient to conclude that the delays caused the decline in revenue. without compelling evidence to support the causal connection between these two events, the report’s recommendations are not worthy of consideration.

second, a central assumption of the report is that knowledge of the properties of metals is necessary for planning in purchasing metals. no evidence is stated in the report to support this crucial assumption. moreover, it is not obvious that such knowledge would be required to perform this task. since planning is essentially a logistical function, it is doubtful that in-depth knowledge of the properties of metals would be helpful in accomplishing this task.

in conclusion, this is a weak argument. to strengthen the recommendation that the manager of the purchasing department be replaced, the author would have to demonstrate that the falling revenues were a result of the delays in manufacturing. additionally, the author would have to show that knowledge of the properties of metals is a prerequisite for planning in purchasing metals.

5. the following appeared in an announcement issued by the publisher of the mercury, a weekly newspaper.“since a competing lower-priced newspaper, the bugle, was started five years ago, the mercury’s circulation has declined by 10,000 readers. the best way to get more people to read the mercury is to reduce its price below that of the bugle, at least until circulation increases to former levels. the increased circulation of the mercury will attract more businesses to buy advertising space in the paper.”

discuss how well reasoned... etc.

a newspaper publisher is recommending that the price of its paper, the mercury, be reduced below the price of a competing newspaper, the bugle. this recommendation responds to a severe decline in circulation of the mercury during the 5-year period following the introduction of the bugle. the publisher’s line of reasoning is that lowering the price of the mercury will increase its readership, thereby increasing profits because a wider readership attracts more advertisers. this line of reasoning is problematic in two critical respects.

while it is clear that increased circulation would make the paper more attractive to potential advertisers, it is not obvious that lowering the subscription price is the most effective way to gain new readers. the publisher assumes that price is the only factor that caused the decline in readership. but no evidence is given to support this claim. moreover, given that the mercury was the established local paper, it is unlikely that such a mass exodus of its readers would be explained by subscription price alone.

there are many other factors that might account for a decline in the mercury’s popularity. for instance, readers might be displeased with the extent and accuracy of its news reporting, or the balance of local to other news coverage. moreover, it is possible the mercury has recently changed editors, giving the paper a locally unpopular political perspective. or perhaps readers are unhappy with the paper’s format, the timeliness of its feature articles, its comics or advice columns, the extent and accuracy of its local event calendar, or its rate of errors.

in conclusion, this argument is weak because it depends on an oversimplified assumption about the causal connection between the price of the paper and its popularity. to strengthen the argument, the author must identify and explore relevant factors beyond cost before concluding that lowering subscription prices will increase circulation and, thereby, increase advertising revenues.

6. the following appeared as part of an article in a magazine devoted to regional life.“corporations should look to the city of helios when seeking new business opportunities or a new location. even in the recent recession, helios’s unemployment rate was lower than the regional average. it is the industrial center of the region, and historically it has provided more than its share of the region’s manufacturing jobs. in addition, helios is attempting to expand its economic base by attracting companies that focus on research and development of innovative technologies.”

discuss how well reasoned... etc.

in this argument corporations are urged to consider the city of helios when seeking a new location or new business opportunities. to support this recommendation, the author points out that helios is the industrial center of the region, providing most of the region’s manufacturing jobs and enjoying a lower-than-average unemployment rate. moreover, it is argued, efforts are currently underway to expand the economic base of the city by attracting companies that focus on research and development of innovative technologies. this argument is problematic for two reasons.

to begin with, it is questionable whether the available labor pool in helios could support all types of corporations. given that helios has attracted mainly industrial and manufacturing companies in the past, it is unlikely that the local pool of prospective employees would be suitable for corporations of other types. for example, the needs of research and development companies would not be met by a labor force trained in manufacturing skills. for this reason, it’s unlikely that helios will be successful in its attempt to attract companies that focus or research and development of innovative technologies.

another problem with the available work force is its size. due to the lower than average unemployment rate in helios, corporations that require large numbers of workers would not find helios attractive. the fact that few persons are out of work suggests that new corporations will have to either attract new workers to helios or pay the existing workers higher wages in order to lure them away from their current jobs. neither of these alternatives seems enticing to companies seeking to relocate.

in conclusion, the author has not succeeded in providing compelling reasons for selecting helios as the site for a company wishing to relocate. in fact, the reasons offered function better as reasons for not relocating to helios. nor has the author provided compelling reasons for companies seeking new business opportunities to choose helios.

7. the following appeared in the health section of a magazine on trends and lifestyles.“people who use the artificial sweetener aspartame are better off consuming sugar, since aspartame can actually contribute to weight gain rather than weight loss. for example, high levels of aspartame have been shown to trigger a craving for food by depleting the brain of a chemical that registers satiety, or the sense of being full. furthermore, studies suggest that sugars, if consumed after at least 45 minutes of continuous exercise, actually enhance the body’s ability to burn fat. consequently, those who drink aspartame-sweetened juices after exercise will also lose this calorie-burning benefit. thus it appears that people consuming aspartame rather than sugar are unlikely to achieve their dietary goals.”

discuss how well reasoned... etc.

in this argument the author concludes that people trying to lose weight are better off consuming sugar than the artificial sweetener aspartame. to support this conclusion the author argues that aspartame can cause weight gain by triggering food cravings, whereas sugar actually enhances the body’s ability to burn fat. neither of these reasons provides sufficient support for the conclusion.

the first reason that aspartame encourages food cravings is supported by research findings that high levels of aspartame deplete the brain chemical responsible for registering a sense of being sated (sated, sating充分滿足), or full. but the author’s generalization based on this research is unreliable. the research was based on a sample in which large amounts of aspartame were administered; however, the author applies the research findings to a target population that includes all aspartame users, many of whom would probably not consume high levels of the artificial sweetener.

the second reason that sugar enhances the body’s ability to burn fat is based on the studies in which experimental groups, whose members consumed sugar after at least 45 minutes of continuous exercise, showed increased rates of fat burning. the author’s general claim, however, applies to all dieters who use sugar instead of aspartame, not just to those who use sugar after long periods of exercise. once again, the author’s generalization is unreliable because it is based on a sample that clearly does not represent all dieters.

to conclude, each of the studies cited by the author bases its findings on evidence that does not represent dieters in general; for this reason, neither premise of this argument is a reliable generalization. consequently, i am not convinced that dieters are better off consuming sugar instead of aspartame.

8. the following appeared in the editorial section of a corporate newsletter.“the common notion that workers are generally apathetic about management issues is false, or at least outdated: a recently published survey indicates that 79 percent of the nearly 1,200 workers who responded to survey questionnaires expressed a high level of interest in the topics of corporate restructuring and redesign of benefits programs.”

discuss how well reasoned... etc.

based upon a survey among workers that indicates a high level of interest in the topics of corporate restructuring and redesign of benefits programs, the author concludes that workers are not apathetic about management issues. specifically, it is argued that since 79 percent of the 1200 workers who responded to survey expressed interest in these topics, the notion that workers are apathetic about management issues is incorrect. the reasoning in this argument is problematic in several respects.

first, the statistics cited in the editorial may be misleading because the total number of workers employed by the corporation is not specified. for example, if the corporation employs 2000 workers, the fact that 79 percent of the nearly 1200 respondents showed interest in these topics provides strong support for the conclusion. on the other hand, if the corporation employs 200,000 workers, the conclusion is much weaker.

another problem with the argument is that the respondents’ views are not necessarily representative of the views of the work force in general. for example, because the survey has to do with apathy, it makes sense that only less apathetic workers would respond to it, thereby distorting the overall picture of apathy among the work force. without knowing how the survey was conducted, it is impossible to assess whether or not this is the case.

a third problem with the argument is that it makes a hasty generalization about the types of issues workers are interested in. it accords with common sense that workers would be interested in corporate restructuring and redesign of benefits programs, since these issues affect workers very directly. however, it is unfair to assume that workers would be similarly interested in other management issues—ones that do not affect them or affect them less directly.

in conclusion, this argument is not convincing as it stands. to strengthen it, the author would have to show that the respondents account for a significant and representative portion of all workers. additionally, the author must provide evidence of workers’ interest other management topics—not just those that affect workers directly.

9. the following appeared in the opinion column of a financial magazine.“on average, middle-aged consumers devote 39 percent of their retail expenditure to department store products and services, while for younger consumers the average is only 25 percent. since the number of middle-aged people will increase dramatically within the next decade, department stores can expect retail sales to increase significantly during that period. furthermore, to take advantage of the trend, these stores should begin to replace some of those products intended to attract the younger consumer with products intended to attract the middle-aged consumer.”

discuss how well reasoned... etc.

sample essay 1:

the argument that department retail sales will increase in the next 10 years and thus department stores should begin to replace products to attract middle-aged consumers is not entirely logically convincing, since it omits certain crucial assumptions

first of all, the argument ignores the absolute amount of retail expenditure of middle-aged and younger consumers devoted to department store products and services. although younger consumers spend a smaller percentage of their retail expenditure to department store products than do the middle-aged consumers, they might actually spend more in terms of the absolute amount.

even if middle-aged consumers are spending more than younger ones in department stores, the argument ignores the possibility that the trend may change within the next decade. younger consumers might prefer to shop in department stores than in other types of stores, and middle-aged consumers might turn to other types of stores, too. this will lead to a higher expenditure of younger consumers in department stores than that of middle-aged consumers.

besides, the argument never addresses the population difference between middle-aged consumers and younger ones. suppose there are more younger consumers than the middle-aged ones now, the total population base of younger consumers will be bigger than that of the middle-aged ones if both of them grow at the same rate in the next decade. thus there will be a bigger younger consumer base.

based on the reasons i listed above, the argument is not completely sound. the evidence in support of the conclusion does little to prove the conclusion since it does not address the assumptions i have already raised. ultimately, the argument might have been more convincing by making it clear that the absolute population of middle-aged consumers are higher than that of the younger consumers and the number will continue to grow in the next decade, and that the middle-aged consumers will continue to spend more money in department stores than younger consumers do in the next decade.

sample essay 2:

the argument that retailers should replace some of the products intended to attract the younger consumers with products intended to attract the middle-aged consumers is not entirely logically convincing, since it ignores certain crucial assumptions.

first, the argument omits the assumption that the business volumes of both the middle-aged consumers and the younger consumers are the same. if the business volume of the middle-aged consumers’ 39% is smaller than that of the younger consumers’ 25%, the retail sales will not increase during the next decade.

second, even if the business volumes of both the middle-aged consumers and the younger consumers were the same in the last decade, the increase of the middle-aged people in the next decade is not the same as the increase of the retail expenditure, for the retail trade depends more on such factors as the economic circumstances, people’s consuming desire.

finally, the argument never assumes the increase of the younger consumers within the next decade. if the younger consumers increase at the same rate and spend the same amount of money on the goods and services of department stores, the retailers should never ignore them.

thus the argument is not completely sound. the evidence in support of the conclusion that the growing number of middle-aged people within the next decade does little to prove the conclusion—that department stores should begin to replace some of their products to attract the middle-aged consumers since it does not address the assumptions i have already raised. ultimately, the argument might have been strengthened by making it clear that the business volumes of both types of consumers are the same and comparable, that the increase of a certain type of consumers are correlated with the increase of the retail sales, and that the growth rate of the younger consumers are the same as that of the middle-aged consumers.

sample essay 3:

based on an expected increase in the number of middle-aged people during the next decade, the author predicts that retail sales at department stores will increase significantly over the next ten years. to bolster this prediction, the author cites statistics showing that middle-aged people devote a much higher percentage of their retail expenditure to department-store services and products than younger consumers do. since the number of middle-aged consumers is on the rise and since they spend more than younger people on department-store goods and services, the author further recommends that department stores begin to adjust their inventories to capitalize on this trend. specifically, it is recommended that department stores increase their inventory of products aimed at middle-aged consumers and decrease their inventory of products aimed at younger consumers. this argument is problematic for two reasons.

first, an increase in the number of middle-aged people does not necessarily portend an overall increase in department-store sales. it does so only on the assumption that other population groups will remain relatively constant. for example, if the expected increase in the number of middle-aged people is offset by an equally significant decrease in the number of younger people, there will be little or no net gain in sales.

second, in recommending that department stores replace products intended to attract younger consumers with products more suitable to middle-aged consumers, the author assumes that the number of younger consumers will not also increase. since a sizable increase in the population of younger consumers could conceivably offset the difference in the retail expenditure patterns of younger and middle-aged consumers, it would be unwise to make the recommended inventory adjustment lacking evidence to support this assumption.

in conclusion, this argument is unacceptable. to strengthen the argument the author would have to provide evidence that the population of younger consumers will remain relatively constant over the next decade.

10. the following appeared in the editorial section of a local newspaper.“this past winter, 200 students from waymarsh state college traveled to the state capitol building to protest against proposed cuts in funding for various state college programs. the other 12,000 waymarsh students evidently weren’t so concerned about their education: they either stayed on campus or left for winter break. since the group who did not protest is far more numerous, it is more representative of the state’s college students than are the protesters. therefore the state legislature need not heed the appeals of the protesting students.”

discuss how well reasoned... etc.

the conclusion in this argument is that the state legislature need not consider the views of protesting students. to support this conclusion, the author points out that only 200 of the 12,000 students traveled to the state capitol to voice their concerns about proposed cuts in college programs. since the remaining students did not take part in this protest, the author concludes they are not interested in this issue. the reasoning in this argument is flawed for two reasons.

first, the author assumes that because only one-tenth of the students took part in the protest, these students’ views are unrepresentative of the entire student body. this assumption is unwarranted. if it turns out, for example, that the protesting students were randomly selected from the entire student body, their views would reflect the views of the entire college. without information regarding the way in which the protesting students were selected, it is presumptuous to conclude that their opinions fail to reflect the opinions of their colleagues.

second, the author cites the fact that the remaining 12,000 students stayed on campus or left for winter break as evidence that they are not concerned about their education. one obvious rejoinder to this line of reasoning is that the students who did not participate did so with the knowledge that their concerns would be expressed by the protesting students. in any case, the author has failed to demonstrate a logical connection between the students’ alleged lack of concern and the fact that they either stayed on campus or left for winter break. without this connection, the conclusion reached by the author that the remaining 12,000 students are not concerned about their education is unacceptable.

as it stands, the argument is not well reasoned. to make it logically acceptable, the author would have to demonstrate that the protesting students had some characteristic in common that biases their views, thereby nullifying their protest as representative of the entire college.

11. the following appeared in the editorial section of a local newspaper.“in the first four years that montoya has served as mayor of the city of san perdito, the population has decreased and the unemployment rate has increased. two businesses have closed for each new business that has opened. under varro, who served as mayor for four years before montoya, the unemployment rate decreased and the population increased. clearly, the residents of san perdito would be best served if they voted montoya out of office and reelected varro.”

discuss how well reasoned... etc.

the recommendation endorsed in this argument is that residents of san perdito vote current mayor montoya out of office, and re-elect former mayor varro. the reasons cited are that during montoya’s four years in office the population has decreased while unemployment has increased, whereas during varro’s term unemployment declined while the population grew. this argument involves the sort of gross oversimplification and emotional appeal typical of political rhetoric; for this reason it is unconvincing.

first of all, the author assumes that the montoya administration caused the unemployment in san perdito as well as its population loss. the line of reasoning is that because montoya was elected before the rise in unemployment and the decline in population, the former event caused the latter. but this is fallacious reasoning unless other possible causal explanations have been considered and ruled out. for example, perhaps a statewide or nationwide recession is the cause of these events. or perhaps the current economic downturn is part of a larger picture of economic cycles and trends, and has nothing to do with who happens to be mayor. yet another possibility is that varro enjoyed a period of economic stability and varro’s own administration set the stage for the unemployment and the decline in population the city is now experiencing under montoya.

secondly, job availability and the economic health of one’s community are issues that affect people emotionally. the argument at hand might have been intentionally oversimplified for the specific purpose of angering citizens of san perdito, and thereby turning them against the incumbent mayor. arguments that bypass relevant, complex reasoning in favor of stirring up emotions do nothing to establish their conclusions; they are also unfair to the parties involved.

in conclusion, i would not cast my vote for varro on the basis of this weak argument. the author must provide support for the assumption that mayor montoya has caused san perdito’s poor economy. moreover, such support would have to involve examining and eliminating other possible causal factors. only with more convincing evidence could this argument become more than just an emotional appeal.

12. the following appeared as part of a promotional campaign to sell advertising space in the daily gazette to grocery stores in the marston area.“advertising the reduced price of selected grocery items in the daily gazette will help you increase your sales. consider the results of a study conducted last month. thirty sale items from a store in downtown marston were advertised in the gazette for four days. each time one or more of the 30 items was purchased, clerks asked whether the shopper had read the ad. two-thirds of the 200 shoppers asked answered in the affirmative. furthermore, more than half the customers who answered in the affirmative spent over $100 at the store.”

discuss how well reasoned... etc.

the conclusion of this argument is that advertising the reduced price of selected items in the daily gazette will result in increased sales overall. to support it, the author cites an informal poll conducted by sales clerks when customers purchased advertised items. each time one or more of the advertised items was sold, the clerks asked whether the customer had read the ad. it turned out that two-thirds of 200 shoppers questioned said that they had read the ad. in addition, of those who reported reading the ad, more than half spent over $100 in the store. this argument is unconvincing for two reasons.

to begin with, the author’s line of reasoning is that the advertisement was the cause of the purchase of the sale items. however, while the poll establishes a correlation between reading the ad and purchasing sale items, and also indicates a correlation, though less significantly, between reading the ad and buying non-sale items, it does not establish a general causal relationship between these events. to establish this relationship, other factors that could bring about this result must be considered and eliminated. for example, if the four days during which the poll was conducted preceded thanksgiving and the advertised items were traditionally associated with this holiday, then the results of the poll would be extremely biased and unreliable.

moreover, the author assumes that the poll indicates that advertising certain sale will cause a general increase in sales. but the poll does not even address the issue of increased overall sales; it informs us mainly that, of the people who purchased sales items, more had read the ad than not. a much clearer indicator of the ad’s effectiveness would be a comparison of overall sales on days the ad ran with overall sales on otherwise similar days when the ad did not run.

in sum, this argument is defective mainly because the poll does not support the conclusion that sales in general will increase when reduced-price products are advertised in the daily gazette. to strengthen the argument, the author must, at the very least, provide comparisons of overall sales reports as described above.

13. the following appeared as part of a campaign to sell advertising time on a local radio station to local businesses.“the cumquat cafe began advertising on our local radio station this year and was delighted to see its business increase by 10 percent over last year’s totals. their success shows you how you can use radio advertising to make your business more profitable.”

discuss how well reasoned... etc.

in an attempt to sell radio advertising time, this ad claims that radio advertising will make businesses more profitable. the evidence cited is a ten percent increase in business that the cumquat cafe has experienced in the year during which it advertised on the local radio station. this argument is unconvincing because two questionable assumptions must be made for the stated evidence to support the author’s conclusion.

the first assumption is that radio advertising alone has caused the increase in business at the cumquat cafe. this assumption is questionable because it overlooks a number of other factors that might have contributed to the cumquat’s success. for example, the cumquat might have changed owners or chefs; it might have launched a coupon ad campaign in the local print media; or it might have changed or updated the menu. yet another possibility is that a local competitor went out of business. these are just a few of the factors that could help explain the cumquat’s growth. because the author fails to eliminate these possibilities, the assumption in question need not be accepted.

even if it is granted that radio advertising is responsible for the cumquat’s success, another assumption must be made before we can conclude that radio advertising will result in increased profits for businesses in general. we must also assume that what is true of the cumquat will likewise be true of most other businesses. but there are all kinds of important differences between cafes and other businesses that could affect how radio audiences react to their advertising. we cannot safely assume that because a small restaurant has benefited from radio advertising, any and all local businesses will similarly benefit.

in conclusion, it would be imprudent for a business to invest in radio advertising solely on the basis of the evidence presented. to strengthen the conclusion, it must be established that radio advertising was the principal cause of increased business at the cumquat. once this is shown, it must be determined that the business in question is sufficiently like the cumquat, and so can expect similar returns from investment in radio ad time.

14. the following appeared as part of a newspaper editorial.“two years ago nova high school began to use interactive computer instruction in three academic subjects. the school dropout rate declined immediately, and last year’s graduates have reported some impressive achievements in college. in future budgets the school board should use a greater portion of the available funds to buy more computers, and all schools in the district should adopt interactive computer instruction throughout the curriculum.”

discuss how well reasoned... etc.

sample essay 1:

the argument that the school board should buy more computers and adopt interactive computer instruction is not entirely logically convincing, since it ignores certain crucial assumptions.

first, the argument assumes that the decline of school dropout and the achievements of last year’s graduates’ results from the adoption of interactive computer instruction. however, there are several reasons why this might not be true. for example, achievements could have been made in other subjects than the ones with interactive computer instruction. or last years’ graduates might not have been given the interactive computer instruction. or the decline of the rate of dropout could be attributed to stricter discipline applied last year.

second, even supposing the nova high school’s decline of the dropout and last year’s graduates’ achievements benefit directly from the usage of interactive computer instruction, the success of the instruction in one school may not ensure the success in other schools. if it does not suit other schools, the instruction will not work.

finally, even if the decline of the rate of dropout and the achievements of the last year’s graduates’ are the direct results of the interactive computer instruction, we still do not know whether the school can afford to apply the instruction on all the subjects or to all the students. if the school does not have sufficient fund and has to cut budgets on other projects such as the library, the quality of the school’s education will also compromise.

thus, the argument is not completely sound. the evidence in support of the conclusion that the dropout rate declined and last year’s graduates made impressive achievements does little to prove the conclusion that other schools should use a greater portion of their funds to apply the instruction since it does not address the assumptions i have already raised. ultimately, the argument might have been strengthened by making it clear that the decline of the dropout rate and the achievements of the graduates are the direct results of interactive computer instruction, that the instruction is also applicable to other schools in the district, and that the instruction is affordable to all the schools in the district.

sample essay 2:

the editorial recommends that the school board of nova high spend a greater portion of available funds on the purchase of additional computers and adopt interactive computer instruction throughout the curriculum. two reasons are offered in support of this recommendation. first, the introduction of interactive computer instruction in three academic subjects was immediately followed by a decline in the school dropout rate. second, last year’s graduates experienced impressive achievements in college. this argument is unconvincing for two reasons.

to begin with, this argument is a classic instance of “after this, therefore because of this” reasoning. the mere fact that the introduction of interactive computer instruction preceded the impressive performance of recent graduates and the decline in the dropout rate is insufficient to conclude that it was the cause of these events. many other factors could bring about these same results. for example, the school may have implemented counseling and training programs that better meet the needs of students who might otherwise leave school to take jobs. in addition, the school may have introduced programs to better prepare students for college.

secondly, the author assumes that the impressive achievements of last year’s graduates bear some relation to the introduction of interactive computer instruction at nova high. however, no evidence is offered to support this assumption. lacking evidence that links the achievements of the recent graduates to the interactive instruction, it is presumptuous to suggest that the computer instruction was in some way responsible for the students’ impressive performance.

in conclusion, the recommendation that nova high spend a greater portion of available funds on the purchase of additional computers and adopt interactive computer instruction throughout the curriculum is ill-founded. to strengthen this recommendation the author would have to demonstrate that the decline in the dropout rate and the impressive performance of recent graduates came about as a result of the use of computer-interactive instruction. all that has been shown so far is a correlation between these events.

15. the following appeared as a part of an advertisement for adams, who is seeking reelection as governor.“re-elect adams, and you will be voting for proven leadership in improving the state’s economy. over the past year alone, seventy percent of the state’s workers have had increases in their wages, five thousand new jobs have been created, and six corporations have located their headquarters here. most of the respondents in a recent poll said they believed that the economy is likely to continue to improve if adams is reelected. adams’s opponent, zebulon, would lead our state in the wrong direction, because zebulon disagrees with many of adams’s economic policies.”

discuss how well reasoned... etc.

this political advertisement recommends re-electing governor adams because he has a proven leadership role in improving the state’s economy. in support of this reason the author cites these statistics: in the past year, most state workers’ wages have gone up; 5,000 new jobs have been created; and six corporations have located in the state. another reason offered for re-electing adams is a recent poll, which indicates that most respondents believe the state economy would continue to improve if he were re-elected. finally, the author claims that rival zebulon would harm the state’s economy because he disagrees with adams’ fiscal policies. this argument is fraught with vague, oversimplified and unwarranted claims.

to begin with, the statistics are intended to support the main claim that the state is economically better off with adams as governor. but these statistics are vague and oversimplified, and thus may distort the state’s overall economic picture. for example, state workers’ pay raises may have been minuscule and may not have kept up with cost of living or with pay for state workers in other states. moreover, the 5,000 new jobs may have been too few to bring state unemployment rates down significantly; at the same time, many jobs may have been lost. finally, the poll indicates that six new corporations located in the state, but fails to indicate if any left.

next, the poll cited by the author is described in the vaguest possible terms. the ad does not indicate who conducted the poll, who responded, or how the poll was conducted. until these questions are answered, the survey results are worthless as evidence for public opinion about adams or his economic policies.

finally, while we have only vague and possibly distorted evidence that the state is better off with adams, we have absolutely no evidence that it would be worse off with zebulon. given that the state economy is good at the moment, none of the author’s reasons establishes that adams is the cause of this. and neither do they establish that the state wouldn’t be even better off with someone else in office.

in conclusion, this argument is weak. to strengthen the argument, the author must provide additional information about the adequacy of state workers’ pay raises, the effect of the 5,000 jobs on the state’s employment picture, the overall growth of corporations in the state, and other features of the state economy. also, the author must support the claims that adams’ actions have caused any economic improvement and that in the future adams will impart more economic benefit than would zebulon.

16. the following appeared as part of an article in the education section of a waymarsh city newspaper.“throughout the last two decades, those who earned graduate degrees found it very difficult to get jobs teaching their academic specialties at the college level. those with graduate degrees from waymarsh university had an especially hard time finding such jobs. but better times are coming in the next decade for all academic job seekers, including those from waymarsh. demographic trends indicate that an increasing number of people will be reaching college age over the next ten years; consequently, we can expect that the job market will improve dramatically for people seeking college-level teaching positions in their fields.”

discuss how well reasoned... etc.

demographic trends that indicate an increase in the number of college-aged people over the next ten years lead the author to predict an improved job market for all people seeking college-level teaching positions in their academic disciplines. moreover, the author argues that since waymarsh university students with advanced degrees had an especially difficult time finding teaching jobs in the past, these trends portend better times ahead for waymarsh graduates. this argument is problematic in three important respects.

first, the author assumes that an increase in the number of college-aged people over the next decade will necessarily result in an increase in the number of people who attend college during this period. while this is a reasonable assumption, it is by no means a certainty. for example, a world war or economic depression in the next decade would certainly nullify this expectation.

second, even if we grant the preceding assumption, we must also consider the additional assumption that increased university enrollments will lead to an increase in teaching positions in all fields. however, it might turn out that some teaching specialties are in greater demand than others in the future, resulting in a disproportionate number of teaching positions available in various fields. consequently, persons trained in some fields might find it more difficult, if not impossible, to find teaching jobs in the future.

finally, little can be foretold regarding the employability of waymarsh graduates in the future based on the information provided in the argument. lacking information about the reasons why waymarsh graduates had an especially difficult time finding teaching jobs, it is difficult to assess their prospects for the future. it is probable, however, that since waymarsh has had an especially hard time placing graduates in the past, the mere fact that more jobs are available will not, by itself, ensure that waymarsh graduates will have an easier time finding teaching jobs during the next decade.

in conclusion, this argument is unconvincing. to strengthen the argument, the author must provide evidence that the only major trend in the next decade will be an increase in the number of people reaching college age. regarding the future prospects for waymarsh graduates, the author must provide evidence that there were no idiosyncratic reasons that prevented them from finding jobs in the past.

17. the following appeared in an article in a consumer-products magazine.“two of today’s best-selling brands of full-strength prescription medication for the relief of excess stomach acid, acid-ease and pepticaid, are now available in milder nonprescription forms. doctors have written 76 million more prescriptions for full-strength acid-ease than for full-strength pepticaid. so people who need an effective but milder nonprescription medication for the relief of excess stomach acid should choose acid-ease.”

discuss how well reasoned... etc.

this ad recommends non-prescription acid-ease over non-prescription pepticaid for relief of excess stomach acid. the only reason offered is that doctors have written 76 million more prescriptions for the full-strength prescription form of acid-ease than for full-strength pepticaid. while this reason is relevant, and provides some grounds for preferring acid-ease over pepticaid, it is insufficient as it stands because it depends on three unwarranted assumptions.

the first assumption is that the prescription form of acid-ease is more popular among doctors. but this might not be the case, even though doctors have written 76 million more prescriptions for acid-ease. acid-ease may have been available for several more years than pepticaid; and in the years when both products were available, pepticaid might have actually been prescribed more often than acid-ease.

the second assumption is that doctors prefer the prescription form of acid-ease for the reason that it is in fact more effective at relieving excess stomach acid. however, doctors may have preferred acid-ease for reasons other than its effectiveness. perhaps acid-ease is produced by a larger, more familiar drug company or by one that distributes more free samples. for that matter, the medical community may have simply been mistaken in thinking that acid-ease was more effective. in short, the number of prescriptions by itself is not conclusive as to whether one product is actually better than another.

the third assumption is that the milder non-prescription forms of acid-ease and pepticaid will be analogous to the full-strength prescription forms of each. but this might not be the case. suppose for the moment that the greater effectiveness of prescription acid-ease has been established; even so, the non-prescription form might not measure up to non-prescription pepticaid. this fact must be established independently.

in conclusion, this ad does not provide enough support for its recommending non-prescription acid-ease over non-prescription pepticaid. to strengthen its argument, the promoter of acid-ease would have to show that (1) the comparison between the number of prescriptions is based on the same time period; (2) its effectiveness is the main reason more doctors have prescribed it, and (3) the comparative effectiveness of the two non-prescription forms is analogous to that of the prescription forms.

18. the following is an excerpt from a memo written by the head of a governmental department.“neither stronger ethics regulations nor stronger enforcement mechanisms are necessary to ensure ethical behavior by companies doing business with this department. we already have a code of ethics that companies doing business with this department are urged to abide by, and virtually all of these companies have agreed to follow it. we also know that the code is relevant to the current business environment because it was approved within the last year, and in direct response to specific violations committed by companies with which we were then working—not in abstract anticipation of potential violations, as so many such codes are.”

discuss how well reasoned... etc.

in this argument, the head of a government department concludes that the department does not need to strengthen either its ethics regulations or its enforcement mechanisms in order to encourage ethical behavior by companies with which it does business. the first reason given is that businesses have agreed to follow the department’s existing code of ethics. the second reason is that the existing code is relevant to the current business environment. this argument is unacceptable for several reasons.

the sole support for the claim that stronger enforcement mechanisms are unnecessary comes from the assumption that companies will simply keep their promises to follow the existing code. but, since the department head clearly refers to rules violations by these same businesses within the past year, his faith in their word is obviously misplaced. moreover, it is commonly understood that effective rules carry with them methods of enforcement and penalties for violations.

to show that a strengthened code is unnecessary, the department head claims that the existing code of ethics is relevant. in partial clarification of the vague term “relevant,” we are told that the existing code was approved in direct response to violations occurring in the past year. if the full significance of being relevant is that the code responds to last year’s violations, then the department head must assume that those violations will be representative of all the kinds of ethics problems that concern the department. this is unlikely; in addition, thinking so produces an oddly short-sighted idea of relevance.

such a narrow conception of the relevance of an ethics code points up its weakness. the strength of an ethics code lies in its capacity to cover many different instances of the general kinds of behavior thought to be unethical—to cover not only last year’s specific violations, but those of previous years and years to come. yet this author explicitly rejects a comprehensive code, preferring the existing code because it is “relevant” and “not in abstract anticipation of potential violations.”

in sum, this argument is naive, vague and poorly reasoned. the department head has not given careful thought to the connection between rules and their enforcement, to what makes an ethics code relevant, or to how comprehensiveness strengthens a code. in the final analysis, he adopts a backwards view that a history of violations should determine rules of ethics, rather than the other way around.

19. the following appeared as part of an article in the travel section of a newspaper.“over the past decade, the restaurant industry in the country of spiessa has experienced unprecedented growth. this surge can be expected to continue in the coming years, fueled by recent social changes: personal incomes are rising, more leisure time is available, single-person households are more common, and people have a greater interest in gourmet food, as evidenced by a proliferation of publications on the subject.”

discuss how well reasoned... etc.

recent social changes in the country of spiessa lead the author to predict a continued surge in growth of that country’s restaurant industry. rising personal incomes, additional leisure time, an increase in single-person households, and greater interest in gourmet food are cited as the main reasons for this optimistic outlook. all of these factors are indeed relevant to growth in the restaurant industry; so the prediction appears reasonable on its face. however, three questionable assumptions operative in this argument bear close examination.

the first dubious assumption is that the supply of restaurants in spiessa will continue to grow at the same rate as in the recent past. however, even in the most favorable conditions and the best of economic times there are just so many restaurants that a given population can accommodate and sustain. it is possible that the demand for restaurants has already been met by the unprecedented growth of the past decade, in which case the recent social changes will have little impact on the growth of the restaurant industry.

a second assumption is that the economic and social circumstances cited by the author will actually result in more people eating out at restaurants. this assumption is unwarranted, however. for example, increased leisure time may just as likely result in more people spending more time cooking gourmet meals in their own homes. also, single people may actually be more likely than married people to eat at home than to go out for meals. finally, people may choose to spend their additional income in other ways—on expensive cars, travel, or larger homes.

a third poor assumption is that, even assuming people in spiessa will choose to spend more time and money eating out, no extrinsic factors will stifle this demand. this assumption is unwarranted. any number of extrinsic factors—such as a downturn in the general economy or significant layoffs at spiessa’s largest businesses—may stall the current restaurant surge. moreover, the argument fails to specify the “social changes” that have led to the current economic boom. if it turns out these changes are politically driven, then the surge may very well reverse if political power changes hands.

in conclusion, this argument unfairly assumes a predictable future course for both supply and demand. to strengthen the argument, the author must at the very least show that demand for new restaurants has not yet been exhausted, that spiessa can accommodate new restaurants well into the future, and that the people of spiessa actually want to eat out more.

20. the following appeared in an article in a health and fitness magazine.“laboratory studies show that saluda natural spring water contains several of the minerals necessary for good health and that it is completely free of bacteria. residents of saluda, the small town where the water is bottled, are hospitalized less frequently than the national average. even though saluda natural spring water may seem expensive, drinking it instead of tap water is a wise investment in good health.”

discuss how well reasoned... etc.

sample essay 1:

the argument that drinking saluda natural spring water instead of tap water is a wise investment in good health is not entirely logically convincing, since it lacks certain supporting factors.

firstly, the argument assumes that saluda natural spring water is the major reason why residents of saluda are less frequently hospitalized than the national average. however, there is little evidence that this water is the only difference between this place and the rest of the country. and the reason why people in other places are more hospitalized are numerous and varied. there are so many other factors that would bring people in other places to hospitals, such as accidents, food contamination, illnesses, etc.

secondly, the argument also assumes that the minerals in saluda national spring water are the key minerals for the good health of the residents of saluda. however, this may not be true. we need not only minerals to keep good heath but also various vitamins. besides, our body needs more minerals than those contained in saluda natural spring water.

finally, even if the saluda water is the major reason why the residents of saluda are less hospitalized, the argument still omits the fact that there is more than one way to keep drinking water free from bacteria. for instance, the most common practice is to boil water up to 100 degree celsius and keep it at that degree for more than 5 minutes. therefore drinking saluda water to keep good health is not the only alternative.

thus, the argument is not completely sound. the evidence in support of the conclusion that the saluda residents are less hospitalized does little to prove the conclusion that drinking saluda natural spring water is a wise investment in good health since it omits the assumptions i have just raised. the argument might have been strengthened by making it plain that saluda natural spring water is the major reason why the residents of saluda are less hospitalized, that the water contains all the major minerals essential for the human body, and that there is no other way to keep water from bacteria.

sample essay 2:

in this argument the author concludes that drinking saluda natural spring water (snsw) is preferable to drinking tap water. three reasons are offered in support of this conclusion: snsw contains several of the minerals necessary for good health, it is completely tree of bacteria, and residents of saluda—the town where it is bottled—are hospitalized less frequently than the national average. this argument is unconvincing because it relies on a variety of dubious assumptions.

the first questionable assumption underlying this argument that tap water does not contain the minerals in question and is not completely free of bacteria. this assumption is not supported in the argument. if tap water is found to contain the same minerals and to be free of bacteria, the author’s conclusion is substantially undermined.

a second assumption of the argument is that the water residents of saluda drink is the same as snsw. lacking evidence to the contrary, it is possible that saluda is not the source of the bottled water but is merely the place where snsw is bottled. no evidence is offered in the argument to dispute this possibility.

finally, it is assumed without argument that the reason residents are hospitalized less frequently than the national average is that they drink snsw. again, no evidence is offered to support this assumption. perhaps the residents are hospitalized less frequently because they are younger than the national average, because they are all vegetarians, or because they exercise daily. that is, there might be other reasons than the one cited to account for this disparity.

in conclusion, this is an unconvincing argument. to strengthen the conclusion that snsw is more healthful than tap water, the author must provide evidence that tap water contains harmful bacteria not found in snsw. moreover, the author must demonstrate that the residents of saluda regularly drink the same water as snsw and that this is why they are hospitalized less frequently than the national average.

21. the following appeared as part of an editorial in an industry newsletter.“while trucking companies that deliver goods pay only a portion of highway maintenance costs and no property tax on the highways they use, railways spend billions per year maintaining and upgrading their facilities. the government should lower the railroad companies’ property taxes, since sending goods by rail is clearly a more appropriate mode of ground transportation than highway shipping. for one thing, trains consume only a third of the fuel a truck would use to carry the same load, making them a more cost-effective and environmentally sound mode of transport. furthermore, since rail lines already exist, increases in rail traffic would not require building new lines at the expense of taxpaying citizens.”

discuss how well reasoned... etc.

the conclusion of this editorial is that the government should lower property taxes for railroad companies. the first reason given is that railroads spend billions per year maintaining and upgrading their facilities. the second reason is that shipping goods by rail is cost-effective and environmentally sound. this argument is unconvincing for several reasons.

first of all, the argument depends upon a misleading comparison between railroad and truck company expenditures. although trucking companies do not pay property tax on roads they use, they do pay such taxes on the yards, warehouses and maintenance facilities they own. and while trucking companies pay only a portion of road maintenance costs, this is because they are not sole users of public roads. railroad companies shoulder the entire burden of maintenance and taxes on their own facilities and tracks; but they distribute these costs to other users through usage fees.

in addition, the author assumes that property taxes should be structured to provide incentives for cost-effective and environmentally beneficial business practices. this assumption is questionable because property taxes are normally structured to reflect the value of property. moreover, the author seems to think that cost-effectiveness and environmental soundness are equally relevant to the question of tax relief. however, these are separate considerations. the environmental soundness of a practice might be relevant in determining tax structuring, but society does not compensate a business for its cost-efficiency.

splitting the issues of cost-efficiency and environmental impact highlights an ambiguity in the claim that railway shipping is more appropriate. on the one hand, it may be appropriate, or prudent, for me to ship furniture by rail because it is cost-effective; on the other hand, it might be appropriate, or socially correct, to encourage more railway shipping because it is environmentally sound. the argument thus trades on an equivocation between social correctness on the one hand, and personal or business prudence on the other.

in sum, this argument is a confusion of weak comparisons, mixed issues and equivocal claims. i would not accept the conclusion without first determining: (1) the factors relevant to tax structure, (2) whether specific tax benefits should accrue to property as well as to income and capital gains taxes, (3) whether railway shipping really does provide greater social benefits, and (4) whether it is correct to motivate more railway shipping on this basis.

22. the following appeared in the editorial section of a newspaper.“as public concern over drug abuse has increased, authorities have become more vigilant in their efforts to prevent illegal drugs from entering the country. many drug traffickers have consequently switched from marijuana, which is bulky, or heroin, which has a market too small to justify the risk of severe punishment, to cocaine. thus enforcement efforts have ironically resulted in an observed increase in the illegal use of cocaine.”

discuss how well reasoned... etc.

the conclusion in this argument is that increased vigilance by drug enforcement authorities has resulted in an increase in the illegal use of cocaine. the author reaches this conclusion on the grounds that drug traffickers have responded to increased enforcement efforts by switching from bulkier and riskier drugs to cocaine. presumably, the author’s reasoning is that the increased enforcement efforts inadvertently brought about an increase in the supply of cocaine which, in turn, brought about the observed increase in the illegal use of cocaine. this line of reasoning is problematic in two important respects.

in the first place, the author has engaged in “after this, therefore because of this” reasoning. the only reason offered for believing that the increased vigilance caused the increase in cocaine use is the fact that the former preceded the latter. no additional evidence linking the two events is offered in the argument, thus leaving open the possibility that the two events are not causally related but merely correlated. this in turn leaves open the possibility that factors other than the one cited are responsible for the increase in cocaine use.

in the second place, the author assumes that an increase in the supply of cocaine is sufficient to bring about an increase in its use. while this is a tempting assumption, it is a problematic one. the presumption required to substantiate this view is that drug users are not particular about which drugs they use, so that if marijuana and heroin are not available, they will switch to whatever drug is available—cocaine in this case. the assumption does not seem reasonable on its face. marijuana, heroin, and cocaine are not alike in their effects on users; nor are they alike in the manner in which they are ingested or in their addictive properties. the view that drug users’ choice of drugs is simply a function of supply overlooks these important differences.

in conclusion, the author has failed to establish a causal link between increased enforcement efforts and the observed increase in illegal cocaine use. while the enforcement activities may have been a contributing factor, to show a clear causal connection the author must examine and rule out various other factors.

23. the following appeared in a speech delivered by a member of the city council.“twenty years ago, only half of the students who graduated from einstein high school went on to attend a college or university. today, two thirds of the students who graduate from einstein do so. clearly, einstein has improved its educational effectiveness over the past two decades. this improvement has occurred despite the fact that the school’s funding, when adjusted for inflation, is about the same as it was twenty years ago. therefore, we do not need to make any substantial increase in the school’s funding at this time.”

discuss how well reasoned... etc.

this speaker draws the conclusion that there is no need to substantially increase funding for einstein high school. to support this conclusion, the speaker claims that einstein has improved its educational efficiency over the past 20 years, even though funding levels have remained relatively constant. his evidence is that two-thirds of einstein’s graduates now go on to college, whereas 20 years ago only half of its students did so. this argument suffers from several critical problems.

to begin with, we must establish the meaning of the vague concept “educational efficiency.” if the term is synonymous with the rate of graduation to college, then the statistics cited would strongly support the argument. but, normally we are interested in something more than just the numbers of students who go on to college from a high school; we also want to know how well the school has prepared students for a successful college experience—that is, whether the school has provided a good secondary education. thus, for the speaker the term “educational efficiency” must essentially carry the same meaning as “educational quality.”

given this clarification, one of the speaker’s assumptions is that the rate of graduation to college has increased because einstein is doing a better job of educating its students. however, the fact that more einstein graduates now go on to college might simply reflect a general trend. and the general trend might have less to do with improved secondary education than with the reality that a college degree is now the standard of entry into most desirable jobs.

but even if the quality of education at einstein had improved, would this be a compelling reason to deny einstein additional funding? i don’t think so. it is possible that the school has managed to deliver better education in spite of meager funding. teachers may be dipping into their own pockets for supplies and other resources necessary for doing their job well. perhaps the quality of education at einstein would improve even more with additional financial support.

in sum, this argument does not establish the conclusion that additional funding for einstein is unnecessary. to do so, the speaker would have to provide evidence that the quality of education at einstein has improved. this could be done by examining student assessment scores or by tracking students through their college careers to see how many successfully graduate and find jobs. in addition, the speaker would also have to show that einstein is doing a good job with adequate financial support, and not merely in spite of insufficient funding.

24. the following appeared in a memo from the customer service division to the manager of mammon savings and loan.“we believe that improved customer service is the best way for us to differentiate ourselves from competitors and attract new customers. we can offer our customers better service by reducing waiting time in teller lines from an average of six minutes to an average of three. by opening for business at 8:30 instead of 9:00, and by remaining open for an additional hour beyond our current closing time, we will be better able to accommodate the busy schedules of our customers. these changes will enhance our bank’s image as the most customer-friendly bank in town and give us the edge over our competition.”

discuss how well reasoned... etc.

the customer-service division of mammon savings and loan recommends that the best way for the bank to attract new customers and differentiate itself from its competitors is to improve its service to customers—specifically, by reducing waiting time in teller lines, opening for business 30 minutes earlier, and closing an hour later. these improvements, it is argued, will give the bank the edge over its competitors and make it appear more customer-friendly. for the most part this recommendation is well-reasoned; a few concerns must be addressed, however.

first, the author assumes that mammon’s competitors are similar to mammon in all respects other than the ones listed. in fact, mammon’s competitors may be more conveniently located to customers, or offer other services or products on more attractive terms than mammon. if so, mammon may not gain the edge it seeks merely by enhancing certain services.

secondly, the author assumes that the proposed improvements will sufficiently distinguish mammon from its competitors. this is not necessarily the case. mammon’s competitors may already offer, or may plan to offer, essentially the same customer-service features as those mammon proposes for itself. if so, mammon may not gain the edge it seeks merely by enhancing these services.

thirdly, the author assumes that mammon can offer these improved services without sacrificing any other current features that attract customers. in fact, mammon may have to cut back other services or offer accounts on less attractive terms, all to compensate for the additional costs associated with the proposed improvements. by rendering its other features less attractive to customers, mammon may not attain the competitive edge it seeks.

in conclusion, mammon’s plan for attracting new customers and differentiating itself from its competitors is only modestly convincing. while improvements in customer service generally tend to enhance competitiveness, it is questionable whether the specific improvements advocated in the recommendation are broad enough to be effective.

25. the following appeared as part of an article in a magazine on lifestyles.“two years ago, city l was listed 14th in an annual survey that ranks cities according to the quality of life that can be enjoyed by those living in them. this information will enable people who are moving to the state in which city l is located to confidently identify one place, at least, where schools are good, housing is affordable, people are friendly, the environment is safe, and the arts flourish.”

discuss how well reasoned... etc.

the author concludes that city l has good schools, affordable housing, friendly people, flourishing arts and a safe environment. to support this claim the author cites an annual survey that ranks cities according to quality of life. two years ago city l was listed 14th in this survey. as it stands this argument is unconvincing.

first, the author fails to indicate what individual characteristics of cities were used as criteria for the ranking. to the extent that the criteria used in the survey were the same as the features listed by the author in the conclusion, the conclusion would be warranted. on the other hand, if the survey employed entirely different criteria—for example, outdoor recreational opportunities or educational achievement levels of adult residents—then the author’s conclusion would be wholly unwarranted.

secondly, the author provides no indication of how each characteristic was weighted in the ranking. for example, city l may have far and away the most flourishing arts scene among the cities surveyed, but it may have poor schools, unfriendly people, and an unsafe environment. the extent to which the survey accurately reflects city l’s overall quality of life in this case would depend largely on the relative weight placed on the arts as a factor affecting quality of life.

thirdly, the author fails to indicate how many cities were included in the survey. th more cities included in the survey, the stronger the argument—and vice versa. for example, if 2,000 cities were surveyed, then city l would rank in the top one percent in terms of quality of life. on the other hand, if only 14 cities were surveyed then city l would rank last.

finally, the author’s conclusion depends on the questionable assumption that the conditions listed by the author have remained unchanged in city l since the survey was conducted two years ago. admittedly, had ten years elapsed the argument would be even weaker. yet two years is sufficient time for a significant change in the overall economy, the city’s fiscal policies, its financial condition, or its political climate. any of these factors can affect the quality of schools, the extent to which art is flourishing, or the cost of housing.

in conclusion, the author does not adequately support the conclusion. to strengthen the argument, the author must show that the criteria used in the survey were the same as the features listed in the conclusion and were weighted in a way that does not distort the picture in city l. to better assess the argument, we would also need more information about the cities included in the survey, as well as what changes in city l have occurred during the past two years.

26. the following appeared in a memorandum from a member of a financial management and consulting firm.“we have learned from an employee of windfall, ltd., that its accounting department, by checking about ten percent of the last month’s purchasing invoices for errors and inconsistencies, saved the company some $10,000 in overpayments. in order to help our clients increase their net gains, we should advise each of them to institute a policy of checking all purchasing invoices for errors. such a recommendation could also help us get the windfall account by demonstrating to windfall the rigorousness of our methods.”

discuss how well reasoned... etc.

sample essay 1:

the argument that checking all purchasing invoices for errors will not only increase the net gains of the clients but also help the firm get the windfall account is not entirely logically convincing, since it ignores certain crucial assumptions.

first, the argument assumes that instituting a policy of checking all purchasing invoices can help find out the errors and inconsistencies. there are a number of reasons why this might not be true. for example, the people who check the accounts will probably make mistakes as anyone else, intentionally or unconsciously. if they do, checking purchasing invoices will not help avoid errors and inconsistencies.

second, even if the checking can help avoid errors and inconsistencies, it will not necessarily save money for the company, for it is also likely that the accounting department makes no mistakes. if they have no errors, how can the checking save money for the company.

finally, even supposing the checking does save money for the company, the argument ignores the fact that the checking itself costs the company money. if the cost is more than the gains from the errors, the company will lose money.

thus, the argument is not completely sound. the evidence in support of the conclusion that the checking of last month’s invoices has saved the company $10,000 does little prove the conclusion that checking all purchasing invoices for errors will not only increase the net gains of the clients but also help the firm get the windfall account since it does not address the assumptions i have already raised. ultimately, the argument might have been strengthened by making it plain that checking all invoices will surely help find out errors, that all the invoices are bound to contain errors, and that the checking itself will not cost much.

sample essay 2:

in this argument a member of a financial management and consulting firm reasons that since windfall ltd. increased its net gains by checking 10 percent of its purchasing invoices for errors, it would be a good idea to advise the firm’s clients to institute a policy of checking all purchasing invoices for errors. two potential benefits are foreseen from this recommendation: it could help the firm’s clients increase their net gains, and it could help the firm land the windfall account. the member’s argument is unconvincing for a couple of reasons.

the main problem with the argument is that the conclusion is based upon insufficient evidence. the fact that some of windfall’s purchasing invoices contained errors might simply be attributable to the sloppy accounting practices of windfall’s suppliers. thus, rather than indicating a general problem, the invoice errors might simply be indicative of a problem that is specific to windfall ltd. in other words, the evidence drawn from windfall’s experience is insufficient to support the conclusion that all purchasing invoices are subject to similar errors.

secondly, the evidence offered in the argument suggests only that companies purchasing from the same suppliers that windfall purchases from are likely to experience similar problems. if the firm’s clients do not purchase from windfall’s suppliers, checking for errors might turn out to be a monumental waste of time.

in conclusion, the author’s argument fails to provide good grounds for instituting the policy of routinely checking purchasing invoices for errors. to strengthen the conclusion the author would have to provide evidence that this is a widespread problem. specifically, what is required are additional instances of purchasing invoices containing errors that are drawn from various companies.

27. the following appeared in a newspaper editorial.“as violence in movies increases, so do crime rates in our cities. to combat this problem we must establish a board to censor certain movies, or we must limit admission to persons over 21 years of age. apparently our legislators are not concerned about this issue since a bill calling for such actions recently failed to receive a majority vote.”

discuss how well reasoned... etc.

based upon a correlation between increases in movie violence and crime rates in cities, the author argues that to combat crime in cities we must either censor movies that contain violence or prohibit people who are under 21 years of age from viewing them. the author further argues that because legislators failed to pass a bill calling for these alternatives, they are not concerned with the problem of crime in our cities. the author’s reasoning is unconvincing, since it suffers from two critical problems.

to begin with, the author’s solution to the problem rests on the claim that portrayals of violence in movies are the cause of crime in the cities. however, the evidence offered is insufficient to support this claim. a mere positive correlation between movie violence and city crime rates does not necessarily prove a causal relationship. in addition, all other prospective causes of city crime such as poverty or unemployment must be ruled out. as it stands, the author’s solution to the problem is based upon an oversimplified analysis of the issue.

another problem with the argument is that the author’s solution assumes that only persons under 21 years of age are adversely affected by movie violence. ultimately, this means that the author is committed to the view that, for the most part, the perpetrators of crime in cities are juveniles under 21. lacking evidence to support this view, the author’s solution cannot be taken seriously.

in conclusion, the best explanation of the failure of the bill calling for the actions proposed in this argument is that most legislators were capable of recognizing the simplistic analysis of the problem upon which these actions are based. rather than providing a demonstration of a lack of concern about this issue, the legislators’ votes reveal an understanding of the complexities of this problem and an unwillingness to accept simple solutions.

28. the following appeared in the editorial section of a local newspaper.“commuter use of the new subway train is exceeding the transit company’s projections. however, commuter use of the shuttle buses that transport people to the subway stations is below the projected volume. if the transit company expects commuters to ride the shuttle buses to the subway rather than drive there, it must either reduce the shuttle bus fares or increase the price of parking at the subway stations.”

discuss how well reasoned... etc.

the author concludes that the local transit company must either reduce fares for the shuttle buses that transport people to their subway stations or increase parking fees at the stations. the reasons offered to support this conclusion are that commuter use of the subway train is exceeding the transit company’s expectations, while commuter use of the shuffle buses is below projected volume. this argument is unconvincing because the author oversimplifies the problem and its solutions in a number of ways.

to begin with, by concluding that the transit company must either reduce shuttle fares or increase parking fees, the author assumes that these are the only available solutions to the problem of limited shuttle use. however, it is possible that other factors—such as inconvenient shuttle routing and/or scheduling, safety concerns, or an increase in carpools—contribute to the problem. if so, adjusting fares or parking fees would might not solve the problem.

in addition, the author assumes that reducing shuttle fees and increasing parking fees are mutually exclusive alternatives. however, the author provides no reason for imposing an either/or choice. adjusting both shuttle fares and parking fees might produce better results. moreover, if the author is wrong in the assumption that parking fees and shuttle fees are the only possible causes of the problem, then the most effective solution might include a complex of policy changes—for example, in shuttle fares, parking fees, rerouting, and rescheduling.

in conclusion, this argument is weak because the author oversimplifies both the problem and its possible solutions. to strengthen the argument the author must examine all factors that might account for the shuttle’s unpopularity. additionally, the author should consider all possible solutions to determine which combination would bring about the greatest increase in shuttle use.

29. the following was excerpted from the speech of a spokesperson for synthetic farm products, inc.“many farmers who invested in the equipment needed to make the switch from synthetic to organic fertilizers and pesticides feel that it would be too expensive to resume synthetic farming at this point. but studies of farmers who switched to organic farming last year indicate that their current crop yields are lower. hence their purchase of organic farming equipment, a relatively minor investment compared to the losses that would result from continued lower crop yields, cannot justify persisting on an unwise course. and the choice to farm organically is financially unwise, given that it was motivated by environmental rather than economic concerns.”

discuss how well reasoned... etc.

this speaker argues that farmers who invested in organic farming equipment should resume synthetic farming because it is financially unwise to continue organic farming. the speaker cites studies showing that farmers who switched to organic farming last year had tower crop yields. based on these studies, the speaker concludes that the relatively inexpensive investment in organic farming equipment cannot justify continuing to farm organically. the speaker also claims that continuing to farm organically is financially unwise because it is motivated by environmental, not economic, concerns. the argument suffers from three problems.

one problem with this reasoning involves the vague comparative claim that farmers who switched to organic farming last year had lower crop yields. we are not informed whether the survey compared last year’s organic crop yields with yields from previous years or with those from synthetic farms. moreover, the author provides no evidence about how the survey was conducted. lacking more information about the survey, we cannot accept the speaker’s conclusion.

secondly, the speaker assumes that the low crop yields for first-time organic farmers last year are representative of crop yields for organic farmers overall. however, more experienced organic farmers might have had much better crop yields last year. also, the first-time organic farmers might improve their own crop yields in future years. moreover, last year’s yield may have been unusually low due to poor weather or other factors, and thus not indicative of future yields.

finally, in asserting that organic farming is financially unwise because it is motivated by environmental instead of economic concerns, the speaker unfairly assumes that a practice cannot be both environmentally and economically beneficial. it is possible that, in the long run, practices that help protect the environment will also result in greater economic benefits. for instance, organic farming methods may better protect soil from depletion of the elements that contribute to healthy crops, providing an economic benefit in the long run.

in conclusion, the speaker’s argument is poorly supported and is short-sighted. to better evaluate the argument, we would need more information about the how the survey was conducted, especially about the comparison the survey makes. to strengthen the argument, the speaker must present evidence that last years’ crop yields from first-time organic farmers are representative of yields among organic farms in general. the author must also provide evidence that environmentally sound practices cannot be economically beneficial as well.

30. the following appeared in a newspaper story giving advice about investments.“as overall life expectancy continues to rise, the population of our country is growing increasingly older. for example, over twenty percent of the residents of one of our more populated regions are now at least 65 years old, and occupancy rates at resort hotels in that region declined significantly during the past six months. because of these two related trends, a prudent investor would be well advised to sell interest in hotels and invest in hospitals and nursing homes instead.”

discuss how well reasoned... etc.

in this argument prudent investors are advised to stop investing in hotels and invest instead in hospitals and nursing homes. the author cites two related trends—an aging population and a decline in hotel occupancy—as grounds for this advice. to illustrate these trends, the author refers to another region of the country, where 20 percent of the population is over 65 years old and where occupancy rates in resort hotels have declined significantly during the past six months. this argument is unconvincing in a couple of important respects.

in the first place, the author provides no evidence to support the claim that the population as a whole is aging and that the hotel occupancy rate in general is declining. the example cited, while suggestive of these trends, is insufficient to warrant their truth because there is no reason to believe that data drawn from this unnamed region is representative of the entire country. for example, if the region from which the data was gathered was florida, it would clearly be unrepresentative. the reason for this is obvious. florida is populated by a disproportionate number of retired people over 65 years old and is a very popular vacation destination during the winter months. moreover, resort hotel occupancy in florida typically declines significantly during the summer months.

in the second place, the author has provided no evidence to support the claim that the decline in hotel occupancy is related to the aging of the population. the author appears to believe that the decrease in occupancy rates at resort hotels is somehow caused by the increase in the number of people over age 65. however, the example cited by the author establishes only that these two trends are correlated; it does not establish that the decline in hotel occupancy is due to an increase in the number of people over the age of 65.

in conclusion, the author’s investment advice is not based on sound reasoning. to strengthen the conclusion, the author must show that the trends were not restricted to a particular region of the country. the author must also show that the cause of the decline in hotel occupancy is the increase in the number of people over 65.

31. the following appeared as part of the business plan of an investment and financial consulting firm.“studies suggest that an average coffee drinker’s consumption of coffee increases with age, from age 10 through age 60. even after age 60, coffee consumption remains high. the average cola drinker’s consumption of cola, however, declines with increasing age. both of these trends have remained stable for the past 40 years. given that the number of older adults will significantly increase as the population ages over the next 20 years, it follows that the demand for coffee will increase and the demand for cola will decrease during this period. we should, therefore, consider transferring our investments from cola loca to early bird coffee.”

discuss how well reasoned... etc.

in this argument a consulting firm recommends the transfer of investments from cola loca to early bird coffee because, during the next 20 years, coffee demand will increase while cola demand will decrease. this prediction is based on the expectation that the number of older adults will significantly increase over the next 20 years, together with statistics, reportedly stable for the past 40 years, indicating that coffee consumption increases with age while cola consumption declines with increasing age. for three reasons, this financial advice may not be sound.

first, the argument assumes that relative supply conditions will remain unchanged over the next twenty years. however, the supply and cost of cola and coffee beans, as well as other costs of doing business as a producer of coffee or cola, may fluctuate greatly over a long time period. these factors may affect comparative prices of coffee and cola, which in turn may affect comparative demand and the value of investments in coffee and cola companies. without considering other factors that contribute to the value of a coffee or cola company, the firm cannot justify its recommendation.

secondly, the argument fails to account for the timing of the increase in coffee consumption. perhaps the population will age dramatically during the next five years, then remain relatively flat over the following 15 years. or perhaps most of the increase in average age will occur toward the end of the 20-year period. an investor has more opportunity to profit over the short and long term in the first scenario than in the second, assuming the investor can switch investments along the way. if the second scenario reflects the facts, the firm’s recommendation would be ill-founded.

finally, the firm unjustifiably relies on the studies that correlate coffee and cola consumption with age. the firm does not provide evidence to confirm the reliability of the studies. moreover, while the phrase “studies suggest” may appear to lend credibility to these claims, the phrase is vague enough to actually render the claims worthless, in the absence of any information about them.

in conclusion, the firm should not transfer investments from cola loca to early bird coffee on the basis of this argument. to better evaluate the recommendation, we would need more information about the study upon which it relies. we would also need more detailed projections of population trends during the next 20 years.

32. the following appeared in the editorial section of a west cambria newspaper.“a recent review of the west cambria volunteer ambulance service revealed a longer average response time to accidents than was reported by a commercial ambulance squad located in east cambria. in order to provide better patient care for accident victims and to raise revenue for our town by collecting service fees for ambulance use, we should disband our volunteer service and hire a commercial ambulance service.”

discuss how well reasoned... etc.

in this argument the author concludes that west cambria can increase revenues and provide better care to accident victims by disbanding the volunteer ambulance service and hiring a commercial one. the author reasons that this change would yield additional revenues because service fees could be imposed for ambulance use. the author also reasons that the city would provide better service to accident victims because a commercial service would respond more quickly to accidents than a volunteer service would. the author’s argument is flawed in two respects.

to begin with, the author’s plan for raising revenue for west cambria is questionable. unless the service fees are considerable or the accident rate is extremely high, it is unlikely that significant revenues will be raised by charging a fee for ambulance use. consequently, revenue generation is not a good reason to disband the volunteer service and hire a commercial service.

next, the author’s belief that better patient care would be provided by a commercial ambulance service than by a volunteer service is based on insufficient evidence. the fact that the commercial service in east cambria has a lower average response time than the volunteer service in west cambria is insufficient evidence for the claim that this will be the case for all commercial services. moreover, the author’s recommendation depends upon the assumption that response time to an accident is the only factor that influences patient care. other pertinent factors—such as ambulance-crew proficiency and training, and emergency equipment—are not considered.

in conclusion, this argument is unconvincing. to strengthen the argument the author would have to show that substantial revenue for the town could be raised by charging service fees for ambulance use. additionally, the author would have to provide more evidence to support the claim that commercial ambulance services provide better patient care than volunteer services.

33. the following is part of a business plan being discussed at a board meeting of the perks company.“it is no longer cost-effective for the perks company to continue offering its employees a generous package of benefits and incentives year after year. in periods when national unemployment rates are low, perks may need to offer such a package in order to attract and keep good employees, but since national unemployment rates are now high, perks does not need to offer the same benefits and incentives. the money thus saved could be better used to replace the existing plant machinery with more technologically sophisticated equipment, or even to build an additional plant.”

discuss how well reasoned... etc.

the author of perks company’s business plan recommends that funds currently spent on the employee benefits package be redirected to either upgrade plant machinery or build an additional plant. the author reasons that offering employees a generous package of benefits and incentives year after year is no longer cost-effective given current high unemployment rates, and that perks can attract and keep good employees without such benefits and incentives. while this argument has some merit, its line of reasoning requires close examination.

to begin with, the author relies on the reasoning that it is unnecessary to pay relatively high wages during periods of high unemployment because the market will supply many good employees at lower rates of pay. while this reasoning may be sound in a general sense, the particular industry that perks is involved in may not be representative of unemployment levels generally. it is possible that relatively few unemployed people have the type of qualifications that match job openings at perks, if this is the case, the claim that it is easier now to attract good employees at lower wages is ill-founded.

secondly, the argument relies on the assumption that the cost-effectiveness of a wage policy is determined solely by whatever wages a market can currently bear. this assumption overlooks the peripheral costs of reducing or eliminating benefits. for example, employee morale is likely to decline if perks eliminates benefits; as a result, some employees could become less productive, and others might quit. even if perks can readily replace those employees, training costs and lower productivity associated with high turnover may outweigh any advantages of redirecting funds to plant construction. moreover, because the recommended reduction in benefits is intended to fund the retrofitting of an entire plant or the building of a new one, the reduction would presumably be a sizable one; consequently, the turnover costs associated with the reduction might be very high indeed.

in conclusion, this argument is not convincing, since it unfairly assumes that a broad employment statistic applies to one specific industry, and since it ignores the disadvantages of implementing the plan. accordingly, i would suspend judgment about the recommendation until the author shows that unemployment in parks’ industry is high and until the author produces a thorough cost-benefit analysis of the proposed plan.

34. the following appeared as part of a plan proposed by an executive of the easy credit company to the president.“the easy credit company would gain an advantage over competing credit card services if we were to donate a portion of the proceeds from the use of our cards to a well-known environmental organization in exchange for the use of its symbol or logo on our card. since a recent poll shows that a large percentage of the public is concerned about environmental issues, this policy would attract new customers, increase use among existing customers, and enable us to charge interest rates that are higher than the lowest ones available.”

discuss how well reasoned... etc.

in this argument the author concludes that the easy credit company would gain several advantages over its competitors by donating a portion of its profits to a well-known environmental organization in exchange for the use of the organization’s logo on their credit card. the author reaches this conclusion on the basis of a recent poll that shows widespread public concern about environmental issues. among the advantages of this policy, the author foresees an increase in credit card use by existing customers, the ability to charge higher interest rates, and the ability to attract new customers. while the author’s argument has some merit, it suffers from two critical problems.

to begin with, the author assumes that the environmental organization whose logo is sought is concerned with the same environmental issues about which the poll shows widespread concern. however, the author provides no evidence that this is the case. it is possible that very few credit-card users are concerned about the issues that are the organization’s areas of concern; if so, then it is unlikely that the organization’s logo would attract much business for the easy credit company.

next, the author assumes that the public’s concern about environmental issues will result in its taking steps to do something about the problem—in this case, to use the easy credit company credit card. this assumption is unsupported and runs contrary to experience. also, it is more reasonable to assume that people who are concerned about a particular cause will choose a more direct means of expressing their concern.

in conclusion, the author’s argument is unconvincing as it stands. to strengthen the argument, the author must show a positive link between the environmental issues about which the public has expressed concern and the issues with which this particular environmental organization is concerned. in addition, the author must provide evidence to support the assumption that concern about a problem will cause people to do something about the problem.

35. the following appeared as part of a recommendation from the financial planning office to the administration of fern valley university.“in the past few years, fern valley university has suffered from a decline in both enrollments and admissions applications. the reason can be discovered from our students, who most often cite poor teaching and inadequate library resources as their chief sources of dissatisfaction with fern valley. therefore, in order to increase the number of students attending our university, and hence to regain our position as the most prestigious university in the greater fern valley metropolitan area, it is necessary to initiate a fund-raising campaign among the alumni that will enable us to expand the range of subjects we teach and to increase the size of our library facilities.”

discuss how well reasoned... etc.

the financial-planning office at fern valley university concludes that it is necessary to initiate a fund-raising campaign among alumni that will enable the university to expand the range of subjects it offers and increase the size of its library facilities. its argument is based on a five-year decline in enrollments and admission applications together with the claim that students cite poor teaching and inadequate library resources as their chief sources of dissatisfaction with fern valley. the conclusion of the financial-planning office is not strongly supported by the reasons given.

to begin with, this argument depends on the assumption that providing a greater range of subjects and a larger library will alleviate the students’ chief sources of dissatisfaction. however, the students have not complained about inadequate course offerings or about the size of the library; their complaint is that the existing courses are poorly taught and that library resources are inadequate. offering more kinds of classes does not improve teaching quality, and increasing a library’s size does nothing to enhance its holdings, or resources. accordingly, the recommendation does not bear directly on the problem as stated.

secondly, the proposal unfairly assumes that the recent enrollment and application decline was caused by poor teaching and inadequate library resources. it is equally possible that all colleges, regardless of teaching quality and library resources, have experienced similar declines. these declines may have been due to unrelated factors, such as unfavorable economic conditions, or an increase in high-paying computer jobs not requiring a college education.

thirdly, the author provides no support for the claim that students are dissatisfied with the teaching and library resources at fern valley. it is possible that the claim is based on hearsay or on scant anecdotal evidence. without more information about the basis of the claim, we cannot be sure that the financial-planning office is addressing the real problems.

in conclusion, the advice of the financial planning office is not well supported. to strengthen the argument, the planning office must provide evidence that students are dissatisfied with the range of subjects and with the library’s size, and that this dissatisfaction is the cause of the recent decline in enrollment and the number of admission applications. to better assess the argument as it stands, we would need to know whether the students’ attitudes were measured in a reliable, scientific manner.

36. the following appeared in an article in a college departmental newsletter“professor taylor of jones university is promoting a model of foreign language instruction in which students receive ten weeks of intensive training, then go abroad to live with families for ten weeks. the superiority of the model, professor taylor contends, is proved by the results of a study in which foreign language tests given to students at 25 other colleges show that first-year foreign language students at jones speak more fluently after only ten to twenty weeks in the program than do nine out of ten foreign language majors elsewhere at the time of their graduation.”

discuss how well reasoned... etc.

this newsletter article claims that professor taylor’s foreign-language program at jones university is a model of foreign language instruction. this conclusion is based on a study in which foreign language tests were given to students at 25 other universities. the study shows that first-year language students at jones speak more fluently after just 10 to 20 weeks in the program than do 90 percent of foreign-language majors at other colleges at graduation. despite these impressive statistics, i am unconvinced by this argument for two reasons.

to begin with, the assumption here is that students from professor taylor’s program have learned more than foreign language students at other universities. however, we are not given enough information about the study to be sure that this comparison is reliable. for example, the article does not tell us whether the foreign language students at jones were given the tests; it only reports that the tests in question were “given to students at 25 other colleges.” if jones students were not tested, then no basis exists for comparing them to students at the other universities. in addition, the article does not indicate whether students at all the universities, including jones, were given the same tests. if not, then again no basis exists for the comparison.

furthermore, we cannot tell from this article whether the universities in the study, or their students, are comparable in other ways. for instance, jones might be a prestigious university that draws its students from the top echelon of high school graduates, while the other universities are lower-ranked schools with more lenient admission requirements. in this event, the study wouldn’t tell us much about professor taylor’s program, for the proficiency of his students might be a function of their superior talent and intelligence.

in conclusion, the statistics cited in the article offer little support for the claim about taylor’s program. to strengthen the argument, the author must show that the universities in the study, including jones, were comparable in other ways, that their foreign language students were tested identically, and that taylor’s program was the only important difference between students tested at jones and those tested at the other universities.

37. the following appeared as part of an article in the business section of a local newspaper.“motorcycle x has been manufactured in the united states for over 70 years. although one foreign company has copied the motorcycle and is selling it for less, the company has failed to attract motorcycle x customers—some say because its product lacks the exceptionally loud noise made by motorcycle x. but there must be some other explanation. after all, foreign cars tend to be quieter than similar american-made cars, but they sell at least as well. also, television advertisements for motorcycle x highlight its durability and sleek lines, not its noisiness, and the ads typically have voice-overs or rock music rather than engine-roar on the sound track.”

discuss how well reasoned... etc.

the author rejects the claim that the loud engine noise of american-made motorcycle x appeals to the manufacturer’s customers and explains why they are not attracted to quieter, foreign-made imitations. the author’s rejection is based on two reasons. first, the author points out that foreign cars tend to be quieter than similar american-made cars, yet they sell just as well. secondly, the author claims that ads for motorcycle x do not emphasize its engine noise; instead, the ads highlight its durability and sleek lines, and employ voice-overs of rock music rather than engine roar. in my view, these reasons do not establish that the quieter engines of the foreign imitations fail to account for their lack of appeal.

to begin with, the first reason rests on the assumption that what automobile customers find appealing is analogous to what motorcycle customers find appealing. this assumption is weak, since although there are points of comparison between automobiles and motorcycles, there are many dissimilarities as well. for example, headroom, smooth ride, and quiet engines are usually desirable qualities in a car. however, headroom is not a consideration for motorcycle customers; and many motorcycle riders specifically want an exciting, challenging ride, not a smooth one. the same may be true of engine noise; it is possible that motorcyclists like what loud engine noise adds to the experience of motorcycle riding.

the author’s second reason is also problematic. although the engine noise of motorcycle x is not explicitly touted in advertisements, it does not necessarily follow that engine noise is not an important selling feature. because motorcycle x has been manufactured in the u.s. for over 70 years, its reputation for engine noise is probably already well known and need not be advertised. moreover, the advertisers might use rock music on motorcycle x ad soundtracks for the specific purpose of suggesting, or even simulating, its loud engine noise.

in conclusion, this author has not provided convincing reasons for rejecting the claim that quieter engines make foreign-made motorcycles less popular. the author’s analogy involving foreign car sales is weak, and the claim about motorcycle x advertisements misses the purpose of including rock music in the ads.

38. the following appeared in the editorial section of a campus newspaper.“because occupancy rates for campus housing fell during the last academic year, so did housing revenues. to solve the problem, campus housing officials should reduce the number of available housing units, thereby increasing the occupancy rates. also, to keep students from choosing to live off-campus, housing officials should lower the rents, thereby increasing demand.”

discuss how well reasoned... etc.

the author of this article argues that, to reverse declining revenues from campus housing rentals, campus housing officials should decrease the number of available housing units and reduce rent prices on the units. the author’s line of reasoning is that fewer available units will limit supply while lower rents will increase demand, thereby improving overall occupancy rates, and that the resulting increase in occupancy rates will, in turn, boost revenues for the campus. this reasoning is unconvincing for several reasons.

to begin with, the author assumes that boosting occupancy rates will improve revenues. all other factors remaining unchanged, this would be the case. however, the author proposes reducing both the supply of units and their rental prices. both of these actions would tend to reduce revenues. the author provides no evidence that the revenue-enhancing effect of a higher occupancy rate will exceed the revenue-decreasing effect of reduced supply and price. without such evidence, the argument is unconvincing.

secondly, the author assumes that lowering rents will lead to higher revenues by increasing demand. however, it is possible that demand would decrease, depending on the extent of the rent reduction as well as other factors—such as overall enrollment and the supply and relative cost of off-campus housing. moreover, even if demand increases by lowering rents, revenues will not necessarily increase as a result. other factors, such as maintenance and other costs of providing campus housing units and the reduced supply of rental units might contribute to a net decrease in revenue.

thirdly, in asserting that lowering rental rates will increase demand, the author assumes that current rental rates are causing low demand. however, low demand for student housing could be a function of other factors. for instance, the student housing units may be old and poorly maintained. perhaps students find the campus housing rules oppressive, and therefore prefer to live off-campus; or perhaps enrollments are down generally, affecting campus housing occupancy.

in conclusion, the author of this editorial has not argued effectively for a decrease in the number of available campus housing units and a reduction in rental rates for those units. to strengthen the argument, the author must show that a rent reduction will actually increase demand, and that the revenue-enhancing effect of greater demand will outweigh the revenue-reducing effect of a smaller supply and of lower rental rates.

39. the following appeared in an avia airlines departmental memorandum.“on average, 9 out of every 1,000 passengers who traveled on avia airlines last year filed a complaint about our baggage-handling procedures. this means that although some 1 percent of our passengers were unhappy with those procedures, the overwhelming majority were quite satisfied with them; thus it would appear that a review of the procedures is not important to our goal of maintaining or increasing the number of avia’s passengers.”

discuss how well reasoned... etc.

the conclusion in this avia airlines memorandum is that a review of the airline’s baggage-handling procedures will not further its goal of maintaining or increasing the number of avia passengers. the author’s line of reasoning is that the great majority of avia passengers are happy with baggage handling at the airline because only one percent of passengers who traveled on avia last year filed a complaint about avia’s procedures. this argument is problematic in two important respects.

first, the argument turns on the assumption that the 99 percent of avia passengers who did not complain were happy with the airline’s baggage-handling procedures. however, the author provides no evidence to support this assumption. the fact that, on the average, 9 out of 1000 passengers took the time and effort to formally complain indicates nothing about the experiences or attitudes of the remaining 991. it is possible that many passengers were displeased but too busy to formally complain, while others had no opinion at all. lacking more complete information about passengers’ attitudes, we cannot assume that the great majority of passengers who did not complain were happy.

secondly, in the absence of information about the number of passengers per flight and about the complaint records of competing airlines, the statistics presented in the memorandum might distort the seriousness of the problem. given that most modern aircraft carry as many as 300 to 500 passengers, it is possible that avia received as many as 4 or 5 complaints per flight. the author unfairly trivializes this record. moreover, the author fails to compare avia’s record with those of its competitors. it is possible that a particular competitor received virtually no baggage-handling complaints last year. if so, avia’s one percent complaint rate might be significant enough to motivate customers to switch to another airline.

in conclusion, the author has failed to demonstrate that a review of the baggage-handling procedures at avia airlines is not needed to maintain or increase the number of avia’s passengers. to strengthen the argument, the author must at the very least provide affirmative evidence that most avia passengers last year were indeed happy with baggage-handling procedures. to better evaluate the argument, we would need more information about the numbers of avia passengers per flight last year and about the baggage-handling records of avia’s competitors.

40. the following appeared as part of an article in a weekly newsmagazine.“the country of sacchar can best solve its current trade deficit problem by lowering the price of sugar, its primary export. such an action would make sacchar better able to compete for markets with other sugar-exporting countries. the sale of sacchar’s sugar abroad would increase, and this increase would substantially reduce sacchar’s trade deficit.”

discuss how well reasoned... etc.

the author of this article argues that the country of sacchar can best solve its current trade deficit problem by lowering the price of its main export, sugar. the line of reasoning is that this action would make sacchar more competitive with other sugar-exporting countries, thereby increasing sales of sacchar’s sugar abroad and, in turn, substantially reducing the trade-deficit. this line of reasoning is unconvincing for a couple of reasons.

in the first place, this argument is based on an oversimplified analysis of the trade deficit problem sacchar currently faces. a trade-deficit occurs when a country spends more on imports than it earns from exports. the author’s argument relies on the assumption that earnings from imports will remain constant. however, the author provides no evidence that substantiates this assumption. it is possible that revenues from imports will increase dramatically in the near future; if so, the course of action proposed by the author might be unnecessary to solve sacchar’s trade deficit problem. conversely, it is possible that revenues from imports are likely to decrease dramatically in the near future. to the extent that this is the case, lowering sugar prices may have a negligible countervailing effect, depending on the demand for sacchar’s sugar.

in the second place, increasing sales by lowering the price of sugar will not yield an increase in income unless the increase in sales is sufficient to overcome the loss in income due to the lower price. this raises three questions the author fails to address. first, will a price decrease in fact stimulate demand? second, is demand sufficient to meet the increase in supply? third, can sacchar increase the sugar production sufficiently to overcome the deficit? in the absence of answers to these questions, we cannot assess the author’s proposal.

in conclusion, the author provides an incomplete analysis of the problem and, as a result, provides a questionable solution. to better evaluate the proposal, we would need to know how revenues from imports are likely to change in the future. to strengthen the argument, the author must provide evidence that demand is sufficient to meet the proposed increase in supply, and that sacchar has sufficient resources to accommodate the increase.

41. the following appeared as part of an article in a trade publication.“stronger laws are needed to protect new kinds of home-security systems from being copied and sold by imitators. with such protection, manufacturers will naturally invest in the development of new home-security products and production technologies. without stronger laws, therefore, manufacturers will cut back on investment. from this will follow a corresponding decline not only in product quality and marketability, but also in production efficiency, and thus ultimately a loss of manufacturing jobs in the industry.”

discuss how well reasoned... etc.

the author of this article warns that stronger laws are needed to protect new kinds of home security systems from being copied and sold by imitators in order to prevent an eventual loss of manufacturing jobs within the industry. this conclusion is based on the following chain of reasoning: with the protection of stronger laws, manufacturers will naturally invest in the development of new home security products and production technologies, whereas without such protection, manufacturers will cut back on investment. if manufacturers cut back on investment, then a decline in product quality and marketability, as well as in production efficiency, will result. this, in turn, will cause the predicted loss of industry jobs. this line of reasoning is unconvincing for several reasons.

to begin with, the author assumes that existing copyright, patent and trade secret laws are inadequate to protect home security system design. but the author never explains why these laws don’t offer sufficient protection, nor does he offer any evidence to show that this is the case.

secondly, the argument depends on the twin assumptions that stronger legal protection will encourage manufacturers to invest in home security-system production, while the absence of strong legal protection will have the opposite effect. the author fails to provide any evidence or reasons for accepting these assumptions about cause-and-effect connections between the law and what happens in the marketplace.

moreover, both of these assumptions can be challenged. it is possible that stronger protections would not greatly affect industry investment or jobs overall, but would instead help to determine which companies invested heavily and, therefore, provided the jobs. for instance, a less-restricted market might foster investment and competition among smaller companies, whereas stronger legal protections might encourage market domination by fewer, larger companies.

in conclusion, i do not find this argument compelling. the author must provide evidence that home security system designs are not being adequately protected by current patent, copyright or trade secret laws. the author must also provide an argument for the assumptions that stronger laws will create more industry jobs overall, while the absence of stronger laws will result in fewer industry jobs.

42. the following appeared in the opinion section of a national newsmagazine.“to reverse the deterioration of the postal service, the government should raise the price of postage stamps. this solution will no doubt prove effective, since the price increase will generate larger revenues and will also reduce the volume of mail, thereby eliminating the strain on the existing system and contributing to improved morale.”

discuss how well reasoned... etc.

the author concludes that a postage-stamp price increase is needed to reduce the deterioration of the postal service. the author reasons that raising the price of stamps will accomplish this goal because it will generate more revenue, thereby eliminating the strain on the system. the author further reasons that a price increase will also reduce the volume of mail, thereby improving the morale of postal workers. the reasoning in this argument is problematic in three respects.

the main problem with the argument is the author’s mistaken assumption that eliminating strain on the system and improving employee morale are mutually achievable by way of an increase in stamp prices. a price increase will generate more revenue only if the volume of mail remains constant or increases. but, if the volume of mail increases or remains constant, worker morale will not be improved. on the other hand, if the price increase reduces the volume of mail, revenues may decrease, and the strain on the system will not be eliminated. consequently, eliminating the strain on the system and improving the morale of the workers cannot both be achieved by simply raising the price of postage stamps.

secondly, the author’s conclusion that the proposed price increase is necessary to reduce deterioration of the postal service relies on the assumption that no other action would achieve the same result. however, the author provides no evidence to substantiate this assumption. it is possible, for example, that careful cost-cutting measures that do not decrease worker morale might achieve the same goal. it is also possible that other revenue-enhancing measures that do not undermine employee morale are available.

thirdly, the author unfairly assumes that reducing mail volume and increasing revenues will improve employee morale. this is not necessarily the case. it is possible that employee morale is materially improved only by other means, and that additional revenues will not be used in ways that improve morale. it is also possible that a decrease in mail volume will result in a reduction of the size of the labor force, regardless of revenues, which in turn might undermine morale.

in conclusion, the author’s proposed solution to the problem of the deterioration of the postal service will not work. raising postage-stamp prices cannot bring about both of the outcomes the author identifies as being necessary to solve the problem. before we can accept the argument, the author must modify the proposal accordingly and must provide more information about the relationship between employee morale and mail volume.

43. the following appeared in an article in the health section of a newspaper.“there is a common misconception that university hospitals are better than community or private hospitals. this notion is unfounded, however: the university hospitals in our region employ 15 percent fewer doctors, have a 20 percent lower success rate in treating patients, make far less overall profit, and pay their medical staff considerably less than do private hospitals. furthermore, many doctors at university hospitals typically divide their time among teaching, conducting research, and treating patients. from this it seems clear that the quality of care at university hospitals is lower than that at other kinds of hospitals.”

discuss how well reasoned... etc.

in this argument the author concludes that university hospitals provide no better care than private or community hospitals. the author bases this conclusion on the following claims about university hospitals: the ones in this region employ 15 percent fewer doctors; they have a 20 percent lower success rate in treating patients; they pay their staffs less money; they make less profit than community hospitals; and they utilize doctors who divide their time between teaching, research and treating patients. this argument is unconvincing for several reasons.

the most egregious reasoning error in the argument is the author’s use of evidence pertaining to university hospitals in this region as the basis for a generalization about all university hospitals. the underlying assumption operative in this inference is that university hospitals in this region are representative of all university hospitals. no evidence is offered to support this gratuitous assumption.

secondly, the only relevant reason offered in support of the claim that the quality of care is lower in university hospitals than it is at other hospitals is the fact that university hospitals have a lower success rate in treating patients. but this reason is not sufficient to reach the conclusion in question unless it can be shown that the patients treated in both types of hospitals suffered from similar types of maladies. for example, if university hospitals routinely treat patients suffering from rare diseases whereas other hospitals treat only those who suffer from known diseases and illnesses, the difference in success rates would not be indicative of the quality of care received.

finally, the author assumes that the number of doctors a hospital employs, its success rate in treating patients, the amount it pays its staff, and the profits it earns are all reliable indicators of the quality of care it delivers. no evidence is offered to support this assumption nor is it obvious that any of these factors is linked to the quality of care delivered to patients. moreover, the fact that doctors in university hospitals divide their time among many tasks fails to demonstrate that they do a poorer job of treating patients than doctors at other kinds of hospitals. in fact, it is highly likely that they do a better job because they are more knowledgeable than other doctors due to their teaching and research.

in conclusion, the author’s argument is unconvincing. to strengthen the argument the author would have to demonstrate that university hospitals in this region are representative of all university hospitals, as well as establishing a causal link between the various factors cited and the quality of care delivered to patients.

44. the following is part of a business plan created by the management of the megamart grocery store.“our total sales have increased this year by 20 percent since we added a pharmacy section to our grocery store. clearly, the customer’s main concern is the convenience afforded by one-stop shopping. the surest way to increase our profits over the next couple of years, therefore, is to add a clothing department along with an automotive supplies and repair shop. we should also plan to continue adding new departments and services, such as a restaurant and a garden shop, in subsequent years. being the only store in the area that offers such a range of services will give us a competitive advantage over other local stores.”

discuss how well reasoned... etc.

the management of the megamart grocery store concludes that adding new departments and services is the surest way to increase profits over the next couple of years. they are led to this conclusion because of a 20 percent increase in total sates, realized after the addition of a pharmacy section to the grocery store. on the basis of this experience, they concluded that the convenience of one-stop shopping was the main concern of their customers. the management’s argument is faulty in several respects.

in the first place, the management assumes that the increase in total sales was due to the addition of the pharmacy section. however, the only evidence offered to support this conclusion is the fact that the addition of the pharmacy preceded the increase in sales. but the mere fact that the pharmacy section was added before the increase occurred is insufficient grounds to conclude that it was responsible for the increase. many other factors could bring about this same result. lacking a detailed analysis of the source of the sales increase, it would be sheer folly to attribute the increase to the addition of the pharmacy section.

in the second place, even if it were the case that the increase in total sales was due to the addition of the pharmacy section, this fact alone is insufficient to support the claim that adding additional departments will increase sales even further. it is quite possible that the addition of the pharmacy section increased sales simply because there was no other pharmacy in the vicinity. the additional proposed departments and services, on the other hand, might be well represented in the area and their addition might have no impact whatsoever on the profits of the store. in other words, there may be relevant differences between the pharmacy section and the additional proposed sections that preclude them from having a similar effect on the sales of the store.

in conclusion, the management’s argument is not well-reasoned. to strengthen the conclusion, the management must provide additional evidence linking the addition of the pharmacy section to the increase in total sales. it must also show that there are no exceptional reasons for the sales increase due to the pharmacy section that would not apply to the other proposed additions.

45. the following appeared as part of a column in a popular entertainment magazine.“the producers of the forthcoming movie 3003 will be most likely to maximize their profits if they are willing to pay robin good several million dollars to star in it— even though that amount is far more than any other person involved with the movie will make. after all, robin has in the past been paid a similar amount to work in several films that were very financially successful.”

discuss how well reasoned... etc.

in this argument the author concludes that paying robin good several million dollars to star in the movie “3003” is the most likely way for the movie’s producers to maximize their profits. the author’s line of reasoning is that because robin has been paid similar amounts of money to work in other films that were financially successful, it is likely that “3003” will also be financially successful if robin stars in it. this argument is unconvincing in two important respects.

the main problem with this argument involves the author’s assumption that the financial success of the other films was due entirely to robin good’s participation. if this were the case, it would certainly make good sense to pay robin handsomely to star in “3003.” however, the author offers no evidence to support this contention.

moreover, there are many factors that could account for the financial success of the movies in which robin previously appeared, other than the mere fact that robin appeared in them. for example, their financial success might have been due to the photography, the plot of the story, the director, or any combination of these. lacking a more detailed analysis of the reasons for the success of these other movies, it is folly to presume that their financial success was entirely due to robin’s participation.

in conclusion, this is a weak argument. to strengthen the conclusion that hiring robin is the best way for the producers of “3003” to maximize their profits, the author would have to provide evidence that the financial success of the movies robin previously worked in resulted solely from the fact that robin starred in them.

46. the following appeared in a memorandum from the directors of a security and safety consulting service.“our research indicates that over the past six years no incidents of employee theft have been reported within ten of the companies that have been our clients. in analyzing the security practices of these ten companies, we have further learned that each of them requires its employees to wear photo identification badges while at work. in the future, therefore, we should recommend the use of such identification badges to all of our clients.”

discuss how well reasoned... etc.

in this argument the directors of a security-and safety-consulting service conclude that the use of photo identification badges should be recommended to all of their clients as a means to prevent employee theft. their conclusion is based on a study revealing that ten of their previous clients who use photo identification badges have had no incidents of employee theft over the past six-year period. the directors’ recommendation is problematic in several respects.

in the first place, the directors’ argument is based on the assumption that the reason for the lack of employee theft in the ten companies was the fact that their employees wear photo identification badges. however, the evidence revealed in their research establishes only a positive correlation between the lack of theft and the requirement to wear badges; it does not establish a causal connection between them. other factors, such as the use of surveillance cameras or spot checks of employees’ briefcases and purses could be responsible for lack of employee theft within the ten companies analyzed.

in the second place, the directors assume that employee theft is a problem that is common among their clients and about which their clients are equally concerned. however, for some of their clients this might not be a problem at all. for example, companies that sell services are much less likely to be concerned about employee theft than those who sell products. moreover, those that sell small products would be more concerned about theft than those that sell large products. consequently, even if wearing badges reduces employee theft, it might not be necessary for all of the firm’s clients to follow this practice.

in conclusion, the director’s recommendation is not well supported. to strengthen the conclusion they must establish a causal relation between the wearing of identification badges and the absence of employee theft. they also must establish that the firm’s clients are sufficiently similar to all profit from this practice.

47. the following appeared as part of an article in the business section of a local newspaper.“the owners of the cumquat cafe evidently made a good business decision in moving to a new location, as can be seen from the fact that the cafe will soon celebrate its second anniversary there. moreover, it appears that businesses are not likely to succeed at the old location: since the cafe’s move, three different businesses—a tanning salon, an antique emporium, and a pet–grooming shop—have occupied its former spot.”

discuss how well reasoned... etc.

in this argument the author concludes that cumquat cafe was correct in its decision to move to a new location. in support of this assessment the author points out that while the cafe has been in business for two years at its new location, three businesses have failed at its previous location. the author’s line of reasoning is that the cause of the failure of the three businesses is the fact that they all occupied the same location. this argument is problematic in two important respects.

in the first place, no evidence has been offered to support the assumption that the reason the three businesses failed was their location. while location is an important contributing factor to a business’ success or failure, it is not the only such factor. many other reasons—poor business practices, lack of advertising, or poor customer service—could just as likely account for their lack of success. lacking a detailed analysis of the reasons these businesses failed, it would be foolish to attribute their failure to their location.

in the second place, while location may have been a factor which contributed to the failure of these businesses, the reason may not have been the location itself but rather the suitability of the business to the location. for example, a pet-grooming shop or a tanning salon located in a downtown metropolitan business district is unlikely to succeed simply because this type of business is obviously unsuitable to the location. on the other hand, a bank in the same location might be extremely successful simply because of its suitability to the location.

in conclusion, the author’s argument is unconvincing. to strengthen the conclusion, the author would have to evaluate other possible causes of the failure of the three businesses, then in each case eliminate all possible causes except location.

48. the following appeared in the editorial section of a local newspaper.“the profitability of croesus company, recently restored to private ownership, is a clear indication that businesses fare better under private ownership than under public ownership.”

discuss how well reasoned... etc.

based upon the profitability of the croesus company and the fact that it was recently converted from public to private ownership, the author concludes that private ownership is better for businesses than public ownership. i find this argument unconvincing in two respects.

in the first place, the evidence the author provides is insufficient to support the conclusion drawn from it. one example is rarely sufficient to establish a general conclusion. unless it can be shown that croesus company is representative of all companies that have converted from public to private ownership, the conclusion that all companies would be more profitable under private ownership is completely unwarranted. in fact, in the face of such limited evidence it is fallacious to draw any conclusion at all.

in the second place, the author assumes that the reason for croesus’ profitability was its conversion from public to private ownership. this assumption, however, is not supported in the argument. in the absence of evidence to support this assumption many other explanations for croesus company’s profitability are possible. for example, its success may be due to the fact that croesus has few competitors or because the product or service it provides is unique, or because it has an exceptionally skilled management team.

in conclusion, this argument is unconvincing. to strengthen the conclusion, additional examples of successful companies that converted from public to private ownership are required. additionally, the author would have to show that the reason for the success of these companies was the fact that they were privately owned.

49. the following appeared in the editorial section of a local newspaper.“if the paper from every morning edition of the nation’s largest newspaper were collected and rendered into paper pulp that the newspaper could reuse, about 5 million trees would be saved each year. this kind of recycling is unnecessary, however, since the newspaper maintains its own forests to ensure an uninterrupted supply of paper.”

discuss how well reasoned... etc.

this editorial begins with the impressive statistic that five-million trees could be saved every year if the morning edition of the nation’s largest newspaper were collected and rendered into pulp that the newspaper could reuse. but then the author goes on to conclude that this kind of recycling is unnecessary because the newspaper maintains its own forests to ensure an uninterrupted supply of paper. this argument is seriously flawed by two unwarranted assumptions.

the first assumption is that the only reason to recycle the newspaper is to ensure a continuous supply of paper. the author reasons that since this need is currently met by the forests that the newspaper maintains, recycling is unnecessary. this reasoning is extremely shortsighted. not only does the author fail to see the ecological advantages of preserving the trees, he also fails to see the obvious economic advantages of doing this. moreover, using recycled paper is the best way to ensure a continuous paper supply because, unlike the forest, paper is a reusable resource.

the second assumption is that only the newspaper would have an interest in the pulp processed from its recycled morning edition. this is probably not the case, however, given the enormous market for recycled paper—for books, packaging, other newspapers, and so on. moreover, there is no direct connection between the newspaper that is recycled and those companies that find uses for the products of recycling. accordingly, contrary to the author’s assumption, there may be a great interest, indeed a need, for pulp from recycling the newspaper in question.

in conclusion, the author’s claim that recycling the newspaper is unnecessary is ill-founded. to strengthen the argument the author would have to show that there are no other compelling reasons to recycle the newspaper besides the one cited in the editorial.

50. the following appeared as part of a business plan recommended by the new manager of a musical rock group called zapped.“to succeed financially, zapped needs greater name recognition. it should therefore diversify its commercial enterprises. the rock group zonked plays the same type of music that zapped plays, but it is much better known than zapped because in addition to its concert tours and four albums, zonked has a series of posters, a line of clothing and accessories, and a contract with a major advertising agency to endorse a number of different products.”

discuss how well reasoned... etc.

the new manager of the rock group zapped believes that name recognition is the key to attaining financial success for the group. to increase name recognition the manager recommends that zapped diversify its commercial enterprises. the grounds for this recommendation is an analogy with zonked, a much better-known rock group that plays the same kind of music as zapped. according to the manager, the main reason zonked is better known than zapped is that zonked participates in several promotional enterprises in addition to concerts and albums. the manager’s recommendation is questionable for two reasons.

in the first place, the author assumes that the only relevant difference between zapped and zonked is that zonked has greater name recognition than zapped. if this were the case, the manager’s recommendation would be apt. however, the fact that the two rock groups play the same kind of music leaves open the question of whether their performance of this music is comparable. if zonked’s performance is sufficiently better than zapped’s, this could go a long way toward explaining why zonked is much better known.

in the second place, the author assumes that name recognition is all that is required for financial success. while name recognition is an important element in determining the success or failure of any enterprise, it is hardly the only element required. other factors are equally important. in the case of rock bands, factors such as musical talent, showmanship, and repertoire play a significant role in determining the financial success of the group. if zonked is superior to zapped in these areas, this difference could account for zonked’s financial success.

in conclusion, the manager’s argument is unconvincing. to strengthen the argument the author would have to show that zapped and zonked are alike in all relevant ways except name recognition.

51. the following appeared in a magazine article on trends and lifestyles.“in general, people are not as concerned as they were a decade ago about regulating their intake of red meat and fatty cheeses. walk into the heart’s delight, a store that started selling organic fruits and vegetables and whole-grain flours in the 1960’s, and you will also find a wide selection of cheeses made with high butterfat content. next door, the owners of the good earth café, an old vegetarian restaurant, are still making a modest living, but the owners of the new house of beef across the street are millionaires.”

discuss how well reasoned... etc.

the author of an article about lifestyle trends concludes that, in general, people are not as concerned as they were a decade ago with regulating their intake of red meat and fatty cheeses. as evidence, the author cites the fact that a wide selection of high-fat cheeses is now available at a long-established grocery store, heart’s delight, which specializes in organic fruits and vegetables and whole grains. the author further points out that the owners of the vegetarian restaurant next door, good earth cafe, now make only a modest living while the owners of the new house of beef across the street are millionaires. this argument is unconvincing.

to begin with, the argument relies on the assumption that the dietary habits and attitudes of customers at these three businesses will reflect those of people generally. but the three businesses, all located in the same area of a single community, just might serve a clientele whose diets differ greatly from the diets of people in other areas of the community, or in other communities. the generalization that the author draws from this biased sample cannot be considered reliable.

in addition, trends at these three businesses do not necessarily reflect the dietary habits and attitudes of their customers in the way the author claims. for example, we are not informed about how well the high-fat cheeses at heart’s delight are selling relative to low-fat and nonfat alternatives. similarly, it is possible that at house of beef menu items other than red meat—such as chicken, fish, or salad bar—are just as popular as red meat among the restaurant’s patrons.

finally, the author assumes that the financial conditions of the owners of the two restaurants were caused by a general lack of concern with regulating red meat and fatty-cheese intake. however, it is equally possible that the lackluster financial success of good earth was caused by mismanagement or increasing overhead costs. furthermore, it is possible that house of beef is generating little business, but its owners were already millionaires before they opened this restaurant or are making their money in other concurrent business endeavors.

in conclusion, the author’s evidence is too weak to support any conclusion about general dietary trends. before we can accept the conclusion, the author must provide evidence from a representative sample of food-service businesses, and must clearly show that sates of red meat and fatty cheeses are increasing relative to sales of low-fat alternatives. the author must also provide evidence that the financial conditions of the owners of the two restaurants were actually caused by a general waning concern with regulating fat intake.

52. the following editorial appeared in the elm city paper.“the construction last year of a shopping mall in downtown oak city was a mistake. since the mall has opened, a number of local businesses have closed, and the downtown area suffers from an acute parking shortage, and arrests for crime and vagrancy have increased in the nearby oak city park. elm city should pay attention to the example of the oak city mall and deny the application to build a shopping mall in elm city.”(新題)

discuss how well reasoned... etc.

“the construction of last year of a shopping mall in downtown oak city has done little for the residents of the community. since the mall has opened, a number of local businesses have closed, and the downtown area suffers from an acute parking shortage. in addition, because the mall’s owner lives in nearby elm city, the profits derived from sales at the mall are not being returned to the community. these problems, coupled with the increase in trash and litter in nearby oak city park, demonstrate that oak city did not use good judgment in allowing the construction of the mall in the first place.”(老題)

discuss how well reasoned... etc.

(此文根據老題寫作)in this editorial the author rebukes oak city for allowing the construction of a new downtown shopping mall. citing a number of problems that have occurred since the building of the mail, the author concludes that the residents of oak city have not benefited from the mail and that oak city exercised poor judgment in allowing the mail to be built. among the problems cited by the author are the closure of local businesses, lack of parking in the downtown area, and increased trash and litter in a city park near the mall. moreover, the author argues that profits derived from sales are not benefiting oak city because the owner of the mall lives in another city. the author’s argument is problematic in several respects.

in the first place, the author assumes that addition of the new mall is the cause of the various problems cited. the only evidence offered to support this claim is that the construction of the mall occurred before these problems manifested themselves. however, this evidence is insufficient to establish the claim in question. a chronological relationship is only one of the indicators of a causal relationship between two events.

in the second place, the author has focused only on negative effects the mall has had on the city. a more detailed analysis of the situation might reveal that the positive benefits for the city far outweigh the problems on which the author focuses. for example, new jobs might have been created for the residents of oak city, and tax revenues might have been increased for the city. lacking a more comprehensive analysis of the impact of the mail on oak city, it is presumptuous on the part of the author to conclude that oak city’s decision to allow the mall to be built was incorrect.

in conclusion, the author’s argument is unconvincing. to strengthen the argument the author would have to demonstrate that the construction of the mall caused the various problems mentioned. the author would also have to show that the negative effects of the project outweighed the positive effects.

53. the following appeared as part of an editorial in a weekly newsmagazine.“historically, most of this country’s engineers have come from our universities; recently, however, our university-age population has begun to shrink, and decreasing enrollments in our high schools clearly show that this drop in numbers will continue throughout the remainder of the decade. consequently, our nation will soon be facing a shortage of trained engineers. if we are to remain economically competitive in the world marketplace, then, we must increase funding for education—and quickly.”

discuss how well reasoned... etc.

an editorial in a weekly news magazine warns that we must quickly increase funding for education in order to remain economically competitive in the world marketplace. the line of reasoning is that the nation will soon face a shortage of engineers because engineers have come from universities, and that our university-age population is shrinking. moreover, decreasing enrollments in high schools clearly show that this drop in university-age students will continue throughout the decade. the author’s argument is not convincing because it is based on several questionable assumptions.

first, the author assumes that because our university-age population is shrinking, university enrollments will likewise shrink. but even if the number of university-age students is dropping, it is possible that a greater proportion of those students will enter universities. if this percentage were sufficiently large, university enrollments could remain relatively stable. moreover, even if overall university enrollments did drop, we must further assume that the number of engineering students would likewise drop. however, decreases in overall enrollments do no necessarily result in proportional enrollment decreases in each field of study. if demand for engineers were high, then a larger percentage of university students might study to become engineers, in which case engineering enrollments could increase or remain constant, while those in other major fields of study would drop disproportionately.

an additional assumption is that economic success in the world marketplace depends on the number of engineers produced by our universities. this assumption is simplistic. professionals in other fields—such as agriculture, banking, and business—may contribute equally to our global success. the author does not explain why the predicted shortage of engineers is more critical than shortages in other fields that might result from shrinking university enrollments. nor does the author demonstrate that providing more funds for education will correct the predicted shortage of engineers. even if all of the previous assumptions are accepted, no connection between increased funding and the desired enrollment increase has been established.

in conclusion, the author has failed to make a convincing case for increased funding for education. before we accept the conclusion, the author must provide evidence that we face a critical shortage of engineers, and that increased funding will have direct bearing on correcting this shortage. as it stands, both these claims rest on unwarranted assumptions.

54. the following appeared in an excelsior company memorandum.“the excelsior company plans to introduce its own brand of coffee. since coffee is an expensive food item, and since there are already many established brands of coffee, the best way to gain customers for the excelsior brand is to do what superior, the leading coffee company, did when it introduced the newest brand in its line of coffees: conduct a temporary sales promotion that offers free samples, price reductions, and discount coupons for the new brand.”

discuss how well reasoned... etc.

this company memorandum recommends that excelsior conduct a temporary sales promotion for its new brand of coffee that includes offering free samples, price reductions, and discount coupons. this recommendation is based on the fact that superior, the leading coffee company, used just such a promotion to introduce the newest brand in its line of coffees. this argument is unconvincing because it relies on three questionable assumptions.

first of all, the argument rests on the assumption that a promotional strategy that works for one company will work for another. however, excelsior and superior may not be sufficiently similar to warrant this assumption. promotional techniques that work for a leader with established name recognition for its brand of coffees may be ineffective for a company with no similar name recognition new to the brand coffee market. accordingly, excelsior might be better advised to employ some other strategy, such as a media advertising plan, to first attain broad name recognition.

the argument also depends on the assumption that excelsior can afford a promotional plan similar to superior’s. however, free samples, price reductions, and discounts all reduce profits and may actually result in temporary losses. while a leading company with other profitable products in the same line can absorb a temporary loss, for a fledgling competitor this strategy might be very risky and may even result in business failure.

finally, the argument relies on the assumption that superior’s promotional campaign for its newest coffee was successful. however, the memo provides no evidence that this was the case. it is possible that the promotion was entirely ineffective, and that superior remains the leader in its field despite this small failure. if so, excelsior may be ill-advised to follow superior’s promotional strategy.

in conclusion, the two companies are too dissimilar to justify the recommendation that excelsior model its promotional strategy on superior’s. to strengthen the argument, the author of the memo must establish that excelsior has sufficient operating capital to launch the recommended sales campaign, and that this strategy would be more effective than another strategy, such as using extensive media advertising.

55. the following appeared as part of an article in a health club trade publication.“after experiencing a decline in usage by its members, healthy heart fitness center built an indoor pool. since usage did not increase significantly, it appears that health club managers should adopt another approach—lowering membership fees rather than installing expensive new features.”

discuss how well reasoned... etc.

because healthy heart fitness centers experienced no significant increase in member usage as a result of building a new indoor pool, the author cautions other health dub managers against installing new features as a means of increasing member usage, instead, they are advised to lower membership fees. this argument is flawed in two critical respects.

first, the conclusion that installing new features at fitness centers will not increase member usage is based on too small a sample to be reliable. the only evidence offered in support of this conclusion is the fact that healthy heart fitness center did not experience an increase. unless it can be shown that healthy heart is typical of all fitness centers, the fact that it experienced no increase in member usage is not grounds for concluding that all fitness centers will experience similar results.

second, the author fails to consider other possible reasons why building an indoor pool failed to increase healthy heart’s member usage. perhaps healthy heart’s members are primarily interested in body-building rather than cardiovascular exercise, or perhaps they prefer racquetball; or perhaps they just don’t like swimming. reasons such as these would help to explain why the addition of a new indoor pool failed to increase member usage. the author’s failure to investigate or even consider other possible explanations for healthy heart’s poor results renders the conclusion based upon them highly suspect.

in conclusion, the author’s argument is not convincing. to strengthen the argument it would be necessary to show that healthy heart fitness center is typical of all fitness centers. additionally, the author would have to show that other possible reasons for the lack of increase in member usage could be eliminated.

56. the following appeared as part of an article in a popular arts and leisure magazine.“the safety codes governing the construction of public buildings are becoming far too strict. the surest way for architects and builders to prove that they have met the minimum requirements established by these codes is to construct buildings by using the same materials and methods that are currently allowed. but doing so means that there will be very little significant technological innovation within the industry, and hence little evolution of architectural styles and design—merely because of the strictness of these safety codes.”

discuss how well reasoned... etc.

the conclusion of this argument is that technological innovation as well as the evolution of architectural styles and design will be minimized in the future. the author’s line of reasoning is that the imposition of strict safety codes on public buildings inhibits the evolution of architectural styles and design, because they discourage technological innovation within the building industry. furthermore, the strictness of the codes governing public buildings discourages technological innovation because the surest way for architects and builders to pass the codes is to construct buildings that use the same materials and methods that are currently allowed. this argument is unconvincing for two reasons.

in the first place, the author’s conclusion goes beyond the evidence presented. the evidence cited pertains only to the construction of public buildings, yet the author draws a conclusion about the building industry as a whole. technological innovation and architectural experimentation in style and design in the construction of private buildings is not precluded by the reasons cited. consequently, in the absence of evidence that similar problems beset the construction of privately owned buildings, the author’s conclusion is not warranted.

in the second place, it is not evident that the strict safety codes governing public buildings will have the effects predicted by the author. architectural styles and design are not dictated solely by the materials or the methods employed in construction. consequently, it is premature to conclude that little evolution in style and design will occur because the materials and methods will likely remain the same. moreover, technological innovation is not restricted to the use of new materials and methods. significant technological innovation can be achieved by applying existing methods to new situations and by finding new uses for familiar materials.

in conclusion, the author has failed to make the case for the claim that technological innovation as well as the evolution of architectural styles and design will be minimized in the future. to strengthen the argument the author would have to show that similar safety code restrictions impede the evolution of the design and the innovation of new technologies in the construction of private buildings. additionally, the author must show that materials and methods are the prime determinants of architectural style and design.

57. the following is from a campaign by big boards, inc., to convince companies in river city that their sales will increase if they use big boards billboards for advertising their locally manufactured products.“the potential of big boards to increase sales of your products can be seen from an experiment we conducted last year. we increased public awareness of the name of the current national women’s marathon champion by publishing her picture and her name on billboards in river city for a period of three months. before this time, although the champion had just won her title and was receiving extensive national publicity, only five percent of 15,000 randomly surveyed residents of river city could correctly name the champion when shown her picture; after the three-month advertising experiment, 35 percent of respondents from a second survey could supply her name.”

discuss how well reasoned... etc.

in an advertising experiment, big board, inc. displayed the name and picture of a. little-known athlete on several of its local billboards over a 3-month period. because the experiment increased recognition of the athlete’s name, big boards now argues that local companies will increase their sales if they advertise their products on big board’s billboards. this argument is unconvincing for two important reasons.

the main problem with this argument is that the advertising experiment with the athlete shows only that name recognition can be increased by billboard advertising; it does not show that product sales can be increased by this form of advertising. name recognition, while admittedly an important aspect of a product’s selling potential, is not the only reason merchandise sells. affordability, quality, and desirability are equally, if not more, important features a product must possess in order to sell. to suggest, as big board’s campaign does, that name recognition alone is sufficient to increase sales is simply ludicrous.

another problem with the argument is that while the first survey—in which only five percent of 15,000 randomly-selected residents could name the athlete—seems reliable, the results of the second survey are questionable on two grounds. first, the argument provides no information regarding how many residents were polled in the second survey or how they were selected. secondly, the argument does not indicate the total number of respondents to the second survey. in the absence of this information about the second survey, it is impossible to determine the significance of its results.

in conclusion, big board’s argument is not convincing. to strengthen the argument, big board must provide additional information regarding the manner in which the second survey was conducted. it must also provide additional evidence that an increase in name recognition will result in an increase in sales.

58. the following appeared as part of an article on government funding of environmental regulatory agencies.“when scientists finally learn how to create large amounts of copper from other chemical elements, the regulation of copper mining will become unnecessary. for one thing, since the amount of potentially available copper will no longer be limited by the quantity of actual copper deposits, the problem of overmining will quickly be eliminated altogether. for another, manufacturers will not need to use synthetic copper substitutes, the production of which creates pollutants. thus, since two problems will be settled—overmining and pollution—it makes good sense to reduce funding for mining regulation and either save the money or reallocate it where it is needed more.”

discuss how well reasoned... etc.

the author contends that it makes good sense to reduce funding for mining regulation, because regulatory problems with over-mining and pollution will be solved when scientists learn how to create large amounts of copper from other chemical elements. one reason the author gives for this conclusion is that the problem of over-mining will be quickly eliminated when the amount of potentially available copper is no longer limited by the quantity of actual copper deposits. another reason given is that pollution problems created by production of synthetic copper substitutes will be eliminated when manufacturers no longer depend on substitutes. this argument is weak because the conclusion goes beyond the scope of the premises and because the argument relies on questionable assumptions.

to begin with, the wording of the conclusion suggests that funding for mining regulation generally should be reduced, yet the premises are about copper mining only. there are many mined resources other than copper; advances in copper synthesis technology will in all likelihood have no bearing on whether regulation of other kinds of mining should be changed.

furthermore, the argument depends on the assumption that copper mining will slow down once copper can be chemically synthesized. however, the author provides no evidence to substantiate this assumption. moreover, it is entirely possible that copper mining will remain less expensive than copper synthesis. if so, there will be no incentives, outside of regulatory ones, to slow down copper mining. in a word, the problem of over-mining will remain.

finally, the argument relies on the assumption that synthesizing copper will not create the same kind of pollution problems as those resulting from the synthesis of copper substitutes. however, the author provides no evidence to substantiate this assumption. without such evidence, we cannot accept the premise that pollution problems will be eliminated by switching from producing copper substitutes to producing copper itself.

in conclusion, i am not convinced on the basis of this argument that the time has come to cut funding for the regulation of mining in general, or even for the regulation of copper mining in particular. to strengthen the argument, the author must restrict the scope of the conclusion to copper mining rather than to mining in general. the author must also provide support for the two assumptions underlying the argument.

59. the following appeared as part of an article in a popular science magazine.“scientists must typically work 60 to 80 hours a week if they hope to further their careers; consequently, good and affordable allday child care must be made available to both male and female scientists if they are to advance in their fields. moreover, requirements for career advancement must be made more flexible so that preschool-age children can spend a significant portion of each day with a parent.”

discuss how well reasoned... etc.

this editorial argues that, since career advancement for scientists typically requires 60 to 80 hours of work per week, affordable all-day child care must be made available to scientists of both genders if they are to advance in their fields. moreover, the editorial urges that requirements for career advancement be made more flexible to insure that pre-school children can spend a significant amount of time each day with a parent. this argument is problematic in two crucial respects.

the major problem with the view expressed in the article is that inconsistent recommendations are endorsed in the argument. on the one hand, scientists are urged to put their children in all-day child-care facilities in order to advance their careers. on the other hand, they are encouraged to spend a significant amount of time each day with their children. obviously, scientists cannot be expected to adhere to both of these recommendations.

another problem is that the recommendations are based on the assumption that all, or at least most, scientists have young or preschool-age children. but the editorial provides no evidence to support this assumption, nor is this assumption very likely to be true. since, childless scientists or scientists whose children are old enough to take care of themselves will have no need for the services advocated in this article, it is doubtful that these recommendations will receive much widespread support.

in conclusion, this argument is unconvincing. to strengthen it, the author must show that most scientists have preschool children and consequently are in need of the recommended services. additionally, the author must address and resolve the apparent conflict between the recommendations.

60. the following appeared as part of a recommendation by one of the directors of the beta company.“the alpha company has just reduced its workforce by laying off fifteen percent of its employees in all divisions and at all levels, and it is encouraging early retirement for other employees. as you know, the beta company manufactures some products similar to alpha’s, but our profits have fallen over the last few years. to improve beta’s competitive position, we should try to hire a significant number of alpha’s former workers, since these experienced workers can provide valuable information about alpha’s successful methods, will require little training, and will be particularly motivated to compete against alpha.”

discuss how well reasoned... etc.

a director of beta company suggests that beta can improve its competitive position by hiring a significant number of former alpha company employees who have recently retired or been laid off. the director’s reasoning is that because alpha manufactures some products similar to beta’s, former alpha employees would be experienced and need little training, could provide valuable information about alpha’s successful methods, and would be particularly motivated to compete against alpha. the director’s argument is problematic in several respects.

first of all, the argument presupposes that alpha’s methods are successful. this is not necessarily the case. to the contrary, the fact that alpha has laid off 15 percent of its employees in every division and at every level suggests that alpha’s methods may have been unsuccessful and that downsizing was necessary for the company to minimize financial losses.

secondly, the director assumes that the former alpha employees hired by beta will be well-trained and valuable. during a typical lay-off, however, the best and most experienced employees are typically the last to be laid off. by following the director’s advice, beta would probably be hiring alpha’s least efficient and least experienced employees—that is, those who would be least valuable to beta.

thirdly, the author assumes that alpha and beta are sufficiently similar so that former alpha employees could provide special value for beta. however, we are informed only that beta manufactures “some products similar to alpha’s.” it is possible that former alpha employees have experience with only a small segment of beta’s product line, and thus have little inside information of any value to beta.

finally, the claim that former alpha employees would be motivated to compete against alpha is partially unwarranted. while many of those who were laid off may be so motivated, those who retired early from alpha probably departed on good terms with alpha, and would in any event be unmotivated to reenter the work force.

in conclusion, the argument fails to provide key facts needed to assess it. to better evaluate the director’s suggestion, we would need more information about why alpha reduced its work force, what type of workers left alpha and under what circumstances, and how similar alpha’s range of products is to beta’s.

61. the following appeared in the letters-to-the-editor section of a local newspaper.“muscle monthly, a fitness magazine that regularly features pictures of bodybuilders using state-of-the-art exercise machines, frequently sells out, according to the owner of skyview newsstand. to help maximize fitness levels in our town’s residents, we should, therefore, equip our new community fitness center with such machines.”

discuss how well reasoned... etc.

in this argument the author concludes that the new community fitness center should be equipped with the state-of-the-art exercise machines featured in muscle monthly magazine. in support of this recommendation two reasons are offered: (1) muscle monthly contains pictures of bodybuilders using such machines, and (2) muscle monthly is a popular magazine, as evidenced by the fact that it frequently sells out at the local newsstand. this argument is questionable on two counts.

first, a major implication of the argument is that the bodybuilders pictured using the machines in muscle monthly magazine reached their state of fitness as a result of using these machines. the only evidence offered to support this contention, however, is the pictures in the magazine. it is possible that the bodybuilders pictured use different equipment for their workouts and are merely posing with the machines for advertising purposes.

second, the author assumes that machines that are suitable for bodybuilding will also be suitable to help maximize the fitness levels of the town’s residents. this assumption is highly questionable. machines designed to increase muscle development are significantly different from those designed to increase cardiovascular fitness. consequently, it is unlikely that the machines pictured in the magazine will be of much use to help maximize the fitness levels of the town’s residents.

in conclusion, this argument is unconvincing. to strengthen the argument the author would have to show that the bodybuilders pictured using the exercise machines actually used the machines to reach their level of muscle development. additionally, the author would have to show that the machines were suitable for increasing the fitness levels of the persons using them.

62. the following appeared as part of an article in the business section of a local newspaper.“the cumquat cafe made a mistake in moving to a new location. after one year at the new spot, it is doing about the same volume of business as before, but the owners of the robowrench plumbing supply wholesale outlet that took over its old location are apparently doing better: robowrench is planning to open a store in a neighboring city.”

discuss how well reasoned... etc.

according to this newspaper article, the cumquat cafe made a mistake by relocating one year ago. the author supports this claim by pointing out that cumquat is doing about the same volume of business as before it moved, while robowrench plumbing supply outlet, which took over cumquat’s old location, is apparently “doing better” because its owners plan to open a new outlet in a nearby city. this argument suffers from several critical flaws.

to begin with, the two businesses are too dissimilar for meaningful comparison. cumquat’s old location may simply have been better suited to hardware, plumbing, and home improvement businesses than to cafes and restaurants. the article’s claim that cumquat made a mistake in moving fails to take this possibility into account.

secondly, the article’s claim that robowrench is “doing better” since it took over cumquat’s old location is too vague to be meaningful. the author fails to provide a second term of this comparison. we are not informed whether robowrench is doing better than before it moved, better than other plumbing stores, or better than cumquat. this uninformative comparison is worthless as evidence from which to judge the wisdom of cumquat’s decision to relocate.

thirdly, the claim that robowrench is doing better is unwarranted by the evidence. the mere fact that robowrench plans to open a new store in a nearby city does not by itself establish that business is good. it is possible that the purpose of this plan is to compensate for lackluster business at the current location. or perhaps the robowrench owners are simply exercising poor business judgment.

finally, the claim that cumquat made a mistake in moving may be too hasty, since the conclusion is based on only one year’s business at the new location. moreover, given the time it ordinarily takes for a business to develop a new customer base in a new location, the fact that cumquat’s volume of business is about the same as before it moved tends to show that the move was a good decision, not a mistake.

in conclusion, the claim that cumquat’s move was a mistake is ill-founded, since it is based on both poor and incomplete comparisons as well as on a premature conclusion. to better assess the argument, we need to know what the author is comparing robowrench’s performance to; we also need more information about the extent of robowrench’s success at this location and why its owners are opening a new store.

63. the following appeared in a memorandum from the director of human resources to the executive officers of company x.“last year, we surveyed our employees on improvements needed at company x by having them rank, in order of importance, the issues presented in a list of possible improvements. improved communications between employees and management was consistently ranked as the issue of highest importance by the employees who responded to the survey. as you know, we have since instituted regular communications sessions conducted by high-level management, which the employees can attend on a voluntary basis. therefore, it is likely that most employees at company x now feel that the improvement most needed at the company has been made.”

discuss how well reasoned... etc.(下文思路單一,兩段都是談調查方法的問題,還可以談後半部分,“採取了措施就一定能達到目的嗎?”)

the director of human resources concludes that most employees at company x feel that the improvement most needed at the company has been satisfactorily addressed. two reasons are offered in support of this conclusion. first, a survey of employees showed that the issue respondents were most concerned about was employee-management communication. second, the company has since instituted regular voluntary sessions for employees and management designed to improve communication. the director’s argument is questionable for two reasons.

to begin with, the validity of the survey is doubtful. lacking information about the number of employees surveyed and the number of respondents, it is impossible to assess the validity of the results. for example, if 200 employees were surveyed but only two responded, the conclusion that most of the employees ranked employee-management communication as the most pressing issue would be highly suspect. because the argument offers no evidence that would rule out interpretations such as this, the survey results are insufficient to support the author’s conclusion.

furthermore, even if the survey accurately ranks certain issues according to level of employee concern, the highest-ranked issue in the survey might not be the issue about which employees are most concerned. why? the improvement most needed from the point of view of the employees might not have appeared as one of the choices on the survey. for example, if the list of improvements presented on the survey was created by management rather than by the employees, then the issues of greatest concern to the employees might not be included on the list. lacking information about how the survey was prepared, it is impossible to assess its reliability. consequently, any conclusion based on it is highly questionable.

in conclusion, the director’s conclusion is not well-founded. to strengthen the argument, additional information regarding the way in which the employee survey was prepared and conducted is required.

64. the following appeared in a memorandum from the vice president of road food, an international chain of fast-food restaurants.“this past year, we spent almost as much on advertising as did our main competitor, street eats, which has fewer restaurants than we do. although it appeared at first that our advertising agency had created a campaign along the lines we suggested, in fact our total profits were lower than those of street eats. in order to motivate our advertising agency to perform better, we should start basing the amount that we pay it on how much total profit we make each year.”

discuss how well reasoned... etc.(下文哥們有逆向思維癖好)

in this memorandum, the vice president of road food suggests that the company motivate its advertising agency to perform better by basing the agency’s pay on the road food’s profits. in support of this suggestion, the vice president points out that although road food initially thought the ad agency was following company recommendations, competitor, street eats earned higher profits last year. the vice president also notes that street eats has fewer restaurants than road food, and that road food spent nearly as much money on advertising as street eats did. this argument is unconvincing, since it relies on dubious assumptions and comparisons.

first, the vice president assumes that the ad campaign caused the low profits. however, the vice president ignores many other factors that contribute to profitability. in particular, the fact that road food has been spending less advertising money per restaurant than street eats suggests that its unwillingness to spend more may be the main reason for disappointing profits.

second, the author implies that the ad agency failed to implement road food’s guidelines, and that this failure was the reason for disappointing profits. however, it is equally possible that the ad agency faithfully followed all suggestions from road food, and that those suggestions were the cause of the disappointing profits. in this respect, the author unfairly shifts blame from road food to the ad agency.

third, the author’s comparison between road food and street eats is less relevant than a comparison between road food’s own profits prior to its latest ad campaign and its profits during this campaign. comparing its own profits during these time periods would more accurately reflect the ad agency’s effectiveness than comparing profits of two different companies.

finally, the author assumes that the ad agency will be more motivated if its fee is based on road food profits. however, the author does not support this claim. in fact, given that road food’s profits have been lower than expected, it is just as likely that the ad agency would be less motivated by the suggested fee structure than by some other fee structure.

in conclusion, the argument is unconvincing as it stands. to strengthen it, the vice president must provide evidence that the ad campaign caused last year’s disappointing profits, and must examine and rule out other factors that may have contributed to disappointing profits.

65. the following appeared in the promotional literature for cerberus dog food.“obesity is a great problem among pet dogs, just as it is among their human owners. obesity in humans is typically caused by consuming more calories than the body needs. for humans, a proper diet for losing weight is a reduced-calorie diet that is high in fiber and carbohydrates but low in fat. therefore, the best way for dog owners to help their dogs lose weight in a healthy way is to restrict the dog’s diet to cerberus reduced-calorie dog food, which is high in fiber and carbohydrates but low in fat.”

discuss how well reasoned... etc.(下文思路單一,兩段都是談類比錯誤。還可以談人即使與狗有可比性,這種狗食也未必是“最好”的治療方案!)

in this argument the makers of cerberus dog food recommend their reduced-calorie product as the best way for dog owners to help their obese dogs lose weight. their reasoning in support of this recommendation is simple. to begin with, they point out that the best way to treat obesity in humans is by a reduced-calorie diet that is high in fiber and carbohydrates but low in fat. second, they indicate that reduced-calorie cerberus dog food is high in fiber and carbohydrates but low in fat. the conclusion drawn from this information is that cerberus dog food is the best way to treat obesity in dogs. this argument is unconvincing for a couple of reasons.

in the first place, the makers of cerberus dog food assume that the cause of obesity in dogs is the same as the cause in humans. given the vast differences between the exercise patterns and basic diets of humans and dogs, this assumption is highly dubious. lacking evidence to support this claim, the argument is unacceptable.

in the second place, the author assumes that the gastrointestinal systems of dogs and humans are sufficiently similar to ensure that treatment that is effective on humans will be equally effective on dos. again, this is a highly dubious assumption due to the obvious physiological differences between humans and dogs. since no evidence has been offered to support this assumption, it too can be rejected.

in conclusion, this argument is unconvincing. to strengthen the argument evidence is required to substantiate the assumption that dogs and humans are sufficiently similar in both their diets and their physiology to warrant similar treatment.

66. the following appeared in an article in a travel magazine.“after the airline industry began requiring airlines to report their on-time rates, speedee airlines achieved the number one on-time rate, with over 89 percent of its flights arriving on time each month. and now speedee is offering more flights to more destinations than ever before. clearly, speedee is the best choice for today’s business traveler.”

discuss how well reasoned... etc.

a travel magazine article claims that speedee airlines is the best choice for today’s business traveler. to support this claim, the author points out that speedee has ranked first in terms of on-time arrival rate since the airline industry began requiring airlines to report their on-time rates. the claim is also based on the assertion that “speedee new offers more flights to more destination than ever before.” this argument suffers from several critical flaws.

first of all, the claim relies on a couple of unwarranted assumptions. one assumption is that on-time rates, number of flights, and destination choices are the only features of airlines service that determine how a particular airline would rank overall for a business traveler. however, the author of this article ignores other factors such as fare prices and discounts, safety record, baggage-handing, and in-flight amenities. another assumption is that speedee’s overall on-time record affects business and no-business travelers equally. however, this is not necessarily the case. speedee may have a poorer record for commuter flights, which are popular among business travelers, than for other flights. if so, the conclusion that speedee is the best choice for the business traveler would be seriously undermined.

secondly, the author’s claim that “speedee now offers more flights to more destinations than ever before” is too vague to be meaningful. we are not informed how many flights or how many destinations were previously offered or how many are offered now. moreover, the article makes no comparison with other airlines regarding these features. without these comparisons, the claim is worthless as a reason for choosing speedee over another airline.

thirdly, the article fails to indicate how long ago the industry began requiring airlines to report on-time rates. if the requirement was imposed recently, then the brief reporting period may be insufficient to show that the airlines’ relative on-time performance will continue in the future. moreover, the article fails to provide evidence that all airlines, regardless of on-time record, actually reported, or that the reports are accurate.

in conclusion, the article’s claim that speedee is the best choice for the business traveler is unsubstantiated and may be too hasty. to better evaluate the article’s claim, we need more information about speedee;s other features that contribute to its overall appeal, about its on-time record for commuter flights specifically, and about the integrity and length of the reporting upon which the ranking was based.

67. the following appeared in a memorandum to the planning department of an investment firm.“costs have begun dropping for several types of equipment currently used to convert solar energy into electricity. moreover, some exciting new technologies for converting solar energy are now being researched and developed. hence we can expect that solar energy will soon become more cost efficient and attractive than coal or oil as a source of electrical power. we should, therefore, encourage investment in solario, a new manufacturer of solar-powered products. after all, solario’s chief executive was once on the financial planning team for ready-to-ware, a software engineering firm that has shown remarkable growth since its recent incorporation.”

discuss how well reasoned... etc.

in this argument the planning department of investment firm reaches the conclusion that the firm should encourage investment in solario—a new manufacturer of solar-powered products. the basis for this recommendation is the expectation that solar energy will soon become more cost efficient and attractive than other forms of energy. this expectation is based on recent declines in the cost of equipment used to convert solar energy into electricity and on new technologies that are being developed for this purpose. an additional reason give in support of this recommendation is that solario’s chief executive was a member of the financial planning team for a company that has shown remarkable growth since its recent incorporation. while this argument has some merit, there are a few assumptions that deserve attention.

in the first place, the author assumes that the previous business experience of solario’s chief executive will be an asset in the development of the new company. while this may be the case, the fact that the two companies deal in vastly different products is cause for some concern. the executive’s expertise in the software-engineering business will not necessarily be applicable to the solar-powered products business.

in the second place, the author assumes that the major impediment to the use of solar-powered products is the cost of solar energy and that, given a choice, consumers would prefer products powered by solar energy over those powered by energy derived from coal or oil. on the face of it, this assumption seems acceptable; but it may be that there are other factors besides cost that make solar energy less desirable than other forms of energy.

in conclusion, this argument is convincing. to strengthen the argument additional evidence indicating consumer preference for solar-powered products over products powered by conventional forms of energy would be desirable.

68. the following appeared in a memorandum from a company’s marketing department.“since our company started manufacturing and marketing a deluxe air filter six months ago, sales of our economy filter—and company profits—have decreased significantly. the deluxe air filter sells for 50 percent more than the economy filter, but the economy filter lasts for only one month while the deluxe filter can be used for two months before it must be replaced. to increase repeat sales of our economy filter and maximize profits, we should discontinue the deluxe air filter and concentrate all our advertising efforts on the economy filter.”

discuss how well reasoned... etc.(此文不錯,共390字)

the company’s marketing department recommends discontinuing a deluxe air filter and concentrating advertising efforts on an economy filter, which requires replacement more often than the deluxe model. this recommendation is based on reports showing that sales of economy filters, and company profits, have dropped significantly since the company began manufacturing and marketing the deluxe filter six months ago. the marketing department’s argument is specious in three important respects.

first, the marketing department assumes that if the company discontinues the new deluxe air filter, customers will resume buying its economy filter. this assumption may not be correct. customers who prefer the deluxe model may do so because it requires replacement less often. thus, instead of buying the company’s economy filters again, these customers may just as likely turn to a competitor for a product similar to the deluxe model. in this event, the result would be lower profits.

secondly, the marketing department fails to recognize alternative strategies that might enhance profits more than discontinuing the deluxe filter would. it is possible that lowering the price of the economy model, raising the price of the deluxe model, or both, may actually maximize profits. a lower-priced economy filter might lure customers from competing products and retain current customers. at the same time, buyers of the deluxe model may place a premium value on its convenience and may be willing to pay an even higher price for the filter.

thirdly, the marketing department unfairly assumes that the availability of its deluxe filter is the cause of decreasing profits. it is equally possible that other factors, such as increased competition or supply prices, or decreased demand for these kinds of filters generally, are responsible for the decrease in profits. if so, discontinuing the deluxe filter may not serve to maximize, or even enhance, the company’s profits.

in conclusion, the department’s argument for discontinuing the deluxe filter is weak because the department has not considered the possible adverse consequences of doing so, or the alternatives to doing so. moreover, the department has failed to establish a clear causal connection between the availability of the deluxe filter and decreasing profits. to strengthen its argument, the department must consider and rule out pricing adjustments as a better strategy to maximize profits, and must provide better evidence that the deluxe filter is the cause of the decrease in profits.

69. the following appeared in a memorandum from the president of a company that makes (舊題有修飾詞glabrous) shampoo.“a widely publicized study claims that hr2, a chemical compound in our shampoo, can contribute to hair loss after prolonged use. this study, however, involved only 500 subjects. furthermore, we have received no complaints from our customers during the past year, and some of our competitors actually use more hr2 per bottle of shampoo than we do. therefore, we do not need to consider replacing the hr2 in our shampoo with a more expensive alternative.”

discuss how well reasoned... etc.

the president of the company that produces glabrous shampoo argues against removing the ingredient hr2 from the shampoo even though a scientific study claims that prolonged use of hr2 can contribute to hair loss. three reasons are cited as the basis for this decision. first, it is argued that since the scientific study involved only 500 subjects, it can be disregarded. second, none of glabrous’ customers have complained of problems during the past year. and, finally, glabrous’ competitors use more hr2 per bottle than glabrous. the president’s decision is problematic in several respects.

to begin with, the fact that the scientific study on hr2 involved only 500 subjects is insufficient grounds to dismiss the results of that study. if the subjects for the study were randomly chosen and represent a diverse cross section of the population of shampoo users, the results will be reliable regardless of the number of participants.

next, the scientific study determined that prolonged use could contribute to hair loss. while “prolonged use” was not defined in the memorandum, the fact that none of glabrous’ customers have complained of problems during the past year is not a reliable reason to believe that problems will not arise in the future.

finally, the fact that glabrous’ competitors use more hr2 in their products than glabrous uses is irrelevant to the question of whether glabrous should remove hr2 from its product. moreover, rather than providing a reason for not removing the compound, this fact serves better as a reason for doing so. by removing hr2 from its product glabrous could gain an edge over its competitors.

in conclusion, the reasoning in this argument is not convincing. to strengthen the argument the author would have to show that the study was biased or was based on too small a sample to yield reliable results.

70. the following appeared as part of a recommendation from the business manager of a department store.“local clothing stores reported that their profits decreased, on average, for the three-month period between august 1 and october 31. stores that sell products for the home reported that, on average, their profits increased during this same period. clearly, consumers are choosing to buy products for their homes instead of clothing. to take advantage of this trend, we should reduce the size of our clothing departments and enlarge our home furnishings and household products departments.”

discuss how well reasoned... etc.

based upon sales reports over a three-month period that indicate an increase in profits for stores that sell products for the home and a decrease in profits for clothing stores, the business manager of a department store concludes that consumers are choosing to purchase home furnishings rather than clothing. on the basis of this conclusion, the manager recommends a reduction in the size of the clothing department and an increase in the size of the home-furnishings department. this recommendation is problematic in two critical respects.

in the first place, the author’s conclusion that consumers are choosing to buy products for their homes instead of clothing is based upon too small a sample. data gathered from a three-month period is insufficient to establish the conclusion drawn from it. it is quite possible that the three-month period chosen is idiosyncratic and not representative of entire year’s sales. if so, reducing the size of the clothing departments and enlarging the home-furnishings departments may be a costly mistake.

in the second place, the data collected during the three month period may be biased. the fact that the data reflects sales in local stores is cause for concern. it is possible that the sales trend in a particular location is not representative of sales in other regions. for example, sales of clothing in florida during the winter months are likely to be quite different from sales of clothing in alaska during the same period.

in conclusion, this argument is not persuasive as it stands. a more convincing argument must provide additional sales data, collected at different periods of the year and at different locations, that substantiates the trend in question.

71. the following appeared in a letter to the editor of a regional newspaper.“in response to petitions from the many farmers and rural landowners throughout our region, the legislature has spent valuable time and effort enacting severe laws to deter motorists from picking fruit off the trees, trampling through the fields, and stealing samples of foliage. but how can our local lawmakers occupy themselves with such petty vandalism when crime and violence plague the nation’s cities? the fate of apples and leaves is simply too trivial to merit their attention.”

discuss how well reasoned... etc.

the author of this editorial asserts that trespassing, vandalism, and theft associated with stealing fruit from farms is a trivial problem and, as a result, enacting laws to protect farm- and land-owners from these crimes is a waste of lawmakers’ time. in support of this claim, the author points out only that the nation’s cities are plagued by far more serious problems of violence and crime. to the extent that this author has provided any argument at all, it is a poor one.

first of all, the author unfairly assumes that if lawmakers are taking rural crime issues seriously, then they cannot be taking urban crime issues seriously. the author is presenting a false dilemma by imposing an either-or choice between two courses of action that need not be mutually exclusive. it is equally possible that legislators can address both areas of concern concurrently.

secondly, the argument relies on the assumption that the legislators in question have the opportunity to address urban crime problems. however, we are not told whether this legislature’s jurisdiction encompasses both rural and urban areas. if it encompasses only rural areas, then the author’s implicit conclusion that the legislators in this region should instead be addressing urban crime problems would be completely undermined.

finally, the author unfairly trivializes the severity of rural crime by simply comparing it with urban crime. while trespassing, vandalism, and fruit-stealing may seem minor peccadilloes, especially compared to violent urban crimes, these rural crimes might nevertheless result in serious financial damage to farm owners, depending on the frequency and extent of the violations. the author fails to provide evidence for the claim that these rural crimes are trivial. instead, the author attempts to call attention to a more dramatic but potentially irrelevant problem.

in conclusion, the argument is weak. it potentially distorts the alternatives available to legislators in the region, as well as deflecting attention from the problem at hand. to better evaluate it, we would need more information about the geographical scope of this legislature’s jurisdiction and about the extent of the fruit-stealing problem in the region.

72. the following appeared as part of an editorial in a campus newspaper.“with an increasing demand for highly skilled workers, this nation will soon face a serious labor shortage. new positions in technical and professional occupations are increasing rapidly, while at the same time the total labor force is growing slowly. moreover, the government is proposing to cut funds for aid to education in the near future.”

discuss how well reasoned... etc.

in this argument the author predicts a nationwide labor shortage in the near future. the basis for this prediction is an increasing demand for highly skilled workers, especially in technical and professional fields, coupled with a slow-growing labor force and a government proposal to cut funds for aid to education. at first glance, the author’s argument appears to be somewhat convincing: but further reflection reveals that it is based on some dubious assumptions.

in the first place the author assumes that the present labor force is immobile and that the demand “for highly skilled workers will have to be met by workers who are entering the labor market for the first time. recent american history, however, shows that this assumption is entirely unfounded. at the beginning of the industrial revolution most americans were farm workers, but by the end of that revolution most had become factory workers. thus, even though the labor pool remained relatively constant during this period, the number of farm workers decreased and the number of factory workers increased. this example clearly demonstrates the mobility of the labor force.

in the second place, the author assumes that the government proposal to cut funds for aid to education will have a significant negative impact on the ability to train workers in technical and professional fields. the fact is, however, that the percentage of students who rely on government aid for their education is relatively small, so the effect of such cuts would be negligible.

in conclusion, this argument is unconvincing. to strengthen the argument the author would have to show that the present work force was relatively static and that the proposed cut in educational aid would have a deleterious effect on the numbers of high skilled workers available to enter the work force in the future.

73. the following appeared as part of a memorandum from a government agency.“given the limited funding available for the building and repair of roads and bridges, the government should not spend any money this year on fixing the bridge that crosses the styx river. this bridge is located near a city with a weakening economy, so it is not as important as other bridges; moreover, the city population is small and thus unlikely to contribute a significant enough tax revenue to justify the effort of fixing the bridge.”

discuss how well reasoned... etc.

the author of this government agency memorandum argues that the government should not spend any money this year fixing the bridge that crosses the styx river, given the limited resources available for building and repair of roads and bridges. the author reasons that this bridge is less important than others because it is located near a city with a weakening economy, and because the city’s small population is unlikely to contribute enough tax revenue to justify fixing their bridge. this argument is unconvincing for four reasons.

first of all, the author unfairly assumes that the importance of a bridge is determined solely by the economic condition of nearby cities. this assumption overlooks other criteria for determining a bridge’s importance—such as the number of commuters using the bridge, the role of the bridge in local emergencies and disasters, and the impact that bridge closure would have on the economies of nearby cities. without accounting for these other potential factors, the author fails to provide a convincing argument that the styx river bridge is unimportant.

secondly, the author fails to provide any evidence that other bridges are more important than the styx river bridge. without such evidence, we cannot accept the author’s conclusion that no government funds should be directed toward maintaining the styx river bridge.

thirdly, the fact that the nearby city has a weakening economy does not prove that the city will not contribute significantly to tax revenues. perhaps tax revenues are based on property taxes, which are not related directly to economic conditions. if so, and if property values and taxes are high in this nearby city, then the city would contribute significantly to tax revenues, and the bridge would be important to maintain those property values and the revenues they generate.

finally, the author assumes that a city should receive government services commensurate with the tax dollars it contributes. substantiating this assumption requires examining the proper duty of government. however, the author provides no such examination. accordingly, this assumption is simply an unproven claim.

in conclusion, this editorial fails to substantiate its claim that the styx river bridge is not important enough for the government to spend tax dollars to maintain and repair it. to strengthen the argument, the author must account for other factors that also determine a bridge’s importance, and must compare the importance of this bridge relative to other bridges.

74. the following appeared as part of an article in an entertainment magazine.“a series of books based on the characters from a popular movie are consistently bestsellers in local bookstores. seeking to capitalize on the books’ success, vista studios is planning to produce a movie sequel based on the books. due to the success of the books and the original movie, the sequel will undoubtedly be profitable.”(本題存在歸因錯誤、類比錯誤、偷換概念(bestseller未必就profitable))

discuss how well reasoned... etc.

in this argument the author reasons that a sequel to a popular movie will be profitable because the original movie was profitable and because books based on the characters of the movie are consistently bestsellers. this argument is unconvincing for several reasons.

in the first place, a great deal of empirical evidences shows that sequels are often not as profitable as the original movie. for example, none of the “superman” movie sequels even approached the success of the original movie. accordingly, the mere fact that the first movie was successful does not guarantee that movies based upon it will also be profitable.

in the second place, a movie’s financial success is a function of many elements in addition to well-liked characters. admittedly, the fact that the books based on the characters of the original film are bestsellers bodes well for the movie’s commercial prospects. however, unless the original cast and production team are involved in making the sequel, there is a good chance it will not be financially successful.

finally, another important element in creating a successful movie is the script. the transformation of a popular book into a popular movie script is a difficult process. examples of best-selling books that were not made into successful movies are commonplace. obviously, the success of the sequel that vista is planning will depend in great part on the screenwriter’s ability to capture the elements of the story that make the books popular. since the difficulties inherent in this process make it hard to predict whether the result will be a success or a failure, the conclusion that the sequel will be profitable is presumptuous.

in conclusion this is an unconvincing argument. to strengthen the argument, it would be necessary to provide assurances that the original cast and production team will be involved in the project and that the script will capture and develop the particular elements responsible for the books’ popularity.

75. the following appeared in a letter to the editor of a popular science and technology magazine.“it is a popular myth that consumers are really benefiting from advances in agricultural technology. granted—consumers are, on the average, spending a decreasing proportion of their income on food. but consider that the demand for food does not rise in proportion with real income. as real income rises, therefore, consumers can be expected to spend a decreasing proportion of their income on food. yet agricultural technology is credited with having made our lives better.”

discuss how well reasoned... etc.

the conclusion of this letter is that consumers are not truly benefiting from advances in agricultural technology. the author concedes that, on the average, consumers are spending a decreasing proportion of their income on food. but the author contends that this would happen without advances in agricultural technology. the author reasons that demand for food does not rise in proportion with real income, so as real income rises, consumers will spend a decreasing portion of their income on food. this argument turns on a number of dubious assumptions.

first of all, while asserting that real incomes are rising, the author provides no evidence to support this assertion; moreover, it might be false. even if salaries and wages go up, this fact may not indicate that real income has increased proportionally. real income takes into account any effect inflation might have on the relative value of the dollar. it is possible that, when salaries and wages are adjusted for inflation, what appear to be increases in real income are actually decreases.

in addition, the author assumes that increases in real income explain why, on the average, consumers are now spending a decreasing proportion of their income on food. but no evidence is provided to show that this explanation is correct. moreover, the author fails to consider and rule out other factors that might account for proportional decreases in spending on food.

finally, the entire argument turns on the assumption that benefits to consumers from advances in agricultural technology are all economic ones—specifically, ones reflected in food prices. the author ignores other likely benefits of agricultural technology that affect food prices only indirectly or not at all. such likely benefits include increased quality of food as it reaches the market and greater availability of basic food items. moreover, the author cannot adequately assess the benefits of agricultural technology solely on the basis of current food prices because those prices are a function of more than just the technology that brings the food to market.

in conclusion, this letter has provided little support for the claim that consumers are not really benefiting from advances in agricultural technology. a stronger argument would account for the benefits of technology other than the current price of food, and would account for other factors that affect food prices. to better evaluate the argument, we would need more information about whether real incomes are actually rising and whether this alone explains why consumers now spend a proportionately smaller amount of income on food.

76. the following appeared in the editorial section of a local newspaper.“this city should be able to improve existing services and provide new ones without periodically raising the taxes of the residents. instead, the city should require that the costs of services be paid for by developers who seek approval for their large new building projects. after all, these projects can be highly profitable to the developers, but they can also raise a city’s expenses and increase the demand for its services.”

discuss how well reasoned... etc.(此文似乎過分短!)

in this editorial the author argues that improvements to existing city services as well as new services should be paid for by developers rather than by taxpayers. in support of this opinion the author points out that developers can make large profits from building projects and that these projects increase the demand for city services and raise the city’s expenses, i disagree with the author’s opinion for two reasons.

first, the fact that developers stand to make profits from their projects is not a good reason to require them to pay more than their fair share of the costs of services. in fact, to require them to do this in order to win approval of their projects is tantamount to robbery. city officials would find it difficult to justify a policy that endorsed this practice. moreover, the adoption of such a practice would discourage the development of new buildings in the city.

second, the increase in demand for city services as well as the increase in the city’s expenses will be most likely offset by the tax revenues these projects generate. consequently, unless the author can demonstrate that the city will incur expenses that are not covered by the increased revenues from these projects, the author’s concern about these issues is unfounded.

in conclusion, i find the author’s reasoning on this issue unconvincing. to strengthen the argument the author would have to show that the city would be harmed financially by approving new building projects.

77. the following appeared in the editorial section of a local newspaper.“in order to avoid the serious health threats associated with many landfills, our municipality should build a plant for burning trash. an incinerator could offer economic as well as ecological advantages over the typical old-fashioned type of landfill: incinerators can be adapted to generate moderate amounts of electricity, and ash residue from some types of trash can be used to condition garden soil.”

discuss how well reasoned... etc.

this newspaper editorial concludes that our city should build a plant for burning trash in order to avoid the serious health threats associated with many landfills. the author adds that an incinerator could offer economic benefits as well, since incinerators can be adapted to generate small amounts of electricity for other uses, and since ash residue from some kinds of trash can be used as a soil conditioner. even if these claims are true, the author’s argument is unconvincing in three important respects.

to begin with, the author fails to consider health threats posed by incinerating trash. it is possible, for example, that respiratory problems resulting from the air pollution caused by burning trash might be so extensive that they would outweigh the health risks associated with landfills. if so, the author’s conclusion that switching to incineration would be more salutary for public health would be seriously undermined.

secondly, the author assumes that discontinuing landfill operations would abate the heath threats they now pose. however, this is not necessarily the case. it is possible that irreversible environmental damage to subterranean water supplies, for example, has already occurred. in this event, changing from landfills to incinerators might not avoid or abate serious public health problems.

thirdly, the author’s implicit claim that incinerators are economically advantageous to landfills is poorly supported. only two small economic benefits of incineration are mentioned, while the costs associated with either burning trash or switching refuse disposal systems are ignored. in all likelihood, such costs would be significant, and may very well outweigh the economic benefits.

in conclusion, the author’s argument provides inadequate justification for switching from one disposal system to the other. as it stands, the argument takes into account only a limited number of benefits from the change, while addressing none of its costs. to better evaluate the argument, we must first examine all the health risks posed by each refuse disposal system and conduct a thorough cost-benefit analysis of each system, taking account of the cost of the new system, the cost of the changeover itself, and the expected costs to the community of health problems resulting from each system.

78. the following appeared in the editorial section of a monthly business newsmagazine.“most companies would agree that as the risk of physical injury occurring on the job increases, the wages paid to employees should also increase. hence it makes financial sense for employers to make the workplace safer: they could thus reduce their payroll expenses and save money.”

discuss how well reasoned... etc.

in this editorial, the author argues that it makes financial sense for employers to make the workplace safer, in support of this claim the author reasons that since wages paid to employees should increase as the risk of physical injury increases, the converse should be true as well. hence, by decreasing the risk of injury, employers could decrease the wages paid to workers and thereby save money. this argument is unconvincing for two reasons.

to begin with the author assumes that because companies would agree that as risk of injury increases wages should also increase, they would also agree that as risk decreases wages should also decrease accordingly. this is tantamount to the assumption that risk of injury is the primary factor that determines workers’ wages. it is obvious that few employers, and even fewer employees, would agree that this is the case. to adopt this position one would have to disregard education, experience, and skill as equally important factors in determining the wages paid to workers.

secondly, the author’s reasoning suggests that the only benefit of a safer workplace is the savings employers could realize from lower wages. this is obviously not true. the costs associated with accidents on the job could far outweigh any savings that could be realized by paying workers lower wages.

in conclusion, the author’s argument is unconvincing. risk of injury is an important factor to consider in determining the wages paid to workers but is not the only such factor. furthermore, there are far better reasons for employers to make the workplace safer than the one presented by the author.

79. the following appeared as part of a company memorandum.“adopting an official code of ethics regarding business practices may in the long run do our company more harm than good in the public eye. when one of our competitors received unfavorable publicity for violating its own code of ethics, it got more attention from the media than it would have if it had had no such code. rather than adopt an official code of ethics, therefore, we should instead conduct a publicity campaign that stresses the importance of protecting the environment and assisting charitable organizations.”

discuss how well reasoned... etc.

this company memorandum suggests that, in lieu of adopting an official code of ethics, the company should conduct a publicity campaign that stresses the importance of promoting certain societal interests. the reason for the suggestion is that an official code of ethics might harm the company in the public eye because a competing company received unfavorable publicity for violating its own ethics code. this argument is unconvincing, since it depends on several unwarranted assumptions as well as arguing against its own conclusion.

first of all, the author unfairly assumes that the two companies are sufficiently similar to ensure the same consequences of adopting an ethics code for this company as for its competitor. the competitor may have adopted an entirely different code from the one this company might adopt—perhaps with unrealistic standards not embraced by any other companies. perhaps the competitor’s violation was extremely egregious, amounting to an aberration among businesses of its type; or perhaps one notorious executive is solely responsible for the competitor’s violation. any of these scenarios, if true, would show that the two companies are dissimilar in ways relevant to the likelihood that this company will experience similar violations and similar publicity if it adopts any ethics code.

secondly, the author unfairly assumes that the competitor was damaged by its code violation and the resulting publicity more than it would have been had it not violated its code. just as likely, however, the violation was necessary to ensure a certain level of profitability or to protect other important interests. without knowing the extent and nature of the damage resulting from the bad publicity or the reason for the violation, we cannot accept the author’s conclusion.

thirdly, the author’s proposal is inconsistent with the author’s conclusion about the consequences of adopting an ethics code. the author suggests that, instead of adopting an ethics code, this company should stress “the importance of protecting the environment and assisting charitable organizations.” this proposal is tantamount to adopting an ethics code. in this sense, the author suggests going against his own advice that the company should not adopt such a code.

in conclusion, differences between this company and its competitor may undermine the author’s conclusion that this company should not adopt an ethics code. to better evaluate the argument, we need more information about the nature of the competitor’s ethics code and about the nature and extent of the violation. to strengthen the argument, the author must accord his advice with his conclusion that the company should not adopt an ethics code.

80. the following appeared in the editorial section of a daily newspaper.“although forecasts of presidential elections based on opinion polls measure current voter preference, many voters keep changing their minds about whom they prefer until the last few days before the balloting. some do not even make a final decision until they enter the voting booth. forecasts based on opinion polls are therefore little better at predicting election outcomes than a random guess would be.”

discuss how well reasoned... etc.

in this editorial the author asserts that opinion polls are little better than random guesses to predicting outcomes of presidential elections. the author’s basis for this assertion is that opinion polls measure only the preferences of voters at the time of the poll and that many voters change their preferences several time before voting—some remaining undecided until the moment they cast their vote. the author’s reasoning is unconvincing in two critical respects.

first of all the predictions based on random guessing are such that the greater the number of candidates, the less likely the prediction will be correct. the reason for this is obvious: random guessing requires that no outside information be allowed to influence the guess. predictions based on opinion polls, on the other hand, will differ considerably from those based on random guesses simply because outside information will influence the result. for example, in a four-person race, random guessing would yield the correct prediction 25 percent of the time, whereas the percentage of correct predictions based on opinion polls would be much higher. the reason for this disparity is simple. opinion polls enable us to narrow the choices. that is, opinion polls serve to reduce the number of viable candidates in the voter’s mind and thereby increase the likelihood that the prediction based on them will be correct.

in addition, while it is true that many voters change their minds several times before voting, and that some remain undecided until entering the voting booth, this is not true of everyone. moreover, people who do change their minds frequently or wait until the last moment to decide have typically narrowed their choice to a few candidates.

in conclusion, the author is mistaken in believing that random guessing would be as reliable as opinion polls in predicting the outcomes of presidential elections.

81. the following appeared in the editorial section of a newspaper in the country of west cambria.“the practice of officially changing speed limits on the highways—whether by increasing or decreasing them—is a dangerous one. consider what happened over the past decade whenever neighboring east cambria changed its speed limits: an average of 3 percent more automobile accidents occurred during the week following the change than had occurred during the week preceding it—even when the speed limit was lowered. this statistic shows that the change in speed limit adversely affected the alertness of drivers.”

discuss how well reasoned... etc.

this editorial asserts that west cambria should not change its highway speed limits because such changes adversely affect driver alertness and are therefore dangerous. to support this claim, the editorial cites statistics indicating that whenever east cambria changed its speed limits, an average of 3 percent more automobile accidents occurred during the week after the change than during the week preceding it, even when the speed limit was lowered. as it stands, this argument suffers from three critical flaws.

first, it is unlikely that the brief one-week periods under comparison are representative of longer time periods. a difference of only 3 percent during one particular week can easily be accounted for by other factors, such as heavy holiday traffic or bad weather, or by problems with reporting or sampling. had the editorial indicated that several speed-limit changes in east cambria contributed to the statistic, the argument would be more convincing; but for all we know, the statistic is based on only one such change. in any event, a one-week period is too brief to be representative because it is likely that accidents will occur more frequently immediately following the change, while people adjust to the new limit, than over the longer term when drivers have become accustomed to the change.

secondly, the editorial fails to acknowledge possible differences in the types of accidents occurring before and after the change. it is possible that the accidents during the week before the change all involved fatalities, while those during the week after the change were minor fender-benders. if so, even though 3 percent more accidents occurred after the change, the author’s argument that changing the speed limit increases danger for drivers would be seriously weakened.

thirdly, the editorial fails to take into account possible differences between east and west cambria that are relevant to how drivers react to speed-limit changes. factors such as the condition of roads, average age and typical driving habits of residents, and weather patterns, would probably affect how well or how quickly drivers adapt to speed-limit changes. thus, changing speed limits in east cambria might be more dangerous than changing them in west cambria.

in conclusion, the statistical evidence cited to support the argument is insignificant and probably unrepresentative. to better evaluate the argument, we need to know how many speed-limit changes contributed to the statistic and when the speed-limit changes were made. finally, to strengthen the argument the author should show that east and west cambria would be similarly affected by speed-limit changes.

82. the following appeared as part of a memorandum from the vice president of nostrum, a large pharmaceutical corporation.“the proposal to increase the health and retirement benefits that our employees receive should not be implemented at this time. an increase in these benefits is not only financially unjustified, since our last year’s profits were lower than those of the preceding year, but also unnecessary, since our chief competitor, panacea, offers its employees lower health and retirement benefits than we currently offer. we can assume that our employees are reasonably satisfied with the health and retirement benefits that they now have since a recent survey indicated that two-thirds of the respondents viewed them favorably.”

discuss how well reasoned... etc.

the vice president of nostrum argues that implementing an increase in health and retirement benefits for employees is not a good idea at this time. his main line of reasoning is that an increase in benefits is both financially unjustified and unnecessary—financially unjustified because last year’s profits were lower than the preceding year’s, and unnecessary because nostrum’s chief competitor offers lower benefits to its employees and because a recent nostrum employee survey indicates that two-thirds of the respondents viewed the current benefits package favorably. while the argument has some merit, it is not completely convincing.

admittedly the vice president’s reasoning linking employee benefits with company profits seems reasonable on its face. companies that are not profitable are ill-advised to take on additional costs such as increased employee benefits. however, the fact that nostrum’s profits last year were lower than the preceding year does not imply that nostrum is experiencing financial difficulties that preclude it from increasing employee benefits at this time. perhaps the previous year’s profits were extremely large; whereas last year’s profits, albeit lower, were sufficient to fund an increase in the benefits package without threatening the company’s bottom line.

also, the fact that nostrum’s chief competitor provides lower benefits to its employees is not a good reason for nostrum to deny an increase to its employees. employee loyalty is an important asset to any company, and providing good pay and good benefits are among the best ways to acquire it. nostrum would be well advised to assure that its employees have little reason to seek employment elsewhere, and especially from its chief competitor.

finally, one can infer from the survey’s results that a full one-third of the respondents may have viewed the current benefits package unfavorably. if so, such widespread satisfaction would weaken the vice president’s argument. lacking more specific information about how these other employees responded, it is impossible to assess the reliability of the survey’s results or to make an informed recommendation.

in conclusion the vice president’s argument against implementing a benefits increase is unconvincing. to strengthen the argument, he must provide evidence that the increase in benefits would have a negative impact on the company’s overall profitability. additionally, he must provide more information about the manner in which the survey was conducted before we can determine the degree of employee satisfaction of the current benefits

83. the following appeared as part of an article on trends in television.“a recent study of viewers’ attitudes toward prime-time television programs shows that many of the programs that were judged by their viewers to be of high quality appeared on (noncommercial) television networks, and that, on commercial television, the most popular shows are typically sponsored by the best-selling products. thus, it follows that businesses who use commercial television to promote their products will achieve the greatest advertising success by sponsoring only highly-rated programs—and, ideally, programs resembling the highly-rated noncommercial programs on public channels as much as possible.”

discuss how well reasoned... etc.

this article concludes that businesses using commercial television to promote their products will achieve the greatest advertising success by sponsoring only highly-rated programs—preferably, programs resembling the highly-rated non-commercial programs on public channels. supporting this claim is a recent study indicating that many programs judged by viewers to be high in quality appeared on noncommercial networks, and that the most popular shows on commercial television are typically sponsored by the best-selling products. this argument is weak because it depends on three questionable assumptions.

the first of these assumptions is that noncommercial public television programs judged by viewers to be high in quality are also popular. however, the study cited by the author concerns viewer attitudes about the “high quality” of programs on noncommercial public television, not about their popularity. a program might rate highly as to quality but not in terms of popularity. thus, the author unfairly assumes that highly-rated public television programs are necessarily widely viewed, or popular.

the argument also assumes that programs resembling popular non-commercial programs will also be popular on commercial television. however, the audiences for the two types of programs differ significantly in their tastes. for example, a symphony series may be popular on public television but not as a prime-time network show, because public-television viewers tend to be more interested than commercial-television viewers in the arts and higher culture. thus, a popular program in one venue may be decidedly unpopular in the other.

a third assumption is that products become best-sellers as a result of their being advertised on popular programs. while this may be true in some cases, it is equally possible that only companies with products that are already best-sellers can afford the higher ad rates that popular shows demand. accordingly, a lesser-known product from a company on a smaller budget might be better off running repeated—but less expensive—ads on less popular shows than by running just one or two costly ads on a top-rated show.

in conclusion, the results of the cited study do not support the author’s conclusion. to better evaluate the argument, we need to know the intended meaning of the phrase “highly-rated.” to strengthen the argument, the author must limit his conclusion by acknowledging that popularity in public television might not translate to popularity in commercial television, and that the best advertising strategy for companies with best-selling products may not be feasible for other businesses.

84. the following appeared as part of an article in the business section of a daily newspaper.“company a has a large share of the international market in video-game hardware and software. company b, the pioneer in these products, was once a $12 billion-a-year giant but collapsed when children became bored with its line of products. thus company a can also be expected to fail, especially given the fact that its games are now in so many american homes that the demand for them is nearly exhausted.”

discuss how well reasoned... etc.

in this argument the author reasons that the failure of company b portends a similar fate for company a. the grounds for this prediction are similarities that exist between the two companies. the line of reasoning is that since both companies produce video-game hardware and software and both enjoy a large share of the market for these products, the failure of one is a reliable predictor of the failure of the other. this argument is unconvincing.

the major problem with the argument is that the stated similarities between company a and b are insufficient to support the conclusion that company a will suffer a fate similar to company b’s. in fact, the similarities stated are irrelevant to that conclusion. company b did not fail because of its market share or because of the general type of product it produced; it failed because children became bored with its particular line of products. consequently, the mere fact that company a holds a large share of the video-game hardware and software market does not support the claim that company a will also fail.

an additional problem with the argument is that there might be relevant differences between company a and company b, which further undermine the conclusion. for example, company a’s line of products may differ from company b’s in that children do not become bored with them. another possible difference is that company b’s share of the market may have been entirely domestic whereas company a has a large share of the international market.

in conclusion this is a weak argument. to strengthen the conclusion the author would have to show that there are sufficient relevant similarities between company a and company b as well as no relevant differences between them.

85. the following appeared as part of an article in a photography magazine.“when choosing whether to work in color or in black-and-white, the photographer who wishes to be successful should keep in mind that because color photographs are more true-to-life, magazines use more color photographs than black-and-white ones, and many newspapers are also starting to use color photographs. the realism of color also accounts for the fact that most portrait studios use more color film than black-and-white film. furthermore, there are more types of color film than black-and-white film available today. clearly, photographers who work in color have an advantage over those who work in black-and-white.”

discuss how well reasoned... etc.

the author concludes that photographers who work in color hold a competitive advantage over those who work in black-and-white. to support this conclusion, the author claims that the greater realism of color accounts for its predominant use in magazines and portraits. the author also points out that newspapers now use color photographs, and that there are more types of color film than black-and-white film available today. this argument is problematic in several important respects.

first, the argument unfairly assumes that working in color is necessary in order to gain an advantage. the author identifies only two areas—magazine and portrait photography—where color predominates. it is possible that the overall demand for black-and-white photography remains high. moreover, the author provides no evidence that the realism of color photography is the reason for its predominance. the predominant use of color may be due to other factors—such as consumer preferences or relative costs of film—which might change at any time.

second, the argument unfairly assumes that a photographer must make an either/or choice between the two types of photography. this assumption presents a false dilemma, since the two media are not necessarily mutually exclusive alternatives. common sense tells us that a photographer can succeed by working in both media.

third, the fact that more kinds of color film are available than black-and-white film accomplishes little to support the argument. the difference in number might be insignificant, and the distinctions among the types of color film might be negligible. in fact, by implying that more choices in film type affords a photographer a competitive advantage, the author actually undermines his larger argument that working solely in color is the best way to succeed in the field of photography.

finally, the argument ignores other factors—such as initiative, creativity, technical skills, and business judgment—that may be more important than choice of medium in determining success in photography. a poorly skilled photographer may actually be disadvantaged by working in color insofar as color work requires greater skill, and insofar as color photographers face keener competition for assignments.

in conclusion, this argument oversimplifies the conditions for gaining an advantage in the field of photography. to better evaluate the argument, we need more precise information as to how large a portion of all photography work today is accounted for by color work. to strengthen the argument, the author must convince us that a photographer must choose one medium or the other rather than working in both.

86. the following appeared as part of a letter to the editor of a local newspaper.“it makes no sense that in most places fifteen year olds are not eligible for their driver’s license while people who are far older can retain all of their driving privileges by simply renewing their license. if older drivers can get these renewals, often without having to pass another driving test, then fifteen year olds should be eligible to get a license. fifteen year olds typically have much better eyesight, especially at night; much better hand-eye coordination; and much quicker reflexes. they are also less likely to feel confused by unexpected developments or disoriented in unfamiliar surroundings, and they recover from injuries more quickly.”

discuss how well reasoned... etc.

the conclusion of this argument is that 15-year-olds should be eligible to obtain a driver’s license. the author employs two lines of reasoning to reach this conclusion. in the first the author reasons that since older drivers can retain their driving privileges by simply renewing their licenses, 15-year-olds should be eligible to obtain a license. in the second, the author reasons that 15-year-olds are physically more capable than older drivers of performing the various skills associated with driving a vehicle and thus should be eligible to get a license. this argument is unconvincing for a couple of reasons.

in the first place, the author assumes that there are no relevant differences between 15-year-olds and older drivers that would justify treating them differently. this assumption is dearly mistaken. the major difference between the two groups, and the major reason 15-year-olds are denied driving privileges, is their relative lack of emotional maturity and social responsibility. this difference is sufficient to justify the policy of allowing older drivers to renew their driving privileges while at the same time denying these privileges to 15-year-olds.

in the second place, even if it is granted that fifteen year olds possess better night vision, reflexes, hand-eye coordination, and are less disoriented in unfamiliar surroundings than older drivers, these abilities do not qualify them to obtain a driver’s license. the author assumes that physical capabilities are the only attributes necessary to operate a motor vehicle. but this assumption is clearly mistaken. in addition to these abilities, drivers must be able to exercise good judgment in all types of driving situations and conditions and must be cognizant of the consequences of their decisions and actions when driving. it is because 15-year-olds typically lack these latter abilities that they are denied driving privileges.

in sum, the author’s argument fails to take into consideration important differences between older drivers and 15-year-olds that justify denying driving privileges to the younger group while at the same time allowing older drivers to retain their privileges by simply renewing their license.

87. the following appeared in an ad for a book titled how to write a screenplay for a movie.“writers who want to succeed should try to write film screenplays rather than books, since the average film tends to make greater profits than does even a best-selling book. it is true that some books are also made into films. however, our nation’s film producers are more likely to produce movies based on original screenplays than to produce films based on books, because in recent years the films that have sold the most tickets have usually been based on original screenplays.”

discuss how well reasoned... etc.

this advertisement for “how to write a screenplay...” concludes that a writer is more likely to be successful by writing original screenplays than by writing books. the ad’s reasoning is based on two claims: (1) the average film tends to be more profitable than even best-selling books, and (2) film producers are more likely to make movies based on original screenplays than on books because in recent years the films that have sold the most tickets have usually been based on original screenplays. i find the ad unconvincing, on three grounds.

first, the mere fact that ticket sales in recent years for screenplay-based movies have exceeded those for book-based movies is insufficient evidence to conclude that writing screenplays now provides greater financial opportunity for writers. ticket-sale statistics from only a few recent years are not necessarily a good indicator of future trends. it is possible that fees paid by movie studios for screenplays might decrease in the future relative to those for book rights. moreover, the argument is based on number of ticket sales, not on movie-studio profits or writer’s fees. it is possible that studio profits and writer fees have actually been greater recently for book-based movies than for those based on original screenplays.

another problem with the ad is that it assumes a writer must make an either-or choice from the outset between writing books and writing screenplays. the argument fails to rule out the possibility that a writer engage in both types of writing as well as other types. in fact a writer may be more successful by doing so. writing in various genres might improve one’s effectiveness in each of them. also, writing a book may be an effective first step to producing a screenplay. in any event, the ad provides no justification for the mutually exclusive choice it imposes on the writer.

a third problem with the ad is its ambiguous use of the word “successful.” the argument simply equates success with movie ticket sales. however, many writers may define writing success in other terms, such as intellectual or artistic fulfillment. the ad’s advice that writing screenplays is the best way to achieve writing success ignores other definitions of success.

in conclusion, this quick pitch for a book is based on simplistic assumptions about ticket sales and writer fees, and on an overly narrow definition of success in writing. to better evaluate this argument, at the very least we would need to know the number of years the cited statistic was based on, and the extent to which ticket sales reflect movie studio profits and writer fees.

88. the following appeared in a memorandum from the electrowares company’s marketing department.“since our company started manufacturing and marketing a deluxe light bulb six months ago, sales of our economy light bulb—and company profits—have decreased significantly. although the deluxe light bulb sells for 50 percent more than the economy bulb, it lasts twice as long. therefore, to increase repeat sales and maximize profits, we should discontinue the deluxe light bulb.”

discuss how well reasoned . . . etc.

in this memorandum electroware’s marketing department reasons that manufacturing and marketing of the company’s deluxe light bulb should be discontinued. the primary factors that influence their decision are a significant decrease in sales of the company’s economy light bulb as well as declining company profits in the six month period following the introduction of the deluxe bulb. presumably, their line of reasoning is that the introduction of the deluxe bulb is responsible for both of these undesirable outcomes. unfortunately, the marketing department’s rationale is problematic for several reasons.

in the first place, the marketing department has engaged in “after this, therefore because of this” reasoning. the only reason offered for the belief that the introduction of the deluxe bulb is responsible for both the decline in sales of the economy bulb and the decline in company profits is the fact that the former preceded the latter. no additional evidence linking these events is provided, thus leaving open the possibility that the event are not causally related but merely correlated. this in turn leaves open the possibility that factors other than the one cited are responsible for the decline in sales of the economy bulb and the decline in company profits.

in the second place, it is not clear in the memorandum exactly how the decline in sales of the economy bulb is related to the decline in company profits. one possibility is that the decline in profits is a direct consequence of the decline in sales of the economy bulb. another is that some other factor such as ineffective marketing of the deluxe bulb or the start-up costs associated with the introduction of the deluxe bulb is responsible for the decline in company profits. until the relationship between the events in question is fully understood it would be folly to act upon the marketing department’s recommendation.

in conclusion, the marketing department has failed to articulate reasons that are sufficient to justify its recommendation. specifically, the department has failed to establish a causal link between the introduction of the deluxe bulb and the declines in sales of the economy bulb and company profits. while the introduction of the deluxe bulb may have been a contributing factor in these declines, to strengthen the marketing department’s position various other factors must be examined and ruled out as possible causes of the company’s misfortune.

89. the following is taken from an editorial in a local newspaper.“over the past decade, the price per pound of citrus fruit has increased substantially. eleven years ago, megamart charged 5 cents apiece for lemons, but today it commonly charges over 30 cents apiece. in only one of these last eleven years was the weather unfavorable for growing citrus crops. evidently, then, citrus growers have been responsible for the excessive increase in the price of citrus fruit, and strict pricing regulations are needed to prevent them from continuing to inflate prices.”

discuss how well reasoned . . . etc.

in this editorial the author argues for the imposition of strict pricing regulations in order to prevent citrus growers from continued inflation of prices of citrus fruit. the need for such regulation is supported by the author’s contention that citrus growers have been unnecessarily raising prices of citrus fruit in the past. the evidence for this allegation is the fact that the price of lemons at megamart has increased from 15 cents per pound to over a dollar a pound during the preceding 11-year period. the author maintains that this increase is unjustifiable because weather conditions have been favorable to citrus production in all but one of those years. this argument is flawed for several reasons.

first and foremost, the author assumes that the only factor that influences the price of citrus fruit is the weather. other factors such as monetary inflation, increased distribution and labor costs, or alterations in supply and demand conditions are completely ignored as possible sources for the increase. the charge that citrus growers have unnecessarily raised prices can be sustained only if these and other possible factors can be completely ruled out as contributing to the price increases. since the author fails to address these factors, the recommendation calling for strict pricing regulations can be dismissed out of hand (adv. 無法控制, 脫手, 告終, 立即) as frivolous.

second, the author assumes that the only way to combat increased prices is through government intervention. in a free enterprise system many other means of affecting the pricing of goods are available. for example, boycotting a product and thereby influencing supply and demand conditions of the commodity is an effective means of influencing the price of the product. in a free market economy the call for price regulation by the government should occur only when all other means to rectify the problem have been exhausted.

in conclusion, the author’s argument is unconvincing. to strengthen the argument it would be necessary to show that the only factor influencing the price increases is the growers’ desire for increased profits.

90. the following appeared as part of an article in a local newspaper.“over the past three years the tartfish industry has changed markedly: fishing technology has improved significantly, and the demand for tartfish has grown in both domestic and foreign markets. as this trend continues, the tartfish industry on shrimp island can expect to experience the same over-fishing problems that are already occurring with mainland fishing industries: without restrictions on fishing, fishers see no reason to limit their individual catches. as the catches get bigger, the tartfish population will be dangerously depleted while the surplus of tartfish will devalue the catch for fishers. government regulation is the only answer: tartfish-fishing should be allowed only during the three-month summer season, when tartfish reproduce and thus are most numerous, rather than throughout the year.”

discuss how well reasoned . . . etc.

in this argument the author concludes that government regulation of the tartfish industry is the only way to prevent the problems associated with over-fishing that plague other fishing industries. the author’s line of reasoning is that without restrictions fishers see no reason to limit their catches and that this will deplete the tartfish population as well as devalue the catch. this line of reasoning is problematic for several reasons.

first, while government regulation may be one way to address the problem, it is by no means the only way. many industries recognize that it is in their self-interest to carefully manage the natural resources on which the industry depends. for example, the oil industry routinely limits production of oil-related products in order to prevent surpluses and lower prices. no evidence has been presented to establish that the tartfish industry is incapable of addressing and solving the problem of over-fishing without government intervention.

second, the author’s line of reasoning defies common sense. the author’s underling assumption is that fishers are motivated only by greed and that they will increase their catches to maximize their profits without regard to the effects over-fishing will have on their livelihood and lifestyle in the future. this assumption is not supported in the argument. moreover, as a generalization, on its face it appears to be false. while some fishers may be driven only by immediate economic gratification and consequently see no reason to limit their catches, no doubt others will see the threat over-fishing presents to their way of life and will voluntarily limit their catches.

finally, the author offers no evidence that limiting the season for catching tartfish to three months in the summer will solve the over-fishing problem. moreover, this proposal is highly questionable since this period coincides with the reproductive period of the tartfish.

in conclusion, the author has not made a convincing case for government regulation of the tartfish industry. to strengthen the conclusion the author must provide evidence for the assertion that government regulation is the only way to solve the problem. furthermore, the author must provide evidence to support the assumption that immediate economic gratification is the only motive that fishers have in pursuing their livelihood.

91. the following appeared in a proposal from the development office at platonic university.“because platonic university has had difficulty in meeting its expenses over the past three years, we need to find new ways to increase revenues. we should consider following the example of greene university, which recently renamed itself after a donor who gave it $100 million. if platonic university were to advertise to its alumni and other wealthy people that it will rename either individual buildings or the entire university itself after the donors who give the most money, the amount of donations would undoubtedly increase.”

discuss how well reasoned . . . etc.

in this argument an analogy is drawn between platonic university and greene university. the author argues that to solve its economic problems, platonic university should follow the example of greene university, which was recently named after a wealthy donor, and offer to rename individual buildings or the university itself in exchange for donations. the author believes that since this tactic worked for greene it would undoubtedly work for platonic, and thus provide the much-needed revenue. this argument is questionable for several reasons.

in the first place, the argument rests upon the assumption that a revenue-producing strategy that works for one university will work for another as well. however, greene and platonic may not be sufficiently similar to warrant this assumption. for example, a small, rural university is less likely to have alumni who could afford to make significant donations than a large, urban university. lacking specific information about the makeup of the universities, and their alumni, it is impossible to assess the likelihood that the strategy employed by greene will work for (原稿空缺)

92. the following appeared as part of an article in the business section of a local newspaper.“hippocrene plumbing supply recently opened a wholesale outlet in the location once occupied by the cumquat cafe. hippocrene has apparently been quite successful there because it is planning to open a large outlet in a nearby city. but the cumquat cafe, one year after moving to its new location, has seen its volume of business drop somewhat from the previous year’s. clearly, the former site is a better business location, and the cumquat cafe has made a mistake in moving to its new address.”

discuss how well reasoned . . . etc.

(原稿空缺)business is obviously unsuitable to the location. on the other hand, a bank in the same location might be extremely successful simply because of its suitability to the location.

in the third place, the author’s claim that hippocrene has been successful at cumquat’s previous location is unwarranted. the fact that hippocrene intends to open a new outlet is insufficient to establish this claim. it is possible that the plan to open a new outlet was prompted by a lack of business at the cumquat location.

finally, the author unfairly assumes that one year’s time at the new location is adequate to conclude whether cumquat made a mistake in moving to that location. its is entirely possible that given more time, perhaps another year or so, cumquat will become profitable at the location. common sense informs me that this is a distinct possibility, since it often takes more than one year for a restaurant to establish a customer base at a given location.

in conclusion, the author’s argument is unconvincing. to strengthen the conclusion the author would have to evaluate other possible causes of the performance of the businesses and eliminate all except location as the cause in each case. additionally, it would be necessary to show that location rather than suitability to a location was the cause of the success of hippocrene and the failure of cumquat.

93. the following appeared in a memorandum from the manager of kmtv, a television station.“applications for advertising spots on kmtv, our local cable television channel, decreased last year. meanwhile a neighboring town’s local channel, koop, changed its focus to farming issues and reported an increase in advertising applications for the year. to increase applications for advertising spots, kmtv should focus its programming on farming issues as well.”

discuss how well reasoned . . . etc.

in this editorial kmtv, a local cable television channel is urged to change its programming focus to farming issues in order to increase advertising revenues. the author’s line of reasoning is that koop’s change in focus was the cause of its increase in advertising and that since this tactic worked for koop it will work for kmtv as well. this line of reasoning is flawed in three important respects.

to begin with, the belief that the change in focus to farming issues was the cause of koop’s increase in advertising applications is unfounded. the only evidence offered to support this belief is that the change in focus preceded the increase in applications. unfortunately, this evidence is insufficient to establish the causal claim in question. consequently, it is possible that koop’s change in focus may not have been related to its increase in revenue in the manner required by the author’s argument.

in addition, the author assumes that the towns that kmtv and koop serve are sufficiently similar to warrant a conclusion based on an analogy between them. even if we accept the view that koop’s change in programming focus to farming issues was responsible for its increase in advertising applications, differences between the towns could drastically alter the outcome for kmtv. for example, if kmtv serves a metropolitan area with little interest in agriculture, changing its programming focus to farming issues would most likely be disastrous. lacking information about the towns koop and kmtv serve it is difficult to assess the author’s recommendation.

finally, the author assumes that kmtv’s decrease in applications for advertising was due to its programming. however, since the author provides no evidence to support this assumption, it may be that the decrease was caused by other factors, such as recession in the local economy or transmission problems at the station. without ruling out these and other possible causes the author cannot confidently conclude that kmtv’s programming was responsible for the decrease in advertising applications at hat station.

in conclusion, the author’s argument is unconvincing. to strengthen the argument the author would have to provide additional evidence for the claim that koop’s change in focus was responsible for its increase in advertising applications and that kmtv’s decrease in applications was due to its programming. furthermore, it would be necessary to show that the towns that koop and kmtv serve are sufficiently similar to justify the analogy between them.

94. the following appeared as part of an article in a computer magazine.“a year ago apex manufacturing bought its managers computers for their homes and paid for telephone connections so that they could access apex computers and data files from home after normal business hours. since last year, productivity at apex has increased by 15 percent. other companies can learn from the success at apex: given home computers and access to company resources, employees will work additional hours at home and thereby increase company profits.”

discuss how well reasoned . . . etc.

in this article the author attributes apex manufacturing’s 15 percent increase in productivity over the past year to its decision to equip its manager with computers and paid telephone connections for their homes so that they would access company computers and files from home after normal business hours. on the basis of apex’s experience the author recommends that other companies follow apex’s example and provide computers and access to company resources to their employees. the author believes that such a policy would increase productivity and profits for other companies, just as it did for apex. the author’s line of reasoning is questionable for several reasons.

first, the author assumes that apex’s increase in productivity is due to its equipping its managers with home computers and access to company resources. however, the only evidence offered in support of this claim is the fact that apex’s increase in productivity occurred after the home computers and after-hours (adv. 工作完畢後) access was provided. unfortunately, this evidence is insufficient to establish the causal claim in question. while temporal precedence is one of the conditions required to establish a causal relationship between two events, by itself it is not a sufficient condition. consequently, it is possible that apex’s increase in productivity is not related to its decision to equip its managers with computers and after-hours access in the fashion required by the author’s argument.

second, the author assumes that apex and other companies are sufficiently similar to warrant a conclusion based on an analogy between them. even if we accept the view that apex’s increase in productivity was brought about by its policy of enabling its managers to work from home, differences between apex and other companies could nullify this result. lacking detailed information about apex and the other companies in question it is difficult to assess the author’s conclusion.

in conclusion, the author’s argument is unconvincing. to strengthen the argument the author would have to provide additional evidence for the claim that apex’s decision to provide its managers with home computers and access to company resources was responsible for its increase in productivity. furthermore, it would be necessary to show that apex and other companies are sufficiently similar to justify the analogy between them.

95. the following was excerpted from an article in a farming trade publication.“farmers who switched from synthetic to organic farming last year have seen their crop yields decline. many of these farmers feel that it would be too expensive to resume synthetic farming at this point, given the money that they invested in organic farming supplies and equipment. but their investments will be relatively minor compared to the losses from continued lower crop yields. organic farmers should switch to synthetic farming rather than persist in an unwise course. and the choice to farm organically is financially unwise, given that it was motivated by environmental rather than economic concerns.”

discuss how well reasoned . . . etc.

in this article the author recommends that farmers who switched from synthetic to organic farming last year should switch back to synthetic farming as soon as possible. citing a decline in crop yields as the primary reason for the recommendation to reverse course, the author predicts that crop yields for organic farmers will continue to be lower unless synthetic farming is resumed. moreover, the author argues that organic farming is not a financially viable choice for farmers in any case because it is motivated by environmental, not economic, concerns. the author’s position is unconvincing for several reasons.

first, there is no evidence that the first-year yields of farmers who switched to organic farming are representative of their future yields. common sense would lead one to expect that first-year yields would be lower simple due to the inexperience of farmers accustomed to synthetic farming methods. moreover, other factors such as weather or infertile seed stock could be responsible for the lower yields. since the author does not address these or other factors that could account for the lower yields, his prediction that yields will continue to be lower unless a switch is made back to synthetic farming is not well founded.

second, the author assumes that economic and environmental concerns are mutually exclusive and that only enterprises motivated by economic concerns are financially rewarding. these assumptions are not supported in the argument. moreover, there are good reasons to suspect they may be false in the case at hand. for example, while it may be true that synthetic farming methods produce significantly higher yields in the short term, it may also be the case that they fail to sustain this yield in the long term, whereas the opposite is true for organic methods. if this were the case, the financial advantage of synthetic over organic method would be illusory.

in conclusion, the author’s prediction that yields will continue to lower for farmers who adopt organic farming methods is not well reasoned. to strengthen this forecast it would be necessary to examine and eliminate other possible factors that could account for the lower yields experienced. lacking a full examination of these factors, it is difficult to accept the author’s position. finally, the author’s view that organic farming is financially unwise is completely unsupported.

96. the following appeared in a letter to prospective students from the admissions office at plateau college.“every person who earned an advanced degree in science or engineering from olympus university last year received numerous offers of excellent jobs. typically, many of the plateau college graduates who want to pursue an advanced degree have gone on to olympus. therefore, enrolling as an undergraduate at plateau college is a wise choice for students who wish to ensure success in their careers.”

discuss how well reasoned . . . etc.

in this letter the plateau college admissions office advises students wishing to ensure success in their careers to enroll at plateau. in support of this advice the admissions office argues that many plateau graduates have pursued advanced degrees at olympus university, and that all students who earned advanced degrees in science or engineering from olympus received numerous offers of excellent jobs. this argument is unconvincing for several reasons.

to begin with, the argument depends upon the assumption that the plateau graduates who have pursued advanced degrees at olympus university did so in science or engineering. given this it is reasonable to conclude that for these students enrolling at plateau was a wise choice. however, for students majoring in disciplines other than science or engineering it is inconclusive whether enrolling at plateau is a good idea or not. for example, it may be the case that for students majoring in philosophy, english, or history, plateau is a poor choice because it has a mediocre program in these areas or because few of its graduates in these disciplines are admitted to olympus.

next, the author has failed to indicate how many plateau graduates who pursued advanced degrees at olympus actually received them. this information is critical to determining whether plateau is a wise choice for students planning to attend olympus. for example, if it turns out that only a small fraction of plateau graduates who attend olympus earn advanced degrees, the advice to attend plateau would be highly dubious.

finally, the fact that all students who earned advanced degrees in science or engineering from olympus university last year received numerous offers of excellent jobs is insufficient to warrant the claim that this pattern will continue in the future. lacking evidence to the contrary, it may be the case that this phenomenon was unique, and was the result of factors other than the fact that these students attended olympus; for example perhaps a major employer moved into the area or the overall economy was unusually healthy.

in sum, the reasons for enrolling at plateau offered by the admissions office are inconclusive. to strengthen the argument it would be necessary to show that plateau graduates in fields other than science and engineering might also benefit from attending olympus. furthermore, to properly evaluate the argument, information regarding the percentage of plateau graduates who received advanced degrees from olympus would be required.

97. the following appeared in a memorandum sent by a vice-president of the nadir company to the company’s human resources department.“nadir does not need to adopt the costly ‘family-friendly’ programs that have been proposed, such as part-time work, work at home, and job-sharing. when these programs were made available at the summit company, the leader in its industry, only a small percentage of employees participated in them. rather than adversely affecting our profitability by offering these programs, we should concentrate on offering extensive training that will enable employees to increase their productivity.”

discuss how well reasoned . . . etc.

in this memorandum the vice president of nadir company recommends against the adoption of “family-friendly” program. the author’s line of reasoning is that family-friendly programs such as part-time work, work-at-home and job-sharing need not be adopted because nadir’s employees will not widely participate in them. the vice president’s recommendation is unconvincing for several reasons.

in the first place, the fact that only a small percentage of summit company’s employees participated in these programs when they were offered is scant evidence that nadir’s employees will do likewise. to warrant this inference the author must assume that summit is representative of other companies such as nadir. unfortunately, the author has failed to provide evidence for this crucial assumption. for example, if summit is an emerging high-tech company whose employees are young and unmarried whereas nadir is an established low-tech company whose employees are middle-aged and married we can expect that the percentage of employees who desire to participate in family-friendly programs would be considerably different. lacking specific information about the companies in question it is difficult to give much credence to the vice president’s position.

in the second place, the vice president has failed to make a case for the contention that the adoption of family-friendly programs will adversely affect nadir’s profitability. on the face of it none of the programs mentioned require capital outlay for new equipment or additional office space. unless the vice president assumes that employees who participate in such programs are less productive than their full-time counterparts it is difficult to comprehend the line of reasoning that leads to this view.

finally, the vice president assumes that “family-friendly” programs will not increase nadir’s productivity. lacking evidence to the contrary, there is little motivation to accept this assumption as true. in fact, common sense suggests that part-time workers and job-sharers would be as productive as, or perhaps more productive than, full-time workers.

in conclusion, the vice president’s recommendation against adopting family-friendly programs is not convincing. to strengthen the conclusion it must be shown that summit is representative of other companies such as nadir. additionally, evidence would have to be provided for the assumption that employees who participate in family-friendly programs are less productive than other employees.

98. the following appeared as part of an article in a trade magazine for breweries.“magic hat brewery recently released the results of a survey of visitors to its tasting room last year. magic hat reports that the majority of visitors asked to taste its low-calorie beers. to boost sales, other small breweries should brew low-calorie beers as well.”

discuss how well reasoned . . . etc.

in this article small breweries are urged to brew low-calorie beers in order to boost sales. in support of this recommendation the author cites a survey conducted at magic hat brewery revealing that a majority of visitors to its tasting room asked to taste its low-calorie beers. presumably, the author’s line of reasoning is that since the survey conducted at magic hat shows a high level of interest in low-calorie beers, other breweries would be wise to brew low-calorie beers as well. the author’s argument is problematic for several reasons.

to begin with, the validity of the survey conducted at magic hat brewery is doubtful. lacking information about the number of visitors surveyed and the number of respondents, it is impossible to assess the results. for example, if 100 visitors were surveyed but only 20 responded, the majority who asked to taste low-calorie beers could be as few as 11 of the 100 visitors. obviously, such result would provide little evidence for the author’s recommendation. because the author offers no evidence that would rule out interpretations such as this, the survey results as stated are insufficient to support the recommendation.

next, even if the results of the survey accurately reflect a high level of interest in low-calorie beers among magic hat’s visitors, this may not be true for other breweries. while the survey is suggestive of a widespread interest in low-calorie beers, it is insufficient to establish this general claim because there is no reason to believe that magic hat brewery is representative of other small breweries. for example, if magic hat specialized in low-calorie beers, the results of the survey would be highly questionable when applied to small breweries in general. once again, because the author offers no evidence that would rule out interpretations such as this, the survey results as stated are insufficient to support the recommendation.

finally, since the author’s recommendation is aimed at boosting breweries’ sales, it must be shown that visitor interest in tasting low-calorie beers resulted in sales of these beers. no evidence is offered in the argument to support this crucial connection. thus the author’s recommendation cannot be taken seriously.

in conclusion, the survey conducted at magic hat brewery offers little support for the author’s recommendation. to strengthen the conclusion the author would have to provide detailed information about the survey that demonstrates its validity. moreover, it would be necessary to show that magic hat brewery was representative of other small breweries and that visitor interest in tasting low-calorie beers resulted in sales of these beers.

99. the following appeared in an editorial from a newspaper serving the town of saluda.“the saluda consolidated high school offers over 200 different courses from which its students can choose. a much smaller private school down the street offers a basic curriculum of only 80 different courses, but it consistently sends a higher proportion of its graduating seniors on to college than consolidated does. by eliminating at least half of the courses offered there and focusing on a basic curriculum, we could improve student performance at consolidated and also save many tax dollars.”

discuss how well reasoned . . . etc.

in this editorial the author recommends that saluda’s consolidated high school eliminate half of its 200 courses and focus primarily on basic curriculum in order to improve student performance and save tax revenues. the author’s recommendation is problematic for several reasons.

to begin with, the author assumes that the only relevant difference between consolidated and the private school is the number of courses offered by each. however, other relevant differences between the schools might account for the difference in the proportion of their graduates who go on to college. for example, the private school’s students might be selected from a pool of gifted or exceptional students, or might have to meet rigorous admission standards whereas consolidated’s students might be drawn from the community at large with little or no qualification for admission.

next, the author assumes that the proportion of students who go on to college is an overall measure of student performance. while this is a tempting assumption, its truth is by no means obvious. if student excellence is narrowly defined in terms of the student’s ability to gain access to college, this assumption is somewhat reasonable. however, given a broader conception of student excellence that takes into account student’s ability to learn and apply their knowledge to new situations, its is not obvious that college admission is reliable indicator of performance. for example, students in non-academic disciplines could conceivably perform at high levels within these disciplines but nevertheless be unable to meet college admission standards.

finally, the author assumes that savings in tax revenues will result from the reduced costs of funding the paired-down curriculum. this is not necessarily true. for example, it could turn out that both programs serve the same number of students and require the same number of classrooms and teacher.

in conclusion, the author has not made a convincing case for the recommendation to eliminate courses at consolidated and focus on a basic curriculum. to strengthen the conclusion the author would have to provide evidence that consolidated and the private school were sufficiently similar to warrant the analogy between them. moreover, the relationship between student performance and college admission and the mechanism whereby savings in tax revenues would be accomplished would have to be clarified.

100. the following appeared as part of an article in the book section of a newspaper.“currently more and more books are becoming available in electronic form — either free-of-charge on the internet or for a very low price-per-book on compact disc *. thus literary classics are likely to be read more widely than ever before. people who couldn’t have purchased these works at bookstore prices will now be able to read them for little or no money; similarly, people who find it inconvenient to visit libraries and wait for books to be returned by other patrons will now have access to whatever classic they choose from their home or work computers. this increase in access to literary classics will radically affect the public taste in reading, creating a far more sophisticated and learned reading audience than has ever existed before.”

*a “compact disc” is a small portable disc capable of storing relatively large amounts of data that can be read by a computer.

discuss how well reasoned . . . etc.

in this article the author concludes that literary classics are likely to be read more widely than ever before. the author’s line of reasoning is that the availability of books in electronic form and access of books via the internet has removed the two major impediments that prevented people from reading literary classics, namely price and convenient access. since books can be accessed from home or work via computers at little or no cost, the author believes that significant changes in the society will occur. specifically, the author maintains that access to literary classics will affect the public’s taste in reading and will result in a more learned and cultured reading audience. the author’s argument is unconvincing for several reasons.

first, the author assumes that price and convenient access are the primary reasons people fail to read literary classics. while this is a tempting assumption, it is not obviously true. for example, other reasons, such as lack of interest in these books or awareness of them on the part of the reading public could equally account for the failure to read them. consequently, it may turn out that, contrary to the author’s expectation, the number of people who read literary classics is unaffected by their increased availability and lower cost.

second, while it may be the case that access to books at affordable prices has increased as a result of new technology, the author provides no evidence for the assumption that access to literary classics at affordable prices has increased as well. on the face of it, this assumption seems innocuous; however there may be reasons that prevent literary classics from being marketed in the fashion described by the author. for example, the inability to secure the requisite permissions to reproduce these books in electronic form, or the lack of commercial interest in marketing them via the internet could undermine the author’s assumption.

in conclusion, this argument is not convincing. to strengthen the argument the author would have to provide evidence for the assumption that price and accessibility are the main reasons people fail to read literary classics. additionally, evidence would be required for the assumption that access to literary classics will be increased.

101. the following appeared as an editorial in a magazine concerned with educational issues.“in our country, the real earnings of men who have only a high-school degree have decreased significantly over the past fifteen years, but those of male college graduates have remained about the same. therefore, the key to improving the earnings of the next generation of workers is to send all students to college. our country’s most important educational goal, then, should be to establish enough colleges and universities to accommodate all high school graduates.”

discuss how well reasoned . . . etc.

this editorial advocates universal college education as a means of improving the earnings of all the next generation of workers. in support of this recommendation the author points out that the real earnings of male high-school graduates have decreased over the past fifteen years whereas the earnings of male college graduates have remained stable over the same period. furthermore, the author argues that a sufficient number of colleges and universities should be built to accomplish this goal. the author’s position is implausible for a number of reasons.

in the first place, the evidence cited by the author pertains only to male high-school and college graduates. no comparable comparison of the earnings of female workers is made, yet the author recommends sending all students to college. if it turns out that no discrepancy between the real earnings of female high-school graduates and female college graduates exists during this same period, the author’s conclusion would be significantly weakened.

in the second place, the author assumes that the primary factor that influences the earnings of workers is their level of education. while this is a reasonable assumption, it is by no means a certainty. for example, in countries undergoing political turmoil and reform, the educated class of citizens is often discriminated against and cannot find work. in such cases, lack of education might turn out to be a distinct economic advantage.

finally, a comparison of workers’ earnings during a 15-year period is insufficient evidence to warrant the author’s recommendation. other factors besides worker’s level of education could account for the discrepancy in earnings during the period cited by the author. for example, the demand for college-educated workers may have outpaced the demand for high-school educated workers during the period in question and as a result increased their earnings disproportionately.

in conclusion, the author’s argument is unconvincing. to strengthen the conclusion the author would have to provide information about the earnings of female workers that showed a trend comparable to the one cited for male workers. additionally, evidence would be required for the assumption that level of education is the primary factor that influences worker’s earnings.

102. the following appeared as part of a business plan created by the management of the take heart fitness center.“after opening the new swimming pool early last summer, take heart saw a 12 percent increase in the use of the center by members. therefore, in order to increase the number of our members and thus our revenues, which depend on membership fees, we should continue to add new recreational facilities in subsequent years: for example, a multipurpose game room, a tennis court, and a miniature golf course. being the only center in the area offering this range of activities would give us a competitive advantage in the health and recreation market.”

discuss how well reasoned . . . etc.

because take heart fitness center experienced a 12 percent increase in member usage as a result of opening a new swimming pool last summer, the author recommends the addition of new recreational facilities in subsequent years as a means of increasing membership in take heart. the author’s recommendation is problematic for several reasons.

first, and foremost, the author assumes that an increase in member usage portends an increase in membership. this assumption may hold true in some cases. however, it is unlikely to hold true in the case at hand, because it is reasonable to expect that members would visit the fitness center to inspect and try out the new swimming pool. this would account for the increase in usage. however, since the author provides no evidence that this new rate of usage was sustained, the abrupt increase in usage provides little evidence that the addition of facilities such as the pool will attract new members.

second, the author assumes that the addition of the swimming pool was responsible for the increase in member usage. however, the only evidence for this claim is insufficient to establish the causal claim in question. while temporal precedence is one of the conditions required to establish a causal relationship between two events, by itself it is not a sufficient condition. consequently, it is possible that the addition of the pool was unrelated to the increase in usage in the manner required by the author’s argument.

finally, the author has provided no evidence to support the contention that take heart will be the only center in the area to offer a wide range of activities to its members and thus have a competitive advantage in the fitness market.

in conclusion, the author’s belief that adding additional recreational facilities will increase take heart’s membership is ill-founded. to strengthen the argument the author would have to provide evidence that member usage is reliable indicator of new membership. additionally, it would be necessary to show that the cause of the increase in usage was the opening of the new pool.

103. the following appeared in a letter from a staff member in the office of admissions at argent university.“the most recent nationwide surveys show that undergraduates choose their major field primarily based on their perception of job prospects in that field. at our university, economics is now the most popular major, so students must perceive this field as having the best job prospects. therefore, we can increase our enrollment if we focus our advertising and recruiting on publicizing the accomplishments of our best-known economics professors and the success of our economics graduates in finding employment.”

discuss how well reasoned . . . etc.

as a means of increasing enrollment at argent university, the office of admissions recommends publicizing the accomplishments of its economics professors and the success of its economics graduates in finding jobs. this recommendation is questionable for a number of reasons.

first, the author assumes that students will continue to perceive economics favorably as a source of employment. this assumption, however, is not supported in the argument. the fact that economics is currently the most popular major at argent establishes only that economics was perceived by current students as the occupation having the best job prospects when they enrolled; it does not establish that incoming students will perceive this field in the same way.

second, since the argument relies entirely upon nationwide surveys that establish a relation between student perception of job prospects and choice of major, information about the manner in which these surveys were conducted would be necessary to properly evaluate the conclusion. specifically, it would be necessary to show that the students who participated in the surveys were representative of students in general and that a sufficient number were included in the surveys to warrant the claim that choice of major is dictated by student perception of prospective employment. without knowing how the surveys were conducted, it is impossible to determine whether they are reliable, and consequently whether the conclusion based on them is sound.

finally, even if we accept the survey results it might be the case that economics is the most popular major at argent for reasons other than students’ perception of job prospects. for example, perhaps economics is favored because it receives more funding and can offer a wider range of courses than other majors, or because more scholarships are available to economics majors than to others.

in sum, the plan to increase enrollment at argent is seriously flawed. to strengthen the proposal it would be necessary to show that economics is currently viewed by students as having the best job prospects. additionally, information validating the reliability of the survey as well as evidence that argent’s students chose economics because of the job prospects would be required.

104. the following appeared as part of a memorandum from the loan department of the frostbite national bank.“we should not approve the business loan application of the local group that wants to open a franchise outlet for the kool kone chain of ice cream parlors. frostbite is known for its cold winters, and cold weather can mean slow ice cream sales. for example, even though frostbite is a town of 10,000 people, it has only one ice cream spot — the frigid cow. despite the lack of competition, the frigid cow’s net revenues fell by 10 percent last winter.”

discuss how well reasoned . . . etc.

in this memorandum the loan department of frostbite national bank recommends against approval of a business loan to a local group that wants to open an ice cream parlor. in support of this decision the loan department points out that frostbite has a reputation for cold winters and sales of ice cream decrease in cold weather. this latter point is buttressed by the fact that frostbite’s only ice cream parlor suffered a 10 percent decline in net revenues the previous winter. the loan department’s decision is questionable for the following reasons.

to begin with, since it is reasonable to expect a decline in ice cream sales during winter months, it is difficult to assess the relevance of the fact that frostbite has cold winters to the potential success of the kool kone franchise. common sense suggests that this fact would be significant only if it turned out that frostbite’s winter season lasted 9 or 10 months as it does in arctic regions. in that case slow sales could be expected for most of the year and the loan department’s opposition to the loan would be readily understandable. if, on the other hand, frostbite’s winter season lasts only a few months and the remainder of the year is warm or hot, it is difficult to comprehend the loan department’s reasoning.

next, the loan department assumes that the frigid cow’s decline in net revenue last winter was a result of slow sales occasioned by cold weather. while this is a possible reason for the decline, it is not the only factor that could account for it. for example, other factors such as poor business practices or lack of inventory could be responsible for the frigid cow’s loss of revenue. the loan department’s failure to investigate or even consider these and other possible explanations for the frigid cow’s decline in revenue renders their decision highly suspect.

in conclusion, the loan department’s decision is ill-founded. to better evaluate the decision, we would need to know more about the length and severity of frostbite’s winter season. moreover, evidence would have to be provided to support the assumption that the frigid cow’s loss of revenue last winter was a direct result of the cold weather.

105. the following appeared as part of a letter to the editor of a local newspaper.“bayview high school is considering whether to require all of its students to wear uniforms while at school. students attending acorn valley academy, a private school in town, earn higher grades on average than bayview students and are more likely to go on to college. moreover, acorn valley reports few instances of tardiness, absenteeism, or discipline problems. since acorn valley requires its students to wear uniforms, bayview high school would do well to follow suit and require its students to wear uniforms as well.”

discuss how well reasoned . . . etc.

in this letter to the editor the author argues that bayview high school should follow the example of acorn valley academy and require its students to wear uniforms to school. in support of this recommendation the author points to acorn’s low rate of absenteeism and tardiness as well as its lack of discipline problems and superior student performance. the author’s recommendation is questionable for a number of reasons.

to begin with, the author assumes that all of the stated benefits are a result of acorn’s requirement that its students wear uniforms. on the face of it this appears to be simplistic assumption. it defies common sense to believe, as the author must, that the primary reason acorn’s students receive higher grades on average and are more likely to go on to college is that they are required to wear uniforms to school. similarly, the author’s belief that acorn’s low rate of tardiness, absenteeism, and discipline problems can be attributed directly to its dress code is not in accord with common sense.

next, the author assumes that the only relevant difference between bayview and acorn is the wearing of school uniforms. this assumption is not supported in the argument. moreover, if it turns out that acorn’s students are gifted and highly motivated to learn whereas bayview’s are unexceptional and lack motivation to learn, common sense indicates that acorn’s students would be more likely to perform better and cause fewer problems than bayview’s.

finally, it is unclear whether bayview suffers from any of the problems the author wishes to correct by mandating its students to wear uniforms. for example, the author states that acorn’s students earn higher grades on average and are more likely to go on to college, but it is unclear whether this is a comparison to bayview’s students or to some other group. lacking assurance that bayview is deficient in the categories mentioned in the letter, it is difficult to accept the author’s recommendation.

in conclusion, the author has failed to provide compelling reasons for the recommendation that bayview’s students be required to wear uniforms. to strengthen the argument the author would have to provide evidence for the assumption that acorn’s requirement that students wear uniform is responsible for the various benefits mentioned. additionally, it would have to be established that bayview is similar in relevant respects to acorn and suffers from the problems that the author’s remedy is intended to correct.

106. the following appeared in a memo to the saluda town council from the town’s business manager.“research indicates that those who exercise regularly are hospitalized less than half as often as those who don’t exercise. by providing a well-equipped gym for saluda’s municipal employees, we should be able to reduce the cost of our group health insurance coverage by approximately 50% and thereby achieve a balanced town budget.”

discuss how well reasoned . . . etc.

in this memo saluda’s business manager recommends that the town provide a gym for its employees as a means of balancing the town’s budget. the manager reasons that since studies show that people who exercise regularly are hospitalized less than half as often than those who don’t exercise, saluda could save approximately 50% on the cost of its group health insurance coverage by providing its employees with a well-equipped gym. the savings on insurance would balance the town’s budget. the manager’s argument is unconvincing because it rests on several unsupported and dubious assumptions.

first, the manager assumes that saluda’s employees will exercise regularly if a well-equipped facility is provided for them. this assumption is questionable since the mere fact that a gym is made available for employee use is no guarantee that they will avail themselves of it at all, let alone on a regular basis.

second, the manager assumes that saluda’s employees do not exercise regularly. once again, the manager offers no support for this crucial assumption. obviously, if all of sauda’s employees already engage in daily exercise, the hospitalization rate will be unaffected by equipping an exercise facility and no savings will be realized on the group health insurance.

third, the manager assumes that there is a direct relation between the hospitalization rate for employees and the cost of their group health insurance such that a reduction in the hospitalization rate will result in a corresponding reduction in the cost of insurance. while this may turn out to be true, the manager has failed to offer any evidence for this claim.

finally, the manager assumes that the cost of building a well-equipped exercise facility will not negate the savings realized on the group health insurance. until evidence has been provided to show that this is not the case, the manager’s plan is unacceptable.

in conclusion, the business manager’s proposal to provide an exercise facility as a means of balancing saluda’s budget is not convincing. to strengthen the argument, evidence would have to be provided for each of the assumptions listed in the previous analysis.

107. the following appeared in a memorandum written by the assistant manager of a store that sells gourmet food items from various countries.“a local wine store made an interesting discovery last month: it sold more french than italian wine on days when it played recordings of french accordion music, but it sold more italian than french wine on days when italian songs were played. therefore, i recommend that we put food specialties from one particular country on sale for a week at a time and play only music from that country while the sale is going on. by this means we will increase our profits in the same way that the wine store did, and we will be able to predict more precisely what items we should stock at any given time.”

discuss how well reasoned . . . etc.

as a means of increasing profits and more accurately predicting what items should be stocked, the assistant manager of a gourmet food store proposes that only music from a particular country be played during the period food items from that country are on sale. the basis for this proposal is the discovery by a wine store that sales of wine from a country increased when music from that country was played. the manager’s proposal is questionable for several reasons.

first, the manager assumes that the sequence of sales experienced by the wine store reflects a general causal pattern. however, there is little evidence to support this assumption. while perhaps indicative of such a pattern, the wine store sales merely demonstrate a correlation between sales of wine from a country and the playing of music from that country. to establish a causal connection between these events it would be necessary to examine and eliminate other possible factors that might account for this phenomenon. in any case, it is highly questionable whether evidence gathered over a one-month period is sufficient to establish the general claim in question.

second, the manager assumes that the wine store increased its profits by playing the appropriate music. however, this is not necessarily the case. it is consistent with the statement that the wine store sold more french than italian wine on days when french music was played and vice versa when italian music was played that no net increase in sales, and thus profit, was realized by this method. for example, it is possible that on days when french music was played seven bottles of french wine were sold and three bottles of italian (were sold) whereas the reverse was the case when italian music was played, and that in both instances only 10 bottles were sold.

in conclusion, the manager’s recommendation is based on two dubious assumptions. to strengthen the argument it would be necessary to provide additional evidence to support the claim that sales of an item are influenced by the type of music played. additionally, the manager would have to provide evidence that the wine store increased its profits by playing the appropriate music.

108. the following appeared in a memorandum from the director of research and development at ready-to-ware, a software engineering firm.“the package of benefits and incentives that ready-to-ware offers to professional staff is too costly. our quarterly profits have declined since the package was introduced two years ago, at the time of our incorporation. moreover, the package had little positive effect, as we have had only marginal success in recruiting and training high-quality professional staff. to become more profitable again, ready-to-ware should, therefore, offer the reduced benefits package that was in place two years ago and use the savings to fund our current research and development initiatives.”

discuss how well reasoned . . . etc.

in this memorandum the director of research and development of ready-to-ware recommends reducing the benefits package offered to employees as a means of increasing profits and funding current research and development initiatives. the director’s line of reasoning is that quarterly profits have declined because of the current benefits package and can be increased by reducing it. moreover, the director argues that the benefits package had little effect in recruiting and training high-quality employees. the director’s argument is questionable for several reasons.

to begin with, the director’s reasoning is a classic instance of “after this, therefore because of this” reasoning. the only evidence put forward to support the claim that the introduction of the benefits package is responsible for the decline in quarterly profits is that the profits declined after the package was introduced. however, this evidence is insufficient to establish the causal claim in question. many other factors could bring about the same result. for example, the company may have failed to keep pace with competitors in introducing new products or may have failed to satisfy its customers by providing adequate support services. until these and other possible factors are ruled out, it is premature to conclude that the introduction of the benefits package was the cause of the decline in profits.

next, the director assumes that the benefits package currently offered is responsible for the marginal success ready-to-ware has experienced in recruiting and training new high-quality professionals. however, no evidence is offered to support this allegation. other reasons for ready-to-ware’s failure to attract high-quality professionals are not considered. for example, perhaps ready-to-ware is not a cutting edge company or is not regarded as a leader in its field. until these and other possible explanations of the company’s marginal success at recruiting and training employees are examined and eliminated it is folly to conclude that the benefits package provided to the professional staff is responsible.

in conclusion, the director has failed to provide convincing reasons for reducing the benefits package ready-to-ware currently offers its professional staff. to further support the recommendation the director would have to examine and eliminate other possible reasons for the decline in ready-to-ware’s quarterly profits and for its lack of success in attracting high-quality professionals.

109. the following appeared in a memorandum from the vice-president of the dolci candy company.“given the success of our premium and most expensive line of chocolate candies in a recent taste test and the subsequent increase in sales, we should shift our business focus to producing additional lines of premium candy rather than our lesser-priced, ordinary candies. when the current economic boom ends and consumers can no longer buy major luxury items, such as cars, they will still want to indulge in small luxuries, such as expensive candies.”

discuss how well reasoned . . . etc.

in this memorandum the vice president of dolci recommends changing the company’s focus to the production of premium high-priced candy products. in support of this proposal the vice president points to the success of dolci’s expensive line of chocolate candies in recent taste test and the increase in sales following the test. an additional rationale for the change in focus stems from the speculation that consumers will continue to purchase expensive candies when they can no longer afford major luxury items. the vice president’s proposal lacks cogency for three reasons.

first, the fact that the premium line of chocolates met with success in a recent taste test is scant evidence of the claim that this line of candies will continue to be successful in the future. to warrant this inference the vice president must assume that the taste test was representative of consumers’ candy preferences in general. unfortunately, the vice president has failed to provide evidence for this crucial assumption.

second, the vice president assumes that the increase in sales experienced after the taste test was brought about by sales of the premium candies. however, the only indication that this was the case is the fact that the increase in sales followed the taste test. unfortunately, this evidence is insufficient to establish the causal claim in question. while temporal precedence is one of the conditions required to establish a causal relationship between two events, by itself it is not a sufficient condition.

finally, while the vice president’s speculation about future sales of premium candies may turn out to be correct, no evidence has been provided to support this prediction.

in conclusion, the vice president has not made a convincing case for the recommendation to shift to dolci’s business focus. to further support this proposal the vice president would have to provide evidence that the taste test was a reliable indicator of consumer’s candy preferences. moreover, supporting evidence would be required for the prediction that consumers will continue to buy premium candies in the event of an economic downturn.

115題issue範文1. in some countries, television and radio programs are carefully censored for offensive language and behavior. in other countries, there is little or no censorship.in your view, to what extent should government or any other group be able to censor television or radio programs? explain, giving relevant reasons and/or examples to support your position.

the extent to which the broadcast media should be censored for offensive language and behavior involves a conflict between our right of free speech and the duty of the government to protect its citizenry from potential harm. in my view, our societal interest in preventing the harm that exposure to obscenity produces takes precedence over the rights of individuals to broadcast this type of content.

first of all, i believe that exposure to obscene and offensive language and behavior does indeed cause similar behavior on the part of those who are exposed to it. although we may not have conclusive scientific evidence of a cause-effect relationship, ample anecdotal evidence establishes a significant correlation. moreover, both common sense and our experiences with children inform us that people tend to mimic the language and behavior they are exposed to.

secondly, i believe that obscene and offensive behavior is indeed harmful to a society. the harm it produces is, in my view, both palpable and profound. for the individual, it has a debasing impact on vital human relationships; for the society, it promotes a tendency toward immoral and antisocial behavior. both outcomes, in turn, tear apart the social fabric that holds a society together.

those who advocate unbridled individual expression might point out that the right of free speech is intrinsic to a democracy and necessary to its survival. even so, this right is not absolute, nor is it the most critical element. in my assessment, the interests served by restricting obscenity in broadcast media are, on balance, more crucial to the survival of a society. advocates of free expression might also point out difficulties in defining “obscene” or “offensive” language or behavior. but in my view, however difficult it may be to agree on standards, the effort is worthwhile.

in sum, it is in our best interest as a society for the government to censor broadcast media for obscene and offensive language and behavior. exposure to such media content tends to harm society and its citizenry in ways that are worth preventing, even in light of the resulting infringement of our right of free expression.

2. “it is unrealistic to expect individual nations to make, independently, the sacrifices necessary to conserve energy. international leadership and worldwide cooperation are essential if we expect to protect the world’s energy resources for future generations.”discuss the extent to which you agree or disagree with the opinion stated above. support your views with reasons and/or examples from your own experience, observations, or reading.

the speaker asserts that an international effort is needed to preserve the world’s energy resources for future generations. while individual nations, like people, are at times willing to make voluntary sacrifices for the benefit of others, my view is that international coordination is nevertheless necessary in light of the strong propensity of nations to act selfishly, and because the problem is international in scope.

the main reason why an international effort is necessary is that, left to their own devices, individual nations, like people, will act according to their short-term motives and self-interest. the mere existence of military weapons indicates that self-interest and national survival are every nation’s prime drivers. and excessive consumption by industrialized nations of natural resources they know to be finite, when alternatives are at hand demonstrates that self-interest and short-sightedness extend to the use of energy resources as well. furthermore, nations, like people, tend to rationalize their own self-serving policies and actions. emerging nations might argue, for example, that they should be exempt from energy conservation because it is the industrialized nations who can better afford to make sacrifices and who use more resources in the first place.

another reason why an international effort is required is that other problems of an international nature have also required global cooperation. for example, has each nation independently recognized the folly of nuclear weapons proliferation and voluntarily disarmed? no: only by way of an international effort, based largely on coercion of strong leaders against detractors, along with an appeal to self-interest, have we made some progress. by the same token (adv. 出於同樣原因), efforts of individual nations to thwart international drug trafficking have proven largely futile, because efforts have not been internationally based. similarly, the problem of energy conservation transcends national borders in that either all nations must cooperate, or all will ultimately suffer.

in conclusion, nations are made up of individuals who, when left unconstrained, tend to act in their own self-interest and with short-term motives. in light of how we have dealt, or not dealt, with other global problems, it appears that an international effort is needed to ensure the preservation of natural resources for future generations.

3. “corporations and other businesses should try to eliminate the many ranks and salary grades that classify employees according to their experience and expertise. a ‘flat’ organizational structure is more likely to encourage collegiality and cooperation among employees.”discuss the extent to which you agree or disagree with the opinion stated above. support your views with reasons and/or examples from your own experience, observations, or reading.

which is a better way to classify and reward employees of a business: a “flat” organizational structure or a hierarchical structure? the speaker prefers a “flat” structure in which distinctions between employees based on education or experience are not used as a basis for monetary rewards. i strongly disagree with the speaker’s view, for two reasons.

in the first place, the speaker’s preference for a “flat” structure is based upon the claim that cooperation and collegiality (the relationship of colleagues; specifically: the participation of bishops in the government of the roman catholic church in collaboration with the pope) among employees is more likely under this system than under a hierarchical one. however, this claim ignores our everyday experience in human interaction. disagreements among coworkers are inevitable. without a clear authoritative figure to resolve them and to make final decisions, disputes are more likely to go unresolved and even worsen, thereby undermining cooperation, congeniality and, ultimately, productivity and profit.

in the second place, whether or not collegiality and cooperation are best fostered by a flat organizational structure is beside the point (adj. 離題,不中肯,不得要領). my main reason for rejecting an organizational structure that does not distinguish workers in terms of their abilities or experience is that under such a system workers have little incentive to improve their skills, accomplish their work-related goals, or assume responsibility for the completion of their assigned tasks. in my experience, human motivation is such that without enticements such as money, status or recognition, few people would accomplish anything of value (of value: 有價值的) or assume responsibility for any task. a flat system actually might provide a distinct disincentive for productivity and efficiency insofar as workers are not held accountable for the quality or quantity of their work. by ignoring human nature, then, a company may be harming itself by encouraging laziness and complacency.

in sum, the speaker’s opinion that a “flat” organizational structure is the best way to promote collegiality and cooperation among employees runs counter to the common sense about how people act in a work environment, and in any case (in any case: adv.無論如何) provides a feeble rationale for the preference of one organizational structure over another.

4. “of all the manifestations* of power, restraint in the use of that power impresses people most.”* manifestations: apparent signs or indicators

explain what you think this quotation means and discuss the extent to which you agree or disagree with it. develop your position with reasons and/or specific examples drawn from history, current events, or your own experience, observations, or reading.

this quote means essentially that people admire powerful individuals who do not use their power to the utmost (adv. 極度) to achieve their goals but rather use only the minimum amount required to attain them. while this view is admirable in the abstract (adv. 抽象地, 理論上), the statement is inaccurate in that it fails to reflect how people actually behave.

the popularity of “revenge” movies aptly illustrates that many people are not impressed with individuals who use restraint when exercising their power. in these movies the protagonist is typically portrayed as having certain physical abilities that would enable him to easily defeat the various adversaries he encounters. in the initial confrontations with these individuals he typically refrains from using his abilities to defeat them. the audience, however, soon grows tired of this, and when the hero finally loses control and completely demolishes his opponent, they burst into applause. this homey example strongly suggests that many people are more impressed with the use of power than with the restraint of its use.

the gulf war provides another example of a situation where restraint in the use of power was not widely acclaimed. when the allied forces under the command of general schwartzkoff showed restraint by not annihilating the retreating iraqi army, the general was widely criticized by the public for not using the force available to him to eliminate this potential enemy once and for all (adv. 徹底地,一勞永逸地). this example shows once again that often people are not impressed by individuals who exhibit restraint in using their power.

in conclusion, the examples cited above clearly indicate that, contrary to the view expressed in the quote, many—if not most—people are more impressed with individuals who utilize their power to the utmost than with those who exercise restraint in the use of their power.

5. “all groups and organizations should function as teams in which everyone makes decisions and shares responsibilities and duties. giving one person central authority and responsibility for a project or task is not an effective way to get work done.”to what extent do you agree or disagree with the opinion expressed above? support your views with reasons and/or specific examples drawn from your own work or school experiences, your observations, or your reading.

which is a more productive method of performing a group task: allowing all group members to share in the decision making, duties and responsibilities, or appointing one member to make decisions, delegate duties and take responsibility? the speaker’s opinion is that the first method is always the best one. in my view, however, each of these alternatives is viable in certain circumstances, as illustrated by two very different examples.

a jury in a criminal trial is good example of a group in which shared decision-making, duties, and responsibility is the most appropriate and effective way to get the job done. each member of the jury is on equal footing with the others. while one person is appointed to head the jury, his or her function is to act as facilitator, not as leader. to place ultimate authority and responsibility on the facilitator would essentially be to appoint a judge, and to thereby defeat the very purpose of the jury system.

by way of contrast, a trauma unit in a hospital is a case in which one individual should assume responsibility, delegate duties and make decisions. in trauma units, split-second (split-second: adj.瞬間發生的) decisions are inherently part of the daily routine, and it is generally easier for one person to make a quick decision than for a team to agree on how to proceed. one could argue that since decisions in trauma units are typically life-and-death (adj. 生死攸關的, 重大的) ones, leaving these decisions to one person is too risky. however, this argument ignores the crucial point that only the most experienced individuals should be trusted with such a burden and with such power; leaving decisions to inexperienced group members can jeopardize a patient’s very life.

in conclusion, i agree that in some situations the best way to accomplish a task is through teamwork-sharing responsibility, duties and decision making. however, in other situations, especially those where quick decisions are necessary or where individual experience is critical, the most effective means is for one individual to serve as leader and assume ultimate responsibility for completing the job.

6. “there is only one definition of success — to be able to spend your life in your own way.”to what extent do you agree or disagree with this definition of success? support your position by using reasons and examples from your reading, your own experience, or your observation of others.

the speaker here defines success simply as the ability to choose how to spend one’s life. under this definition, people who have the freedom to do whatever they want at any time (at any time: adv.任何時候) they choose would presumably be the most successful ones, while those who have no such freedom would be the biggest failures. viewing the definition in this light reveals three serious problems with it.

the chief problem with this definition of success is that by the definition nearly all people would be regarded as failures. the reason for this is simple. most people have extremely limited choices in what they can do and when they can do it. in other words, unrestricted freedom of choice is a luxury only a few people—perhaps a handful of (adj. 少數,一小撮) tyrannical dictators and ultra-wealthy individuals—can afford.

secondly, people who have a high degree of freedom in choosing their lifestyle often acquire it through means that would not earn them the accolade of being successful. for example, lottery winners or people who inherit a great deal of money may be able to spend their life in any way they choose, but few people would regard them as successful merely due to their financial fortune.

a third reason this definition of success is unacceptable is that it repudiates some of our basic intuitions about success. for most people, success is related to achievement. the more you achieve, the more successful you are; conversely, the less you achieve the less successful you are. defining success in terms of freedom of choice ignores this intuition.

in sum, the proposed definition of success is far too limiting, and it belies our intuition about the concept. i think that most people would agree with me that success is better defined in terms of the attainment of goals.

7. “the best way to give advice to other people is to find out what they want and then advise them how to attain it.”discuss the extent to which you agree or disagree with the opinion expressed above. support your point of view with reasons and/or examples from your own experience, observations, or reading.

some people think that the best way to advise people is simply to find what they want and help them attain it? in my view, this method is generally not the best way to proceed in advising others; it ignores the plain truth that many people do not know what they want and do not know what is best for them.

my main reason for rejecting this technique is that people very rarely have any clear idea of what they want. this applies not only to consumer items such as clothing, cars and luxury items but also to what they want out of life in general. in fact, numerous studies have shown that most people cannot list the ten things they want most out of life, even if given considerable time to think about it.

my second reason for rejecting this method is that more often than not (時常) what people want is not what is best for them. parents continually face this problem when advising their children. for example, suppose a child wants to quit school and get a job. surely, the parents would be derelict in helping their child attain this want instead of convincing the child that continuing education would be in his or her best interest.

admittedly, following the proposed advising method would result in a high rate of compliance, since the person being advised would act consistently with his or her own will by following the advice. however, as noted above, acting according to what one wants is not necessarily desirable. proponents of this method might also point to college counselors as models of this technique. however, college counselors should not necessarily be held up as models for advising people generally, let alone as models for advising students.

in conclusion, i do not agree that the best way to advise people is to find what they desire and help them achieve it. in my estimation the pitfalls of such a technique outweigh any of its potential advantages.

8. “for hundreds of years, the monetary system of most countries has been based on the exchange of metal coins and printed pieces of paper. however, because of recent developments in technology, the international community should consider replacing the entire system of coins and paper with a system of electronic accounts of credits and debits.”discuss the extent to which you agree or disagree with the opinion stated above. support your views with reasons and/or examples from your own experience, observations, or reading.

the prospect of converting the world’s monetary system of metal coins and printed paper into a computerized system of credits and debits is intriguing. opponents of the idea regard a digital economy as a dangerous step toward a totalitarian society in which an elite class dominates an information-starved lower class. my view, however, is that conversion to a digital economy has far-reaching economic and social virtues that outweigh the potential risk of misuse by a political elite.

supporters of the idea of “digital cash” view the move to a digital economy as the next logical step toward a global system of free trade and competition. herein (adv. 於此, 在這裡) lies the main virtue of a digital economy. in facilitating trade among nations, consumers worldwide would enjoy a broader range of goods at more competitive prices.

in addition, a digital economy would afford customers added convenience, while at the same time saving money for businesses. making purchases with electronic currency would be simple, fast, and secure. there would be no need to carry cash and no need for cashiers to collect it. a good example of the convenience and savings afforded by such a system is the “pay and go” gasoline pump used at many service stations today. using these pumps saves time for the customer and saves money for the business.

a third benefit of such a system is its potential to eliminate illegal monetary transactions. traffickers of illegal arms and drugs, dealers in black-market contraband, and counterfeiters all rely on tangible currency to conduct their activities. by eliminating hard currency, illegal transactions such as these would be much easier to track and record. as a result, illegal monetary transactions could be virtually eliminated. a related benefit would be the ability to thwart tax evasion by collecting tax revenues on transactions that otherwise would not be recorded.

to sum up, i think it would be a good idea to convert current monetary systems into a system of electronic accounts. the economic benefits, convenience and savings afforded by such a system, along with the potential to reduce crime, far outweigh the remote loss of a significant social or political shift toward totalitarianism.

9. “employees should keep their private lives and personal activities as separate as possible from the workplace.”discuss the extent to which you agree or disagree with the opinion stated above. support your views with reasons and/or examples from your own experience, observations, or reading.

should employees leave their personal lives entirely behind them when they enter the workplace, as the speaker suggests here? while i agree that employees should not allow their personal lives to interfere with their jobs, the speaker fails to consider that integrating personal life with work can foster a workplace ambiance that helps everyone do a better job, thereby promoting success for the organization.

engaging coworkers in occasional conversation about personal interests and activities can help build collegiality among coworkers that adds to their sense of common purpose on the job. managers would be well advised to participate in and perhaps even plan the sharing of personal information—as a leadership tool as well as a morale booster. an employee feels valued when the boss takes time to ask about the employee’s family or recent vacation. the employee, in turn, is likely to be more loyal to and cooperative with the boss. company-sponsored social events—picnics, parties, excursions, and so forth—also help to produce greater cohesiveness in an organization, by providing opportunities for employees to bond with one another in ways that translate into (v. 翻譯成, 轉化為) better working relationships.

admittedly, employees should guard against allowing their personal life to impinge upon their job performance or intrude on coworkers. excessive chatting about non-business topics, frequent personal telephone calls, and the like, are always distracting. and romances between coworkers are best kept confidential, at least to the extent they disrupt work or demoralize or offend other employees. by the same token, however, employees who are too aloof—sharing nothing personal with others—may be resented by coworkers who perceive them as arrogant, unfriendly, or uncooperative. the ill-will and lack of communication that is likely to result may ultimately harm the organization.

in the final analysis, employees should strike a careful balance (strike a balance: v. 結帳, 公平處理) when they mix their personal lives with their jobs. although there are some circumstances in which bringing one’s personal life to the job may be counterproductive, for many reasons it is a good idea to inject small doses of personal life into the workplace.

10. “in any enterprise, the process of making or doing something is ultimately more important than the final product.”discuss the extent to which you agree or disagree with the opinion expressed above. support your point of view with reasons and/or examples from your own experience, observations, or reading.

the question at hand (adv. 在手邊, 在附近, 即將到來) is whether the process of making or doing something is ultimately more important than the final product. process may not always be more important than product, but it often is. a process may provide an opportunity for new and important discoveries with ramifications far beyond the current product; moreover, a process can often be an important end in and of itself for those engaged in it.

new discoveries are often unexpectedly made during routine processes. such was the case with alexander fleming in 1928, who while conducting an unremarkable study of bacteria, discovered inadvertently that mold growing on one of his cultures was killing the bacteria. his ordinary process led to an unexpected and remarkable end: the development of penicillin.

process also offers opportunities for refining old methods and inventing new ones. for example, as the defense industry slowed down after the cold war, many methods and technologies for weapons production proved useful in other areas from commercial aviation to medical technology. the same has been true of technologies developed for the space program, which now find broad application in many other fields.

finally, in my observation and experience, people become caught up in processes primarily for the challenge and enjoyment of the activity, not merely to produce some product. once the process has culminated in a final product, the participants immediately search for a new process to involve themselves with. from a psychological standpoint, then, people have a need to busy themselves with meaningful activities—i.e., processes. so most processes can fittingly be characterized as ends in themselves insofar as they fulfill this psychological need.

in sum, the process of making or doing something frequently has implications far beyond the immediate product. for this reason, and because process fills a basic human need, i strongly agree with the speaker’s assertion the process is ultimately more important than product.

11. “when someone achieves greatness in any field — such as the arts, science, politics, or business — that person’s achievements are more important than any of his or her personal faults.”discuss the extent to which you agree or disagree with the opinion stated above. support your views with reasons and/or examples from your own experience, observations, or reading.

perhaps in some instances the personal failings of great achievers are unimportant relative to the achievements. in many cases, however, the relative significance of personal failings can be very great, depending on two factors: (1) the extent to which the failing is part of the achievement process itself, and (2) the societal impact of the achiever’s failing apart from his or her own success.

personal failings and achievement are often symbiotically related. the former test the would-be achiever’s mettle; they pose challenges—necessary resistance that drives one to achieve despite the shortcoming. personal failings may also compel one to focus on one’s strengths, thereby spawning achievement. for example, poor academic or job performance may propel a gifted entrepreneur to start his or her own business. in the arts, a personal failing may be a necessary ingredient or integral part of the process of achieving. artists and musicians often produce their most creative works during periods of depression, addiction, or other distress. in business, insensitivity to the “human” costs of success has bred grand achievements, as with the questionable labor practices of the great philanthropist andrew carnegie.

a second type of personal failing is one that is unrelated to the achievement. modern politics is replete with examples: the marital indiscretions of the great leader john f. kennedy and the paranoia of the great statesman richard nixon, to name just two. were the personal failings of these two presidents less “important” than their achievements? in the former example, probably so. in the latter example, probably not since it resulted in the watergate scandal—a watershed event in american politics. in cases such as these, therefore, the societal impact of shortcoming and achievement must be weighed on a case-by-case basis.

in sum, history informs us that personal failings are often part-and-parcel (n. 重要的部分) of great achievements; even where they are not, personal shortcomings of great achievers often make an important societal impact of their own.

12. “education has become the main provider of individual opportunity in our society. just as property and money once were the keys to success, education has now become the element that most ensures success in life.”in your opinion, how accurate is the view expressed above? explain, using reasons and examples based on your own experience, observations, or reading.

which factor offers more opportunities for success in our society: education or money and property? in my view, education has replaced money and property as the main provider of such opportunities today. i base my view on two reasons. first, education—particularly higher education (高等教育)—used to be available only to the wealthy but now is accessible to almost anyone. second, because of the civil-rights movement and resulting laws, businesses are now required to hire on the basis of merit rather than the kinds of personal connections traditionally common among the wealthy.

education probably always played a key role in determining one’s opportunities for success. but in the past, good post-secondary education was available mainly to the privileged classes. because money and property largely determined one’s access to higher education, money and property really were the critical factors in opening doors to success. however, higher education is more egalitarian today. given our vast numbers of state universities and financial-aid programs, virtually anyone who meets entrance requirements for college can obtain an excellent college education and open up windows of opportunity in life.

another reason those opportunities will be open to educated young people from middle-class and poorer backgrounds is that hiring is more meritocratic today than ever before. in principle (原則上;大致上;通常on principle: 按照原則(或道德標準);根據原則(或道德標準)), at least, we have always been a society where all people are equal; yet in the past, children of the wealthy and the well connected could expect to obtain higher-status jobs and to receive better pay. but the laws and programs resulting from our civil-rights struggles have produced a modern business climate in which jobs are available on an equal-opportunity basis, and in which candidates have a legal right to be judged on the merit of their educational background and experience.

in conclusion, education is probably the main factor in opening doors to success for young people in our society. the fact that education has supplanted money and property in this role is owing to a more egalitarian system of higher education, as well as to more merit-based hiring practices that generally value individual education over family fortune or connections.

13. “responsibility for preserving the natural environment ultimately belongs to each individual person, not to government.”discuss the extent to which you agree or disagree with the opinion stated above. support your views with reasons and/or examples from your own experience, observations, or reading.

while nearly everyone would agree in principle that certain efforts to preserve the natural environment are in humankind’s best interest, environmental issues always involve a tug of war (n. 拔河, 兩派間的激烈競爭) among conflicting political and economic interests. for this reason, and because serious environmental problems are generally large in scale, government participation is needed to ensure environmental preservation.

experience tells us that individuals (and private corporations owned by individuals) tend to act on behalf of their own short-term economic and political interest, not on behalf of the environment or the public at large. for example, current technology makes possible the complete elimination of polluting emissions from automobiles. nevertheless, neither automobile manufacturers nor consumers are willing or able to voluntarily make the short-term sacrifices necessary to accomplish this goal. only the government holds the regulatory and enforcement power to impose the necessary standards and to ensure that we achieve such goals.

aside from the problems of self-interest and enforcement, environmental issues inherently involve public health and are far too pandemic in nature for individuals to solve on their own. many of the most egregious environmental violations traverse state and sometimes national borders. environmental hazards are akin to those involving food and drug safety and to protecting borders against enemies; individuals have neither the power nor the resources to address these widespread hazards.

in the final analysis, only the authority and scope of power that a government possesses can ensure the attainment of agreed-upon environmental goals. because individuals are incapable of assuming this responsibility, government must do so.

14. “organizations should be structured in a clear hierarchy in which the people at each level, from top to bottom, are held accountable for completing a particular component of the work. any other organizational structure goes against human nature and will ultimately prove fruitless.”discuss the extent to which you agree or disagree with the opinion expressed above. support your point of view with reasons and/or examples from your own experience, observations, or reading.

the speaker claims that all organizations should include a clear hierarchy of accountability because any other structure would work against human nature and therefore prove fruitless in the end. this claim gives rise to complex issues about human nature and the social structures best suited to it. in my view, the claim assumes a distortedly narrow view of human nature, ignoring certain aspects of it that are undermined by hierarchical structure in ways that ultimately hurt the organization.

first, the organizational structure the speaker recommends undermines the nexus between worker and product that facilitates efficiency and productivity. when employees are responsible for just their small component of work, they can easily lose sight of larger organizational goals and the importance of their role in realizing these goals. in turn, workers will feel alienated, unimportant, and unmotivated to do work they are proud of. these effects cannot help but damage the organization in the end.

second, compartmentalizing tasks in a hierarchical structure stifles creativity. an acquaintance of mine worked for a company that had established a rigid organizational barrier between designers and engineers. the designers often provided the engineers with concepts that were unworkable from an engineering standpoint. conversely, whenever an engineer offered a design idea that allowed for easier engineering, the designers would simply warn the engineer not to interfere. this is a typical case where organizational barriers operate against creativity, harming the organization in the end.

third, strict hierarchy undermines the collegiality and cooperation among coworkers needed for a sense of common purpose and pride in accomplishment. the message from the designers to the engineers at my friend’s company produced just the opposite—resentment between the two departments, low morale among the engineers whose creative suggestions were ignored, and ultimate resignation to do inferior work with an attitude that developing ideas is a waste of time.

in sum, the speaker seems to assume that humans are essentially irresponsible and unmotivated, and that they therefore need external motivation by way of a layered bureaucratic structure. the speaker misunderstands human nature, which instead requires creative exercise and sense of purpose and pride in accomplishment. by stifling these needs with organizational barriers, the organization is ultimately worse off.

15. “nations should cooperate to develop regulations that limit children’s access to adult material on the internet.” **the internet is a worldwide computer network.

discuss the extent to which you agree or disagree with the opinion stated above. support your views with reasons and/or examples from your own experience, observations, or reading.

the issue here is whether an international effort to regulate children’s access to adult material on the internet is worthwhile. in my view, nations should attempt to regulate such access by cooperative regulatory effort. i base this view on the universality and importance of the interest in protecting children from harm, and on the inherently pandemic nature of the problem.

adults everywhere have a serious interest in limiting access by children to pornographic material. pornographic material tends to confuse children—distorting their notion of sex, of themselves as sexual beings, and of how people ought to treat one another. particularly in the case of domination and child pornography, the messages children receive from pornographic material cannot contribute in a healthy way to their emerging sexuality. given this important interest that knows no cultural bounds, we should regulate children’s access to sexually explicit material on the internet.

however, information on the internet is not easily contained within national borders. limiting access to such information is akin to preventing certain kinds of global environmental destruction. consider the problem of ozone depletion thought to be a result of chloroflourocarbon (cfc) emissions. when the government regulated cfc production in the u.s., corporations responsible for releasing cfc’s into the atmosphere simply moved abroad, and the global threat continued. similarly, the internet is a global phenomenon; regulations in one country will not stop “contamination” overall. thus, successful regulation of internet pornography requires international cooperation, just as successful cfc regulation finally required the joint efforts of many nations.

admittedly, any global regulatory effort faces formidable political hurdles, since cooperation and compliance on the part of all nations—even warring ones—is inherently required. nevertheless, as in the case of nuclear disarmament or global warming, the possible consequences of failing to cooperate demand that the effort be made. and dissenters can always be coerced into compliance politically or economically by an alliance of influential nations.

in sum, people everywhere have a serious interest in the healthy sexual development of children and, therefore, in limiting children’s access to internet pornography. because internet material is not easily confined within national borders, we can successfully regulate children’s access to adult materials on the internet only by way of international cooperation.

16. “public buildings reveal much about the attitudes and values of the society that builds them. today’s new schools, courthouses, airports, and libraries, for example, reflect the attitudes and values of today’s society.”discuss the extent to which you agree or disagree with the opinion stated above. support your views with reasons and/or examples from your own experience, observations, or reading.

the extent to which new public buildings reflect societal values and attitudes depends on whether one considers a building’s intended function or its design. in the former sense, new public buildings do mirror society, while in the latter sense they do not.

the intended uses of new public buildings say something about our priorities and values as a society. for example, proliferation of public cultural centers and schools reflects a societal concern for the arts and education, respectively, while new prison construction indicates a heightened concern for safety and security.

the design of new public buildings, however, fails to mirror society, for two reasons. first, modern democratic states do not have the luxury of making cultural “statements” at any expense. functionality and fiscal accountability dictate the face of public architecture today. second, public participation in the process is limited. new buildings typically reflect the architect’s eccentric vision or the preference of a few public officials, not the populace’s values and attitudes. in england, for example, prince charles oversees and approves the design of new public buildings. the resulting conventional designs suggest his unwillingness to break from tradition. yet it would seem unfair to assign his lack of vision to english society. in denver, the controversial design of a new airport met with public outcry for its appearance, expense, and lack of functionality. does the airport reflect the values of denver’s denizens? probably not.

in conclusion, while modern public buildings seem to reflect the values and attitudes of a society in their function, they do not necessarily do so in their design.

17. “some people believe that the best approach to effective time management is to make detailed daily and long-term plans and then to adhere to them. however, this highly structured approach to work is counterproductive. time management needs to be flexible so that employees can respond to unexpected problems as they arise.”discuss the extent to which you agree or disagree with the opinion expressed above. support your point of view with reasons and/or examples from your own experience, observations, or reading.

the speaker claims that a detailed time-management plan fails to afford adequate flexibility to deal with the unexpected at the workplace. he seems to offer an either/or choice between planning one’s time rigidly, by detailing important daily as well as long-term plans, and not planning at all; and he prefers the second choice. the speaker’s claim is overly simplistic, since it is possible for a detailed time-management plan to also provide flexibility.

working at any job without a detailed road map (n. 行車圖, 公路線路圖) for the immediate and longer-term can trivialize the efforts of both employees and organizational units so that all their efforts become aimless. the only sensible way proceed is to consider first one’s most important long-term objectives; then an organizational unit and its employees can order daily and weekly tasks according to how much each adds to the achievement of those objectives. with a broader perspective, workers can eliminate from the list those daily activities that may seem urgent or may be most enjoyable but don’t really contribute to long-term job goals or to organizational objectives.

a detailed time-management system need not be inflexible. knowing which items to eliminate from a “to-do (bustle, stir, fuss)” list gives a time-management plan its flexibility. when the unexpected arises, it can be judged according to its role in fulfilling long-term goals. if what at first seemed urgent turns out not to be important, it can be deferred to another time or ignored altogether. but if something unexpected needs handling in order to fulfill an important business or life plan, it will take priority over lesser activities in the daily or weekly schedule. for instance, i might have a meeting planned for one o’clock with coworkers to decide the location of an awards banquet, and find out at noon that an important client is thinking of switching to our competitor but wants to talk with me first. i can easily discern that the banquet meeting is less important than a critical meeting with a valuable client.

in conclusion, effective time management must involve a detailed scheduling of tasks. but it also requires determining which tasks are more central than others to the satisfaction of long-term objectives. this way, the daily or weekly schedule becomes not just a list of tasks to check off (vt. 校對,查對,清點), but a flexible plan that can accommodate important urgencies while allowing us to bypass less significant scheduled tasks and ignore unimportant interruptions.

18. “if the primary duty and concern of a corporation is to make money, then conflict is inevitable when the corporation must also acknowledge a duty to serve society.”from your perspective, how accurate is the above statement? support your position with reasons and/or examples from your own experience, observations, or reading.

we take for granted that a primary objective and obligation of a corporation is to maximize profits. but does this mean a corporation cannot also fulfill its obligations to society? the speaker claims that the two duties necessarily conflict. in my view, however, a corporation’s duties to maximize shareholder wealth and to serve society will at times coincide and at times conflict; and when they do conflict, neither takes automatic precedence over the other.

beyond the obvious duty to maximize shareholder wealth, corporations indeed owe a duty to serve society, especially the immediate community, which permits corporations to operate in exchange for an implicit promise that the corporations will do no harm and will bring some benefit to the community. these duties can often be fulfilled together. for example, a successful corporation brings jobs and related economic benefit to the community. and, by contributing to community activities and changes in other ways, the corporation gains a reputation for social responsibility that often helps it become even more successful.

however, at times these duties do conflict. consider, for instance, a company that unknowingly leaks into the ground a toxic substance that threatens to contaminate local groundwater. while the company may favor an inexpensive containment program, community leaders may want the company to go further by cleaning up and restoring their environment—even if the expense will force the company to leave and take jobs from the community. whatever the company decides, it should not assume that protecting profits automatically outweighs social obligation. in many instances it does not, as highly visible tobacco, automobile safety, and asbestos liability cases aptly illustrate. such examples reveal a limit as to how far a corporation can ethically go in trading off the well being of the community for the sake of its own profits.

in sum, corporations have duties both to do well and to do good. although conflict between these duties is not inevitable, it does occur. determining which duty takes precedence in time of conflict requires careful consideration of all the ethical ramifications of each alternative.

19. some employers who recruit recent college graduates for entry-level jobs evaluate applicants only on their performance in business courses such as accounting, marketing, and economics. however, other employers also expect applicants to have a broad background in such courses as history, literature, and philosophy.do you think that, in the application process, employers should emphasize one type of background — either specialization in business courses or a more varied academic preparation — over the other? why or why not? develop your position by using reasons and/or examples from your own experience, observations, or reading.

sample essay 1:

whether an employer should emphasize specialization in business courses or a more varied academic preparation is a controversial one. on the one hand, the increasing diversification of business activities requires employees to have specialized knowledge. on the other hand, the capricious nature of the market needs employees to have a more varied academic preparation so that he could handle unexpected situations. however, in the final analysis, i believe that an employer should emphasize specialized knowledge in business courses.

one reason for my belief is that there are special requirements for each position of a company and only those who have adequate knowledge for the position can take the position. if everyone does his job well, the whole company will prosper.

another reason for my belief lies in the fact that entry-level employees do not need a varied academic preparation, for they do not have to handle complicated situations. unlike those of a senior staff member, their responsibilities are clearly defined in the job description.

perhaps the best reason for my belief is that one’s energy is limited. if the employer expects their employees to have a more varied academic preparation, college graduates will spend less time on their own special field of study. as a result, they may not have adequate special knowledge for their future positions.

for the reasons above i therefore believe that an employer should emphasize specialization in business courses in the application process. although general knowledge is also important in many respects, a specialist is more useful for a company.

sample essay 2:

in recruiting for entry-level jobs, should employers stress a broad liberal arts education, a technical business background, or should employers favor neither one over the other? in my view, while the ideal job candidate has significant academic experience in both realms, whether employers should favor one type of background over the other depends on the nature of the particular job and the anticipated length of employment.

first, a strong business background is more critical for some entry-level jobs than for others. fledgling accountants, financial analysts, and loan officers cannot perform optimally without a solid academic background in accounting, finance, and banking. even in sales of financial products and services, new employees need extensive technical knowledge to educate the customer and to be effective salespeople. however, in other entry-level positions—such as personnel, advertising and marketing—technical business knowledge may not be as critical as a broad experience with various types of people and an enlightened view of different cultures.

second, the employer’s hiring decision should also depend on the anticipated length of employment. in recruiting short-term workers, especially for positions that are labor intensive and where judgment and experience are not of paramount importance, the applicant who is strongly business-oriented may be the better choice. on the job, this applicant will probably be more pragmatic, and spend less time pondering the job and more time doing it. however, an employer looking for a long-term employee may be better served by hiring an applicant with a strong liberal arts background. by way of their more general education, these applicants have acquired a variety of general, transferable skills. they may be more adept than their colleagues with business-only backgrounds at recognizing and solving management problems, dealing with business associates from different cultures, and viewing issues from a variety of perspectives. all of these skills contribute to a person’s lifelong ability to adapt to and even anticipate changes that affect the company, and to move easily into new positions as such changes demand.

in sum, recruiters for entry-level jobs should avoid preferring one type of applicant over another in all cases. instead, recruiters should consider the immediate technical demands of the job as well as the prospect of advancement and long-term employment within the company.

20. “in this age of automation, many people complain that humans are becoming subservient to machines. but, in fact, machines are continually improving our lives.”discuss the extent to which you agree or disagree with the opinion expressed above. support your point of view with reasons and/or examples from your own experience, observations, or reading.

sample essay 1:

the issue of whether machines are an advantage or disadvantage to humans is a controversial one. on the one hand, humans are more and more dependent on machines. on the other hand, machines are making our lives better and better. however, in the final analysis, i believe that the advantages of machines outweigh their disadvantages.

one reason for my belief is that machines have made our lives much easier than before. for example, with help of my computer, i can navigate on the internet everyday, searching for the information i need, while my automatic washing machine is doing my laundry for me. my mobile phone connects me with my friends and my office wherever i go. i cannot image what my life would be like without all these machines and devices.

another reason for my belief is that machines can do many dangerous work for us. for example, a robot bomb expert can dismantle a bomb for the police so that no one will be hurt. other robots can work under extreme weather conditions.

perhaps the best reason for my belief is that machines have opened more and more possibilities for humans. for instance, a spaceship can take us to outer space where we had never dared to go. likewise, a submarine can bring us to the bottom of the ocean, which used to be forbidden area to humans. i believe that there will be more machines doing hazardous jobs.

for all these reasons, i therefore believe that machines are so important to humans that we cannot do without them. of course, machines have also brought with it many disadvantages. such machines as calculators, cars, typewriters have made some people lazy, stupid, weak, and clumsy. however, whether machines are beneficial to humans depends on how you use them. we can use machines to save us time and then use the time to do more creative work or to enjoy life. anyway, there are still more advantages than disadvantages. (326 words)

sample essay 2:

in some respects humans serve machines, while in other respects machines serve us by enhancing our lives. while mechanical automation may have diminished our quality of life on balance (adv. 總而言之), digital automation is doing more to improve our lives than to undermine our autonomy.

consider first mechanical automation, particularly assembly line manufacturing. with automation came a loss of pride in and alienation from one’s work. in this sense, automation both diminished our quality of life and rendered us slaves to machines in our inability to reverse “progress.” admittedly, mechanical automation spawned entire industries, creating jobs, stimulating economic growth, and supplying a plethora of innovative conveniences. nevertheless, the sociological and environmental price of progress may have outweighed its benefits.

digital automation has brought its own brand of alienation. computer automation, and especially the internet, breeds information overload and steals our time and attention away from family, community, and coworkers. in these respects, digital automation tends to diminish our quality of life and create its own legion of human slaves. on the other hand, by relegating repetitive tasks to computers, digital technology has spawned great advances in medicine and physics, helping us to better understand the world, to enhance our health, and to prolong our lives. digital automation has also emancipated architects, artists, designers, and musicians, by opening up creative possibilities and by saving time. perhaps most important, however, information technology makes possible universal access to information, thereby providing a democratizing influence on our culture.

in sum, while mechanical automation may have created a society of slaves to modem conveniences and unfulfilling work, digital automation holds more promise for improving our lives without enslaving us to the technology.

21. “job security and salary should be based on employee performance, not on years of service. rewarding employees primarily for years of service discourages people from maintaining consistently high levels of productivity.”discuss the extent to which you agree or disagree with the opinion stated above. support your views with reasons and/or examples from your own experience, observations, or reading.

according to the statement, in order to ensure high productivity, companies should base their employees’ salaries and job security solely on job performance, and not on length of service to the company. i agree that salary increases and job security are powerful incentives to high achievement and should generally go to those who do the best work. however, to ensure employee productivity, companies must also reward tenured employees with cost-of-living raises—though not with job security.

on the one hand, rewarding average job performance with large pay increases or promises of job security is a waste of resources—for two reasons. first, complacent employees will see no reason to become more productive. secondly, those normally inclined to high achievement may decide the effort isn’t worthwhile when mediocre efforts are amply compensated. companies should, therefore, adjust their pay schedules so that the largest salaries go to the most productive employees.

on the other hand, employees who perform their jobs satisfactorily should be given regular, though small, service-based pay increases—also for two reasons. first, the cost of living is steadily rising, so on the principle of fair compensation alone, it is unjust to condemn loyal employees to de facto salary reductions by refusing them cost-of-living raises. secondly, failure to adjust salaries to reflect the cost of living may be counterproductive for the firm, which will have difficulty attracting and retaining good employees without such a policy.

in the final analysis, the statement correctly identifies job performance as the single best criterion for salary and job security. however, the statement goes too far; it ignores the fact that a cost-of-living salary increase for tenured employees not only enhances loyalty and, in the end, productivity, but also is required by fairness.

22. “clearly, government has a responsibility to support the arts. however, if that support is going to produce anything of value, government must place no restrictions on the art that is produced.”to what extent do you agree or disagree with the opinion expressed above? develop your position by giving specific reasons and/or examples from your own experience, observations, or reading.

the speaker here argues that government must support the arts but at the same time impose no control over what art is produced. the implicit rationale for government intervention in the arts is that, without it, cultural decline and erosion of our social fabric will result. however, i find no empirical evidence to support this argument, which in any event is unconvincing in light of more persuasive arguments that government should play no part in either supporting or restricting the arts.

first, subsidizing the arts is neither a proper nor a necessary job for government. although public health is generally viewed as critical to a society’s very survival and therefore an appropriate concern of government, this concern should not extend tenuously to our cultural “health” or well being. a lack of private funding might justify an exception; in my observation, however, philanthropy is alive and well today, especially among the new technology and media moguls.

second, government cannot possibly play an evenhanded role as arts patron. inadequate resources call for restrictions, priorities, and choices. it is unconscionable (無節制的;過度的) to relegate normative (conforming to or based on norms *normative behavior* *normative judgments*) decisions as to which art has “value” to a few legislators and jurists (法學家;法理學家: one having a thorough knowledge of law; especially: judge), who may be unenlightened in their notions about art. also, legislators are all too likely to make choices in favor of the cultural agendas of those lobbyists with the most money and influence.

third, restricting artistic expression may in some cases encroach upon the constitutional right of free expression. in any case, governmental restriction may chill creativity, thereby defeating the very purpose of subsidizing the arts.

in the final analysis, government cannot philosophically or economically justify its involvement in the arts, either by subsidy or sanction. responsibility lies with individuals to determine what art has value and to support that art.

23. “schools should be responsible only for teaching academic skills and not for teaching ethical and social values.”discuss the extent to which you agree or disagree with the opinion expressed above. support your point of view with reasons and/or examples from your own experience, observations, or reading.

the speaker asserts that schools should teach only academic skills, and not ethical or social values. i agree with the speaker insofar as instruction on certain moral issues is best left to parents and churches. however, in my view it is in the best interests of a democratic society for schools to teach at least the values necessary to preserve freedom and a democratic way of life, and perhaps even additional values that enrich and nurture a society and its members.

we all have in interest in preserving our freedom and democratic way of life. at the very least (prep. 最低限度), then, schools should provide instruction in the ethical and social values required for our democracy to survive—particularly the values of respect and tolerance. respect for individual persons is a basic ethical value that requires us to acknowledge the fundamental equality of all people, a tenet of a democratic society. tolerance of differences among individuals and their viewpoints is required to actualize many of our basic constitutional rights—including life, liberty, pursuit of happiness, and freedom of speech and religion.

while respect and tolerance are the minimal values that schools should teach, the list should ideally go further—to include caring, compassion, and willingness to help one another. a democracy might survive without these values, but it would not thrive. respect and tolerance without compassion, it seems to me, breed a cool aloofness that undermines our humanity, and leaves those in the worst position to suffer more and suffer alone—an unhealthy state for any society.

admittedly, schools should avoid advocating particular viewpoints on controversial moral issues such as abortion or capital punishment. instruction on issues with clear spiritual or religious implications is best left to parents and churches. even so, schools should teach students how to approach these kinds of issues—by helping students to recognize their complexity and to clarify competing points of view. in doing so, schools can help breed citizens who approach controversy in the rational and responsible ways characteristic of a healthy democracy.

in sum, schools should by all means refrain from indoctrinating our young people with particular viewpoint on controversial questions of morality. however, it is in a democratic society’s interest for schools to inculcate the democratic values of respect and tolerance, and perhaps even additional values that humanize and enrich a society.

24. “a powerful business leader has far more opportunity to influence the course of a community or a nation than does any government official.”discuss the extent to which you agree or disagree with the opinion stated above. support your views with reasons and/or examples from your own experience, observations, or reading.

historical examples of both influential public officials and influential business leaders abound. however, the power of the modern-era business leader is quite different from that of the government official. on balance, the ceo seems to be better positioned to influence the course of community and of nations.

admittedly the opportunities for the legislator to regulate commerce or of the jurist to dictate rules of equity are official and immediate. no private individual can hold that brand of influence. yet official power is tempered by our check-and-balance system (制約平衡制度) of government and, in the case of legislators, by the voting power of the electorate. our business leaders are not so constrained, so, their opportunities far exceed those of any public official. moreover, powerful business leaders all too often seem to hold de facto legislative and judicial power by way of their direct influence over public officials, as the clinton administration’s fund-raising scandal of 1997 illuminated all too well.

the industrial and technological eras have bred such moguls of capitalism as pullman, rockefeller, carnegie, and gates, who by the nature of their industries and their business savvy, not by force of law, have transformed our economy, the nature of work, and our very day-to-day (adj. 日常的, 逐日的) existence. of course, many modern-day public servants have made the most of their opportunities—for example, the crime-busting (bust: to break or smash especially with force;) mayor rudolph giuliani and the new-dealing president franklin roosevelt. yet their impact seems to pale next to those of our modern captains of industry.

in sum, modem business leaders by virtue of the far-reaching impact of their industries and of their freedom from external constraints, have supplanted lawmakers as the great opportunists of the world and prime movers of society.

25. “the best strategy for managing a business, or any enterprise, is to find the most capable people and give them as much authority as possible.”discuss the extent to which you agree or disagree with the opinion stated above. support your views with reasons and/or examples from your own experience, observations, or reading.

is the most effective management approach to hire the best people, then to give them as much autonomy as possible to serve the firm’s goals? this strategy would certainly enhance an employee’s sense of involvement, purpose and personal worth. it would also benefit the firm by encouraging employees to work creatively and productively. but the strategy requires two constraints to operative effectively.

first, the strategy must be constrained by strong leadership that provides clear vision and direction. simply putting the most capable people together, and letting them loose on projects will provide neither. thinking so involves the mistaken assumption that just because the parts of a whole are good, the collection of the parts into a whole will be equally good. business organizations are more than just the sums of their excellent parts; to be similarly excellent, the organization must also be unified and cohesive. and it is strong and visionary leadership that provides these two ingredients.

second, the strategy must be constrained by an organizational structure that brings all individual efforts together as a coherent whole. of course, structure can be crippling, heavily layered; overly bureaucratic organizations probably stifle more creative productivity than they inspire. still, individuals will be capable at some things and not others, so some organization of efforts is always called for. the moderate—and perhaps optimal—approach would be to create a structure that gives individuals some authority across areas relating to their field of expertise, while reserving final authority for higher-level managers. for example, no individual in a finance department should have much authority over a design department. however, within the design department, individual researchers, artists, drafters, and engineers can all contribute meaningfully to one another’s projects, and a flexible organizational structure would allow them to do so.

in sum, the advice to hire the best people and give them wide authority requires modification. hiring capable people and granting them some concurrent authority across areas related to their expertise is better advice. moreover, solid leadership and a cohesive organizational structure are prerequisites—both are needed to coordinate individual efforts toward the accomplishment of common goals.

26. “location has traditionally been one of the most important determinants of a business’s success. the importance of location is not likely to change, no matter how advanced the development of computer communications and others kinds of technology becomes.”discuss the extent to which you agree or disagree with the opinion stated above. support your views with reasons and/or examples from your own experience, observations, or reading.

in retail, or “storefront (n. 店頭, 店面),” business, location is still a key ingredient of business success. the extent to which this will continue to be true, given the inexorable growth of internet commerce, will vary among industries.

in more traditional retail sectors, such as clothing, cosmetics, and home improvement, an in-person (adv. 親自, 外貌上) visit to a retail store is often necessary—to try on (v. 試穿, 試驗) clothes for fit, compare fragrances, or browse among a full selection of textures, colors, and styles. also, activities such as shopping and dining out are for many consumers enjoyable experiences in themselves, as well as excuses to get out of the house and mingle with others in their community. finally, shipping costs for large items such as appliances and home-improvement items render home shopping impracticable. thus, burgeoning technologies pose no serious threat to main street, and location will continue to play a pivotal role in the fate of many retail businesses.

nevertheless, technology-related industries are sure to move away from physical storefronts to virtual ones. products that can be reduced to digital “bits and bites,” such as books and magazines, recordings, and software applications, are more efficiently distributed electronically. computer hardware will not disappear from main street quite so quickly, though, since its physical look and feel enters into the buying decision. computer superstores should continue to thrive alongside companies such as dell, which does not distribute through retail stores.

in conclusion, consumer demand for convenient location will continue with respect to certain tangible products, while for other products alternative distribution systems will gradually replace the storefront, rendering location an obsolete issue.

27. “a company’s long-term success is primarily dependent on the job satisfaction and the job security felt by the company’s employees.”discuss the extent to which you agree or disagree with the opinion stated above. support your views with reasons and/or examples from your own experience, observations, or reading.

i agree that job satisfaction is an important factor in determining whether a company will be successful in the long term. however, other factors typically play just as vital a role in the ultimate success or failure of a business. at the same time, job security is becoming decidedly unimportant for many employees and, in any event, often leads to substandard job performance.

i agree that business success is more likely when employees feel satisfied with their jobs. employees who dislike the workplace or their jobs are not likely to reach their potential performance levels; they may tend to arrive late for work, perform their tasks in an unimaginative and sluggish manner, or take excessive sick leaves. nevertheless, a firm’s long-term success may equally result from other factors such as finding a market niche for products, securing a reputation for quality products and services, or forming a synergistic alliance with a competitor. this list hardly exhausts all the factors that can contribute to a firm’s ultimate success, and no one of them—including job satisfaction—is pivotal in every case.

while job satisfaction clearly boosts employee morale and contributes to the overall success of a company, the same cannot be said for job security. admittedly an employee worried about how secure his or her job is might be less creative or productive as a result. by the same token, however, too much confidence in the security of one’s job can foster complacency, which, in turn, may diminish employees’ creativity and productivity. moreover, many employees actually place job security relatively low on the list of what they want in a job. in fact, more and more workers today are positively uninterested in long-term job security; instead, they are joining firms for the sole purpose of accomplishing near-term professional goals, then leaving to face the next challenge.

to sum up, the claim at issue overrates the importance of job satisfaction and security by identifying them as the key factors in a company’s long-term success. job satisfaction among employees is very important, but it is not clearly more important than many other factors. at the same time, job security is clearly less important, and even unimportant in some cases.

28. “because businesses use high-quality advertising to sell low-quality products, schools should give students extensive training in how to make informed decisions before making purchases.”discuss the extent to which you agree or disagree with the opinion expressed above. support your point of view with reasons and/or examples from your own experience, observations, or reading.

this argument is untenable for two reasons. first, the claim that high-quality ads are used to promote tow-quality products is unsupported empirically and by common sense. second, undue attention by schools to consumerism (n保護消費者權益運動) is unnecessary and inappropriate, especially for younger students.

regarding the first reason, empirical evidence does not suggest that high-quality advertising is used to promote low-quality products. to the contrary (adv. 相反地), companies that produce low-quality products seem to resort to low-budget, poor-quality ads, especially in broadcast media. firms that take pride in the quality of their products are far more likely also to produce ads they can be proud of. furthermore, high-quality products are more likely to succeed in the marketplace and thereby generate the revenues needed to ensure high production value in advertising.

as for the second reason, it is not the job of our schools to breed legions of smart shoppers. teachers should devote class time to examining the market place of ideas, not that of consumer goods and services, which students spend sufficient time examining outside the classroom. admittedly consumerism and advertising may be appropriate topics for college-level marketing and psychology courses. however, undue focus on media and materialism may give younger students a distortedly harrow view of the world as little more than a flea market. additionally, revealing the deceptive side of the advertising business may breed unhealthy cynicism among youngsters, who need positive messages, not negative ones, during their formative years.

in sum, the premise that high-quality ads tout low-quality products is specious at best; in any event, for schools to provide extensive training in consumerism would be to assign them an inappropriate role and to foster in impressionable minds a distortedly narrow and unhealthy view of the world.

29. “too many people think only about getting results. the key to success, however, is to focus on the specific task at hand and not to worry about results.”what do you think this piece of advice means, and do you think that it is, on the whole, worth following? support your views with reasons and/or examples drawn from your own experience, observations, or reading.

this advice means fundamentally that if we focus our attention on the details of a project rather than on the end product (最後產物,最終產品), the result will be better than if we proceed the other way around (adv. 從相反方向). admittedly, this advice has some merit; by focusing on the details at hand one is less likely to become discouraged by the daunting or overwhelming tasks ahead in an ambitious project. otherwise, however, i think this advice is poor,

the central problem with this advice is that focusing attention completely on the task at hand without reference to how that task is related to the end product would be virtually impossible to do. the reason for this is simple. without some reference to a goal or a result we would have no idea of what task to perform in the first place. as a result, the various tasks we engage in would be somewhat random and, in turn, no matter how diligent and careful we were in performing them the likelihood of producing worthwhile or successful end products would be minimal.

to ensure good results, one should instead take a balanced approach to the task at hand (adv. 在手邊, 在附近, 即將到來). by a balanced approach i mean paying attention to both the desired result and the specific tasks that are required to achieve it. house building provides a good example of this approach. the house plan not only contains a rendering of the finished product but also contains detailed drawings and descriptions of each of the specific components required to ensure a successful result. moreover, the order of the tasks is determined with reference to this result. in my estimation, virtually all successful projects proceed in the fashion illustrated in this example.

in sum, i don’t think that the advice offered in the statement is worth following. in my view, following this advice is more likely to produce unsuccessful results than successful ones.

30. “companies benefit when they discourage employees from working extra hours or taking work home. when employees spend their leisure time without ‘producing’ something for the job, they will be more focused and effective when they return to work.”discuss the extent to which you agree or disagree with the opinion expressed above. support your point of view with reasons and/or examples from your own experience, observations, or reading.

according to this statement, companies would be well advised to discourage employees from working overtime or from taking projects home, since employees are more productive when they return to the job after a break from their work. while i agree with this policy in general, on some occasions the company stands to benefit more from asking employees to forego (variant of forgo: give up) leisure time than from insisting they be rested and refreshed when they come to work.

in the normal course of business operations, companies benefit when they discourage employees from putting in long hours or from taking work home. breaks from work provide opportunities to enjoy outside interests and activities, and to spend important time with friends and family. employees who make time for relationships and leisure activities will find that they return to the job refreshed and with new perspectives on the challenges they face at work. both of these factors contribute to clearer focus on the task at hand and greater efficiency.

at the same time, every organization is familiar with the press of crucial deadlines and other crisis situations. at such times a company should call upon employees to work overtime, and even to take projects home, especially when doing so might make the difference between the business’ success or failure. moreover, it is in the company’s best interest to reward the devoted worker accordingly—not in order to encourage workaholic (工作狂) habits but rather to foster (rather後面應該跟原形!) good will and loyalty.

in sum, i agree that encouraging employees to make a habit of working after hours or taking work home is generally counterproductive for an enterprise. nevertheless, in exceptional situations, especially where the company is at great risk, calling on employees to forego their ordinary schedules and to work overtime is well justified.

31. “financial gain should be the most important factor in choosing a career.”discuss the extent to which you agree or disagree with the opinion stated above. support your views with reasons and/or examples from your own experience, observations, or reading.

financial gain is certainly one factor to consider when selecting a career. but many people do not, and should not, focus on this factor as the main one. the role that money plays in career choice should depend on the priorities, goals and values of the particular person making the choice.

the main problem with selecting a career primarily on the basis of money is that for many people to do so would be to ignore one’s personal values, needs, and larger life goals. indeed, many people appreciate this notion when they choose their career. for example, some people join one of the helping professions, such as nursing, teaching or social work, well aware that their career will not be financially lucrative. their choice properly stems from an overriding altruistic desire, not from an interest in financial gain. others choose to pursue intellectual or creative fulfillment—as writers, artists, or musicians—knowing that they are trading off dollars for non-tangible rewards. still others forego economic gain to work as full-time parents; for these people, family and children are of paramount importance in life. finally, many people subordinate economic prospects to their desire to live in a particular location; these people may place a high value on recreation, their physical health, or being near a circle of friends.

another problem with focusing primarily on money when selecting a career is that it ignores the notion that making money is not an end in the end of itself, but rather a means of obtaining material goods and services and of attaining important goals—such as providing security for oneself and one’s family, lifelong learning, or freedom to travel or to pursue hobbies. acknowledging the distinction, one may nevertheless select a career on the basis of money—since more money can buy more goods and services as well as the security, freedom, and time to enjoy them. even so, one must strike a balance, for if these things that money is supposed to provide are sacrificed in the pursuit of money itself, the point of having money—and of one’s career selection—has been lost.

in conclusion, economic gain should not be the overriding factor in selecting a career. while for a few people the single-minded pursuit of wealth may be fulfillment enough, most people should, and indeed do, temper the pursuit of wealth against other values, goals, and priorities. moreover, they recognize that money is merely a means to more important objectives, and that the pursuit itself may undermine the achievement of these objectives.

32. “you can tell the ideas of a nation by its advertisements.”explain what you think this quotation means and discuss the extent to which you agree or disagree with it. develop your position with reasons and/or specific examples drawn from history, current events, or your own experience, observations, or reading.

in order to determine whether advertisements reflect a nation’s ideas, it is necessary to determine whether advertisements present real ideas at all, and, if so, whose ideas they actually reflect. on both counts, it appears that advertisements fail to accurately mirror a nation’s ideas.

indisputably, advertisements inform us as to a nation’s values, attitudes, and priorities—what activities are worthwhile, what the future holds, and what is fashionable and attractive. for instance, a proliferation of ads for sport-utility vehicles reflects a societal concern more for safety and machismo (男子氣概an exaggerated or exhilarating sense of power or strength) than for energy conservation and frugality, while a plethora of ads for inexpensive on-line brokerage services reflects an optimistic and perhaps irrationally exuberant economic outlook. however, a mere picture of a social more, outlook, or fashion is not an “idea”—it does not answer questions such as “why” and “how”?

admittedly, public-interest advertisements do present ideas held by particular segments of society—for example, those of environmental and other public-health interest groups. however, these ads constitute a negligible percentage of all advertisements, and they do not necessarily reflect the majority’s view. consequently, to assert that advertisements reflect a nation’s ideas distorts reality. in truth (adv. 事實上,的確,說實在的,實際上,本質上), they mirror only the business and product ideas of companies whose goods and services are advertised and the creative ideas of advertising firms. moreover, advertisements look very much the same in all countries. western and eastern alike. does this suggest that all nations have essentially identical ideas? certainly not.

in sum, the few true ideas we might see in advertisements are those of only a few business concerns and interest groups; they tell us little about the ideas of a nation as a whole.

33. “people are likely to accept as a leader only someone who has demonstrated an ability to perform the same tasks that he or she expects others to perform.”discuss the extent to which you agree or disagree with the opinion stated above. support your views with reasons and/or examples from your own experience, observations, or reading.

people are more likely to accept the leadership of those who have shown they can perform the same tasks they require of others. my reasons for this view involve the notions of respect and trust.

it is difficult for people to fully respect a leader who cannot, or will not, do what he or she asks of others. president clinton’s difficulty in his role as commander-in-chief (n. 總司令) serves as a fitting and very public example. when clinton assumed this leadership position, it was well known that he had evaded military service during the vietnam conflict. military leaders and lower-level personnel alike made it clear that they did not respect his leadership as a result. contrast the clinton case with that of a business leader such as john chambers, ceo of cisco systems, who by way of his training and experience as a computer engineer earned the respect of his employees.

it is likewise difficult to trust leaders who do not have experience in the areas under their leadership. the clinton example illustrates this point as well. because president clinton lacked military experience, people in the armed forces found it difficult to trust that his policies would reflect any understanding of their interests or needs. and when put to the test, he undermined their trust to an even greater extent with his naive and largely bungled attempt to solve the problem of gays (<美俚> 同性戀者, 尤指男性同性者) in the military. in stark contrast, president dwight eisenhower inspired nearly devotional trust as well as respect because of his role as a military hero in world war ii.

in conclusion, it will always be difficult for people to accept leaders who lack demonstrated ability in the areas under their leadership. initially, such leaders will be regarded as outsiders, and treated accordingly. moreover, some may never achieve the insider status that inspires respect and trust from those they hope to lead.

34. “all citizens should be required to perform a specified amount of public service. such service would benefit not only the country as a whole but also the individual participants.”discuss the extent to which you agree or disagree with the opinion stated above. support your views with reasons and/or examples from your own experience, observations, or reading.

the potential benefits of mandatory public service must be weighed against administrative problems and concerns about individual liberty. on balance (adv. 總而言之), the costs to a nation and to the participants would probably exceed the benefits.

admittedly, a colorable (adj.似是而非的) argument can be made for mandatory public service. it would help alleviate “free-rider” problems, where those who do not contribute benefit from the efforts of those who do. it would mitigate pressing social problems—with education, public health and safety, and the environment. it might instill in participants a sense of civic duty, community, and individual responsibility. finally, it has worked on a smaller scale, particularly in urban areas, where renewal projects succeed in making communities safer, healthier, and more prosperous.

far more compelling, however, are the arguments against mandatory public service. first, who would make assignments and decide what projects are worthwhile, and how would compliance be assured? resolving enforcement issues would require government control, in turn requiring increased taxes and/or cuts in other social programs, thereby nullifying the benefits of mandatory public service. second, a mandatory system would open the floodgates to incompetence and inexperience. finally, the whole notion seems tantamount to communism insofar as each citizen must contribute, according to his or her ability, to a strong state. modern history informs us that such systems do not work. one could argue that mandatory public service is simply a tax in the form of labor rather than dollars. however, compulsory labor smacks (v. 帶有..風味) of involuntary servitude, whereas financial taxes do not.

in conclusion, logistical and philosophical barriers to mandating public service outweigh its potential benefits for the nation as well as for participants.

35. “business relations are infected through and through with the disease of short-sighted motives. we are so concerned with immediate results and short-term goals that we fail to look beyond them.”assuming that the term “business relations” can refer to the decisions and actions of any organization—for instance, a small family business, a community association, or a large international corporation—explain the extent to which you think that this criticism is valid. in your discussion of the issue, use reasons and/or examples from your own experience, your observation of others, or your reading.

i agree with the speaker that decisions and actions of businesses are too often “infected” by short-sighted motives. admittedly, attention to immediate results and short-term goals may be critical, and healthy, for survival of a fledgling company. however, for most established businesses, especially large corporations, failure to adequately envision the long-term implications of their actions for themselves and for others is all-too common and appropriately characterized as a “disease.”

the business world is replete with evidence that companies often fail to envision the long-term implications of their actions for themselves. businesses assume excessive debt to keep up with booming business, ignoring the possibility of a future slowdown and resulting forfeiture or bankruptcy. software companies hastily develop new products to cash in on (v. 靠..賺錢, 乘機利用) this year’s fad, ignoring bugs and glitches in their programs that ultimately drive customers away. and manufacturers of inherently dangerous products cut safety corners (concern?) to enhance short-term profits, failing to see the future implications: class action liability suits, criminal sanctions, and shareholder revolts.

similarly, businesses fail to see implications of their actions for others. motivated only by the immediate bottom line, movie studios ignore the deleterious effects that movie violence and obscenity may have on their patrons and on the society at large. captains of the energy industry pay lip service to environmental ramifications of unbridled energy use for future generations, while their real concern is with ensuring near-term dependence on the industry’s products or services. and manufacturers of dangerous products do a long-term disservice to others, of course, by cutting corners in safety and health.

in sum, i think the criticism that businesses are too concerned with immediate results and not concerned enough with the long-term effects of their actions and decisions is for the most part a fair assessment of modern-day business.

36. “businesses and other organizations have overemphasized the importance of working as a team. clearly, in any human group, it is the strong individual, the person with the most commitment and energy, who gets things done.”discuss the extent to which you agree or disagree with the opinion stated above. support your views with reasons and/or examples from your own experience, observations, or reading.

the relationship between teamwork and individual strength, energy, and commitment is complex; whether they operate in a complementary or antagonistic manner depends on: (1) the goals toward which the traits are directed, (2) the degree of emphasis on teamwork, and (3) the job of the individual within an organization.

a person’s ability to work effectively in a team is not in consistent per se with personal strength, energy, and commitment. if exercised in a self-serving manner—for example, through pilfering or back stabbing—these traits can operate against the organization. conversely, if directed toward the firm’s goals, these traits can motivate other team members, thereby advancing common goals. world war ii generals patton and rommel understood this point and knew how to bring out the best individual qualities in their troops, while at the same time instilling a strong sense of team and common purpose.

nevertheless, over-emphasizing teamwork can be counterproductive for an organization. a successful team requires both natural leaders and natural followers; otherwise, a team will accomplish little. undue emphasis on teamwork may quell initiative among natural leaders, thereby thwarting team goals. also, teamwork can be overemphasized with a commissioned sales force of highly competitive and autonomic individuals. overemphasis on teamwork here might stifle healthy competition, thereby defeating a firm’s objectives. in other organizational areas, however, teamwork is critical. for example, a product-development team must progress in lock-step (鎖步) fashion toward common goals, such as meeting a rollout (: the public introduction of a new aircraft; broadly: the widespread public introduction of a new product) deadline.

in sum, individual strength, commitment, and energy can complement a strong team approach; as long as individual autonomy is not undermined, all can operate in a synergistic manner to achieve an organization’s goals.

37. “since science and technology are becoming more and more essential to modern society, schools should devote more time to teaching science and technology and less to teaching the arts and humanities.”discuss the extent to which you agree or disagree with the opinion stated above. support your views with reasons and/or examples from your own experience, observations, or reading.

because scientific knowledge is increasingly important in our technological world and in the practical world of jobs and careers, schools should devote sufficient time to teaching mathematics and science. this is not to say, however, that schools should devote less time to the arts or humanities. to the contrary, in a technological age the study of arts and humanities is probably more important than ever—for three reasons.

first of all, studying the arts and humanities can help students become better mathematicians and scientists. for example, recent studies of cognitive development show that studying music at an early age can strengthen a child’s later grasp of mathematics. and understanding philosophical concepts has helped scientists recognize their own presuppositions, and frame their central questions more accurately.

secondly, studying the creative and intellectual achievement of others helps inspire our own creativity and intellectual questioning. this is particularly important in an era dominated by technology, where we run a serious risk of becoming automatons who fit neatly into the efficient functioning of some system.

finally, technology is valuable as an efficient means to our important goals. but neither technology, nor the science on which it is founded, decides which goals are best, or judges the moral value of the means we choose for their attainment. we need the liberal arts (文科) to help us select worthwhile ends and ethical means.

in conclusion, schools should not devote less time to the arts and humanities. these areas of study augment and enhance learning in mathematics and science, as well as helping to preserve the richness of our entire human legacy while inspiring us to further it. moreover, disciplines within the humanities provide methods and contexts for evaluating the morality of our technology and for determining its proper direction.

38. “courtesy is rapidly disappearing from everyday interactions, and as a result, we are all the poorer for it.”from your perspective, is this an accurate observation? why or why not? explain, using reasons and/or examples from your own experience, observations, or reading.

the speaker claims that simple courtesy and good manners are disappearing from modern life, and that the quality of our lives is therefore deteriorating. while i do encounter frequent instances of discourtesy and bad manners, i also encounter many instances of the opposite behavior. for this reason, and because negative experiences tend to be more memorable and newsworthy (adj. 有報導價值的), i find the speaker’s claim to be dubious.

most people encounter multiple instances of ordinary courtesy and good manners every day—simple acts such as smokers asking whether anyone minds if they light up, people letting others with fewer items ahead in grocery-store lines, and freeway drivers switching lanes to accommodate faster drivers or those entering via on-ramps. admittedly, most people also encounter discourtesy or poor manners on a daily basis—people using obscene language in public places where young children are present, and business associates intentionally ignoring phone calls, to name a few. however, such acts do not prove that good manners and courtesy are disappearing; they simply show that both courtesy and discourtesy abound in everyday life. thus the claim that courtesy and good manners are disappearing grossly (adv. 非常, 粗, 很) distorts reality.

another reason that the claim is suspect is that we tend to remember negative encounters with people more so than positive ones, probably because bad experiences tend to be more traumatic and sensational, if not more interesting to talk about. the news stories that the media chooses to focus on certainly support this rationale. however the fact that we remember, hear about, and read about discourtesy more than about courtesy shows neither that discourtesy is increasing nor that courtesy is decreasing. it simply shows that negative experiences leave stronger impressions and tend to be more sensational. in fact, i suspect that if one were to tally up (vt.總結,加一) one’s daily encounters with both types of behavior, one would conclude that good manners and courtesy are far more prevalent than the opposite behavior.

in conclusion, the speaker’s claim that common courtesy and good manners are disappearing is not born out by everyday experience. i suspect the speaker has failed to consider that negative experiences leave stronger impressions on our memory and are more interesting to relate (講;敘述) to others than positive ones.

39. “it is difficult for people to achieve professional success without sacrificing important aspects of a fulfilling personal life.”discuss the extent to which you agree or disagree with the opinion stated above. support your views with reasons and/or examples from your own experience, observations, or reading.

are professional success and a fulfilling personal life mutually exclusive? probably not, although it is more difficult today to achieve both.

undeniably, today’s professionals must work long hours to keep their heads above water (keep one’s heads above water: v. 免遭滅頂之災, 不負債), let alone to get ahead in life financially. this is especially true in japan, where cost of living, coupled with corporate culture, compel professional males to all but (adv. 幾乎, 差一點) abandon their families and literally to work themselves to death. while the situation here in the states (united states) may not be as critical, the two-income family is now the norm, not by choice but by necessity.

however, our society’s professionals are taking steps to remedy the problem. first, they are inventing ways—such as job sharing and telecommuting (遠程交換,遠程辦公)—to ensure that personal life does not take a backseat (n. 后座, 次要位置) to career. second, they are setting priorities and living those hours outside the workplace to the fullest. in fact, professional success usually requires the same time-management skills that are useful to find time for family, hobbies, and recreation. one need only look at the recent american presidents—clinton, bush, reagan, and carter—to see that it is possible to lead a balanced life which includes time for family, hobbies, and recreation, while immersed in a busy and successful career. third, more professionals are changing careers to ones which allow for some degree of personal fulfillment and self-actualization (n. 自我實現,自我潛力充分發揮). besides, many professionals truly love their work and would do it without compensation, as a hobby. for them, professional fulfillment and personal fulfillment are one and the same (n. 同一個, 完全一回事).

in conclusion, given the growing demands of career on today’s professionals, a fulfilling personal life remains possible by working smarter, by setting priorities, and by making suitable career choices.

40. “with the increasing emphasis on a global economy and international cooperation, people need to understand that their role as citizens of the world is more important than their role as citizens of a particular country.”discuss the extent to which you agree or disagree with the opinion stated above. support your views with reasons and/or examples from your own experience, observations, or reading.

with the growth of the global economy and the need for international cooperation, every human being has assumed a role as citizen of the world. does this mean that our roles as citizens of our respective nations are thereby superseded by our role as world citizens, as the speaker suggests? not at all. good citizenship at one level is often compatible with good citizenship at another. in fact, being a good citizen in one social domain can help one be a better citizen in another.

good global citizenship is not incompatible with good citizenship at other levels. consider, for example, one’s efforts as a citizen to preserve the natural environment. one particular person might, for example: (1) lobby legislators to enact laws preserving an endangered redwood forest, (2) campaign for nationally-elected officials who support clean air laws, and (3) contribute to international rainforest (n. 雨林) preservation organizations. this one person would be acting consistently as a citizen of community, state, nation and world.

admittedly, conflicting obligations sometimes arise as a result of our new “dual” citizenship. for example, a u.s. military official with an advisory role in a united nations peace-keeping force might face conflicting courses of action—one that would secure u.s. military interests, and another that would better serve international interests. however, the fact that such a conflict exists does not mean that either action is automatically more obligatory—that is, that one’s role as either u.s. citizen or world citizen must invariably supersede the other. instead, this situation should be resolved by carefully considering and weighing the consequences of each course of action.

moreover, being a good citizen in one social context can often help one be a better citizen in another. for example, volunteering to help underprivileged children in one’s community might inspire one to work for an international child-welfare organization. and inculcating civic values—such as charity and civic pride—may give rise to personal traits of character that transfer to all social domains and contexts.

in sum, although our “dual” citizenship may at times lead to conflicts, one role need not automatically take precedence over the other. moreover, the relationship between the two roles is, more often than not, a complementary one—and can even be synergistic.

41. “the best way to preserve the natural environment is to impose penalties—whether fines, imprisonment, or other punishments—on those who are most responsible for polluting or otherwise damaging it.”discuss the extent to which you agree or disagree with the opinion expressed above. support your point of view with reasons and/or examples from your own experience, observations, or reading.

imposing heavy penalties on those who pollute or destroy the environment is one way to preserve our environment. but it is not the only way; nor is it the best way. penalties may elicit grudging compliance, but other approaches—those that instill a sense of genuine commitment—are likely to be more effective in the long term.

admittedly, motivating compliance with environmental regulations by way of penalties will serve environmental goals up to a point. the deterrent effect of these remedies cannot be denied. yet it should not be overstated. some businesses may attempt to avoid punishment by concealing their activities, bribing (lobbying) legislators to modify regulations, or moving operations to jurisdictions (n. 許可權,管轄區域) that allow their environmentally harmful activities. others might calculate the trade-off (交易, 平衡) between accepting punishment and polluting, budget in advance for anticipated penalties, then openly violate the law. my intuition is that this practice is a standard operating mode among some of our largest manufacturers.

a better way to ensure environmental protection is to inculcate a sense of genuine commitment into our corporate culture—through education and through shareholder involvement. when key corporate executives become committed to values, the regulations associated with those values become a codification of conscience rather than obstacles to circumvent. the machinations and maneuverings described earlier will thereby be supplanted by thoughtful concern about all the implications of one’s actions. moreover, commitment-driven actions are likely to benefit the environment over and above what the law requires. for example, while a particular regulation might permit a certain amount of toxic effluents, businesses committed to environmental protection may avoid harmful emissions altogether.

instilling a genuine sense of commitment through education and shareholder action is not just a better approach in theory, it is also less costly overall than a compliance-driven approach. regulatory systems inherently call for legislative committees, investigations and enforcement agencies, all of which adds to the tax burden of the citizens whom these regulations are designed to protect. also, delays typically associated with bureaucratic regulation may thwart the purpose of the regulations, since environmental problems can quickly become very grave.

in sum, penalties for violating environmental-protection laws are essentially expensive band-aids (adj. 補綴的, 權宜的). a commitment-based approach, involving education and shareholder activism, can instill in corporate culture a sense an environmental conscience, resulting in far more effective environmental protection.

42. “scientists are continually redefining the standards for what is beneficial or harmful to the environment. since these standards keep shifting, companies should resist changing their products and processes in response to each new recommendation until those recommendations become government regulations.”discuss the extent to which you agree or disagree with the opinion stated above. support your views with reasons and/or examples from your own experience, observations, or reading.

the speaker argues that because scientists continually shift viewpoints about how our actions affect the natural environment, companies should not change their products and processes according to scientific recommendations until the government requires them to do so. this argument raises complex issues about the duties of business and about regulatory fairness and effectiveness. although a wait-and-see (adj. 觀望的) policy may help companies avoid costly and unnecessary changes, three countervailing considerations compel me to disagree overall with the argument.

first, a regulatory system of environmental protection might not operate equitably. at first glance, a wait-and-see response might seem fair in that all companies would be subject to the same standards and same enforcement measures. however, enforcement requires detection, and while some violators may be caught, others might not. moreover, a broad regulatory system imposes general standards that may not apply equitably to every company. suppose, for example, that pollution from a company in a valley does more damage to the environment than similar pollution from a company on the coast. it would seem unfair to require the coastal company to invest as heavily in abatement or, in the extreme (adv. 非常, 極端), to shut down the operation if the company cannot afford abatement measures.

secondly, the argument assumes that the government regulations will properly reflect scientific recommendations. however, this claim is somewhat dubious. companies with the most money and political influence, not the scientists, might in some cases dictate regulatory standards. in other words, legislators may be more influenced by political expediency and campaign pork (pork: government money, jobs, or favors used by politicians as patronage) than by societal concerns.

thirdly, waiting until government regulations are in place can have disastrous effects on the environment. a great deal of environmental damage can occur before regulations are implemented. this problem is compounded whenever government reaction to scientific evidence is slow. moreover, the epa (environmental protection agency 美國環保署) might be overburdened with its detection and enforcement duties, thereby allowing continued environmental damage by companies who have not yet been caught or who appeal penalties.

in conclusion, despite uncertainty within the scientific community about what environmental standards are best, companies should not wait for government regulation before reacting to warnings about environmental problems. the speaker’s recommended approach would in many cases operate inequitably among companies: moreover, it ignores the political-corruption factor as well as the potential environmental damage resulting from bureaucratic delay.

43. “the most important reason for studying history is not that knowledge of history can make us better people or a better society but that it can provide clues to solving the societal problems that we face today.”discuss the extent to which you agree or disagree with the opinion expressed above. support your point of view with reasons and/or examples from your own experience, observations, or reading.

examining history makes us better people insofar as (vt.在..範圍內,在..限度內to the extent or degree that) it helps us to understand our world. it would seem, therefore, that history would also provide useful clues for dealing with the same social ills that have plagued societies throughout history. on balance, however, the evidence suggests otherwise.

admittedly, history has helped us learn the appropriateness of addressing certain issues, particularly moral ones, on a societal level. attempts to legislate morality invariably fail, as illustrated by prohibition (<美>禁酒令) in the 1930s and, more recently, failed federal legislation to regulate access to adult material via the internet. we are slowly learning this lesson, as the recent trend toward legalization of marijuana for medicinal purposes and the recognition of equal rights for same-sex partners both demonstrate.

however, the overriding lesson from history about social ills is that they are here to stay. crime and violence, for example, have troubled almost every society. all manner of reform, prevention, and punishment have been tried. today, the trend appears to be away from reform toward a “tough-on-crime” approach. is this because history makes clear that punishment is the most effective means of eliminating crime? no; rather, the trend merely reflects current mores, attitudes, and political climate. also undermining the assertion that history helps us to solve social problems is the fact that, despite the civil-rights efforts of martin luther king and his progenies, the cultural gap today between african-americans and white americans seems to be widening. it seems that racial prejudice is here to stay. a third example involves how we deal with the mentally ill segment of the population. history reveals that neither quarantine, nor treatment or accommodation solves the problem, only that each approach comes with its own tradeoffs.

to sum up, while history can teach us lessons about our social problems, more often than not the lesson is that there are no solutions to many social problems—only alternate ways of coping with them.

44. “all companies should invest heavily in advertising because high-quality advertising can sell almost any product or service.”discuss the extent to which you agree or disagree with the opinion expressed above. support your point of view with reasons and/or examples from your own experience, observations, or reading.

the speaker claims that high-quality ads can sell almost anything, and that companies should accordingly invest heavily in such advertising. i agree that the quality of an ad can in some instances play a pivotal role in a product’s success or failure in the marketplace. however, the speaker overgeneralizes, for advertising is far more critical in some businesses and for some products than for others.

certain types of businesses benefit greatly from investing in high-quality advertising. fledgling companies, for example, may require an extensive top-notch (adj. 拔尖的) advertising campaign to achieve the name recognition that older competitors already enjoy. even established companies may need an expensive ad campaign when introducing new products or venturing into new markets. companies selling products that are no utilitarian value perhaps stand to gain the most from an extensive high-quality advertising effort. consider, for example, the kinds of products that are marketed by means of the most extensive and expensive advertising: beer, cigarettes, soft drinks, and cosmetics. none of these products has any utility. their success depends on consumers’ fickle tastes, their emotions, and their subjective perceptions. accordingly, influencing consumer attitudes through popular and appealing ads is about the only way to increase sales of such products.

in some industries, however, substantial investment in high-quality advertising simply does not make sense from a cost-effectiveness (節省成本的) viewpoint. pharmaceutical companies, for example, might be better off limiting their advertising to specialized publications, and focus instead on other kinds of promotional programs, such as the distribution of free samples. and widespread, flashy advertising would probably have a limited effect on overall sales for companies such as deere and caterpillar, whose name recognition and long-standing reputations for quality products are well established and whose customers are unlikely to be swayed by sensational ads.

in sum, the speaker overgeneralizes. not all companies have an equal need to invest heavily in high-quality advertising. companies with new products and products that have little utility stand to benefit (stand to gain: v.一定獲利) most from expensive, high-quality advertising. but other companies, especially those whose customers are businesses (a commercial or sometimes an industrial enterprise; also: such enterprises *the business district*) rather than consumers, would be better off focusing on product quality and reputation, not on sensational advertising.

45. “the most effective way for a businessperson to maximize profits over a long period of time is to follow the highest standards of ethics.”discuss the extent to which you agree or disagree with the opinion stated above. support your views with reasons and/or examples from your own experience, observations, or reading.

the speaker claims that following high ethical standards is the best way to maximize profits in the long run. however, this claim seems to be more of a normative statement than an empirical observation. the issue is more complex than the speaker suggests. in my observation, the two objectives at times coincide but at other times conflict.

in many ways behaving ethically can benefit a business. ethical conduct will gain a company good reputation that earns repeated business. treating suppliers, customers and others fairly is likely to result in their reciprocating. finally, a company that treats its employees fairly and with respect will gain their loyalty which, in turn, usually translates into higher productivity.

on the other hand, taking the most ethical course of action may in many cases reduce profits, in the short run and beyond. consider the details of a merger in which both firms hope to profit from a synergy (n.最佳協合作用,企業合併後的協力優勢) gained thereby. if the details of the merger hinge on (v. 靠..轉動, 以..為轉移) the ethical conviction that as few employees as possible should lose their jobs, the key executives may lose sight of the fact that a leaner, less labor-intensive organization might be necessary for long-term survival. thus, undue concern with ethics in this case would results in lower profits and perhaps ultimate business failure.

this merger scenario points out a larger argument that the speaker misses entirely-that profit maximization is per se the highest ethical objective in private business. why? by maximizing profits, businesses bestow a variety of important benefits on their community and on society: they employ more people, stimulate the economy, and enhance healthy competition. in short, the profit motive is the key to ensuring that the members of a free market society survive and thrive. while this argument might ignore implications for the natural environment and for socioeconomic (of, relating to, or involving a combination of social and economic factors) justice, it is a compelling argument nonetheless.

thus the choice to follow high ethical standards should not be made by thinking that ethical conduct is profitable. while in some cases a commitment to high ethical standards might benefit a company financially, in many cases it will not. in the final analysis, businesses might best be advised to view their attempts to maximize profits as highly ethical behavior.

46. businesses are as likely as are governments to establish large bureaucracies, but bureaucracy is far more damaging to a business than it is to a government.discuss the extent to which you agree or disagree with the opinion expressed above. support your point of view with reasons and/or examples from your own experience, observations, or reading.

contrary to the statement’s premise, my view is that businesses are less likely than government to establish large bureaucracies, because businesses know that they are more vulnerable than government to damage resulting from bureaucratic inefficiencies. my position is well supported by common sense and by observation.

first, public administrators lack the financial incentives to avoid bureaucratic waste. in contrast, inefficiencies in a private corporation will reduce profits, inflicting damage in the form of job cuts, diminishing common-stock value, and reducing employee compensation. these are ample incentives for the private firm to minimize bureaucratic waste.

second, there is almost no accountability among government bureaucrats. the electorate’s voting power is too indirect to motivate mid-level administrators, whose salaries and jobs rarely depend on political elections. in contrast, private corporations must pay strict attention to efficiency, since their shareholders hold an immediate power to sell their stock, thereby driving down the company’s market value.

third, government is inherently monopolistic, large, and unwieldy; these features breed bureaucracy. admittedly some corporations rival state governments in size. yet even among the largest companies, the profit motive breeds a natural concern for trimming waste, cutting costs, and streamlining operations. even virtual monopolies strive to remain lean and nimble in order to maintain a distance from upstart competitors. when government pays lip service to efficiency, shrewd listeners recognize this as political rhetoric designed only to pander to the electorate.

in the final analysis, financial incentives, accountability, and competition all distinguish private business from government, both in terms of their likelihood of establishing large bureaucracies and in terms of the damage that these bureaucracies can inflict on the organization.

47. the primary responsibility for preventing environmental damage belongs to government, not to individuals or private industry.discuss the extent to which you agree or disagree with the opinion expressed above. support your point of view with reasons and/or examples from your own experience, observations, or reading.

the responsibility for preventing environmental damage should be shared by government, private industry and individuals alike. the primary obligation, however, belongs to individuals. moreover, within organizations like the government or a corporation, responsibility should be increasingly distributed to individuals according to level of authority.

the primary obligation to preserve the environment belongs to individuals for the reason that assigning responsibility to a government or corporation is problematic. this is because abstract entities like these do not fulfill the usual criteria for being responsible. an entity can shoulder (vt. 肩負, 承當) responsibility only if it can be held accountable for its actions. furthermore, being held accountable for an action requires that the entity act willingly and on the basis of conscious intentions. but governments and businesses are abstractions, having neither will nor consciousness beyond that of the individuals within them.

still, we can make some sense of treating corporations and governments as if they were individuals. they are individuals under the law, and therefore subject to laws, penalties, and lawsuits. they can even be identified as beneficiaries in wills. nevertheless, when responsibility is vaguely allocated to abstract entities like governments or corporations, it becomes easy for those within such organizations to cover individual actions that result in devastation to the environment. consider the famous case of the exxon valdez accident and oil spill off the alaskan coast. while it was easy to single out captain hazelwood and determine his blameworthiness the night of the mishap, it was not so easy to identify those responsible at higher levels. someone was responsible for hiring hazelwood; others should have known about his drinking or other job-related problems. thus when we do assign responsibility to governments or business organizations, it must be clearly distributed to individuals in relevant lines of authority (n.權力系統) within the organization.

in conclusion, individuals are mainly responsible for protecting the environment. and while it makes some sense in a vague way to talk about the similar responsibilities of government and industry, in the end such obligations will belong to individuals within them. therefore, some individuals will assume greater shares of responsibility for the environment, since they act in positions of authority on behalf of government or industry.

48. in matching job candidates with job openings (職位空缺,職務空缺,職工空缺,就業機會), managers must consider not only such variables as previous work experience and educational background but also personality traits and work habits, which are more difficult to judge.what do you consider essential in an employee or colleague? explain, using reasons and/or examples from your work or worklike experiences, or from your observations of others.

in the hiring process, it is more difficult to assess personality and work habits than to determine work experience and educational background. even so, it is important to try and judge the less quantifiable (adj.可以計量的) characteristics of a prospective colleague or employee—such as honesty, reliability, creativity, self-motivation, and the capacity to get along and work well with others. if it doesn’t seem obvious that these are important qualities in a coworker, then consider the alternatives.

first of all, dishonest or unreliable workers harm an organization in many ways. dishonest employees impose costs on a company whether they steal on the grand or small scale; just taking a few days of unwarranted sick leave here and there (from time to time) can add up to significant lost productivity. and lying about progress on a project can result in missed deadlines and even lost contracts. unreliability works the same way; if an employee cannot meet deadlines or fails to appear at important meetings, the organization will suffer accordingly.

in addition, coworkers who lack motivation or creativity take some of the life out of an organization. to the extent that employees simply plug along, the company will be less productive in contrast, employees who have imagination and the motivation to implement ideas are productive and can spark those around them to greater achievement.

finally, employees who cannot get along with or work well with others can as well be detrimental to the organization. the mere presence of a troublemaker is disruptive; moreover, the time such people spend on petty disagreements is time away from getting the job done successfully. in addition, those who cannot smoothly coordinate their efforts with others will end up making things more difficult for everyone else.

in conclusion, it may not be easy to judge the personality traits and work habits of prospective employees, but it certainly is worth the effort to try. having coworkers who are honest, reliable, creative, self-motivated, compatible with one another and good team players will greatly enhance everyone’s work life, and benefit an organization in the most significant way—with greater productivity.

49. “ask most older people to identify the key to success, and they are likely to reply ‘hard work.’ yet, i would tell people starting off in a career that work in itself (本身;實質上) is not the key. in fact, you have to approach work cautiously—too much or too little can be self-defeating.”to what extent do you agree or disagree with this view of work? develop your position by using reasons and/or examples from your reading, experience, or observations.

there is no doubt that hard work contributes to success. yet a person can work awfully (adv. 非常, 很, 十分) hard and still achieve very little. in order to bring about success, hard work has to be directed by clear goals and the knowledge of how to reach them. moreover, imagination, intelligence and persistence can be equally important to success.

individual success is gauged by the extent to which one reaches his important personal goals. and it takes careful planning to set goals and discover the best means of realizing them. before hard work even begins, therefore, considerable time and effort should be spent on planning.

intelligence and imagination play important roles in planning. imagination helps one to envision new solutions to problems, and new means by which to achieve goals. intelligence helps one research and critically evaluate the possibilities that imagination has provided. together, imagination and intelligence can even help one avoid certain kinds of hard work, by producing more efficient ways to accomplish goals.

finally, persistence is crucial to success. sometimes, rewards do not come quickly—even when one carefully sets the goals, creatively and intelligently plans ways to achieve them, and works hard according to plan. tradition has it, for example, that thomas edison made thousands of attempts to create a light bulb before he finally succeeded. in the face of countless failures, he refused to quit. in fact, he considered each failure a successful discovery of what not to do!

in conclusion, it is true that there is no substitute for hard work. but hard work is an ingredient of success, and not the key. hard work can produce real accomplishment only if it is directed by a plan involving some idea of one’s goals and the means to them. and a good plan, as well as its successful implementation, requires imagination, intelligence, and persistence.

50. how far should a supervisor go in criticizing the performance of a subordinate? some highly successful managers have been known to rely on verbal abuse and intimidation.do you think that this is an effective means of communicating expectations? if not, what alternative should a manager use in dealing with someone whose work is less than satisfactory? explain your views on this issue. be sure to support your position with reasons and/or examples from your own experience, observations, or reading.

unsatisfactory employee performance demands appropriate response from a manager or supervisor. the question is what is appropriate? some managers might claim that verbal abuse and intimidation are useful in getting employees to improve. while this may be true in exceptional cases, my view is that the best managerial responses generally fulfill two criteria: (1) they are respectful; and (2) they are likely to be the most effective in the long run.

treating employees with respect is important in all contexts. respect, in the most basic sense, involves treating a person as equal in importance to oneself. for a manager or supervisor, this means recognizing that occupying a subordinate position does not make a worker a lesser person. and it means treating subordinates as one would want to be treated—honestly and fairly. using threats or verbal abuse to elicit better employee performance amounts to treating a worker like the office copy machine—as an object from which to get what one wants.

moreover, while verbal abuse might produce the desired reaction at a particular time, it is likely to backfire later. nobody likes to be abused or intimidated. if such methods were the general practice in an office or division, overall morale would probably be low. and it is unlikely that employees would give 100 percent to managers who so obviously disregarded (treat as unworthy of regard or notice) them.

more beneficial in the long run would be careful but clear feedback to the worker about specific deficiencies, along with ideas and encouragement about improvement. in addition, supervisors should allow employees to explain the problem from their point of view and to suggest solutions. of course, a supervisor should never mislead a subordinate into thinking that major problems with work performance are insignificant or tolerable. still, an honest message can be sent without threats or assaults on self-esteem.

in conclusion, supervisors should avoid using verbal abuse and threats. these methods degrade subordinates, and they are unlikely to produce the best results in the long run. it is more respectful, and probably more effective overall, to handle cases of substandard work performance with clear, honest and supportive feedback.

51. “the presence of a competitor is always beneficial to a company. competition forces a company to change itself in ways that improve its practices.”discuss the extent to which you agree or disagree with the opinion stated above. support your views with reasons and/or examples from your own experience, observations, or reading.

we ordinarily think, as the speaker does here, that the presence of competition is always healthy for business because it sparks efficiency and innovation. while competition is generally good for business in these respects, the speaker here ignores the many problems that can accrue from attempting to keep up with or beat a competitor, and that may be decidedly detrimental to a business.

admittedly, competition among businesses can occasion (: bring about, cause) all sorts of improved practices. the need for competitive product pricing can motivate effective micro-management of production and marketing costs. competition for market share can spark invention and innovation in product design that lead to the cutting edge of technology. external competition is known to inspire team spirit within an organization, thereby yielding greater productivity. and competition can challenge a company to streamline operations, thereby improving efficiency.

but taken too far, attempting to keep up with or beat competitors brings about detrimental results for a company. in some cases, companies compromise product quality by switching to inferior, less expensive materials in order to keep prices competitive. other times, plant managers ignore important employee-safety measures just to save money. and companies are even known to trade off consumer safety in the interest of competition. perhaps the paradigmatic case involved the ford pinto, where ford management rejected an inexpensive retrofit (更新,改進to install [new or modified parts or equipment] in something previously manufactured or constructed) that would have saved hundreds of lives in rear-end collisions, solely in order to shave a few dollars off the car’s sticker price (n. 標價, 定價a manufacturer’s suggested retail price that is printed on a sticker and affixed to a new automobile), thereby enhancing the car’s competitiveness.

competition can even bring about large-scale social change that some consider undesirable. for instance, the emergence of large, efficient factory farms has resulted in the virtual disappearance of family farming in the u.s. and it isn’t clear that the factory farms always improve farming practices, in the case of the tomato, the old homegrown kind are far superior in taste and texture to the tough, underripe version that has been genetically engineered for machine picking in huge quantity.

in conclusion, competition frequently motivates changes that are beneficial in many ways. but competition is a double-edged sword that can also result in inferior or unsafe products and dangerous working conditions for employees. moreover, large competitors can swallow up smaller concerns without yielding noticeably better products or practices.

52. “successful individuals typically set their next goal somewhat—but not too much—above their last achievement. in this way, they steadily raise their level of aspiration.”in your opinion, how accurate is this statement? explain, using specific reasons and examples from your reading, your own experience, or your observation of others.

i agree generally that setting new goals in small increments above past accomplishments is a reliable path to achieving those goals. i think anyone would be hard-pressed (處境困難的,處境艱難的hard put; also: being under financial strain. hard put: barely able: faced with difficulty or perplexity *was hard put to find an explanation*) to find fault with this advice. nevertheless, in some exceptional instances, a more dramatic “leap-frog (vt. 蛙跳般地躍過)” approach may be more appropriate, or even necessary, to achieve a significant goal.

the virtues of setting goals in small, easily-attainable increments are undeniable. overwhelming challenges are reduced to readily attainable tasks. a psychological boost is afforded by each intermediate success, helping to ensure that the achiever won’t become discouraged and give up. each step in this process can raise one’s level of aspiration, and in manageable proportions that make success more likely. moreover, this approach can be used by anyone—a sedentary office worker who decides to complete the new york marathon; a paralegal (n. 律師的專職助手, 律師幫辦) who wishes to become a surgeon; or a small business owner who aspires to become ceo of a fortune 500 company.

in some instances, however, the step-by-step (逐步的) approach is not adequate. for example, many great creative achievements—in art, music, and literature—are made not by the achiever’s disciplined setting of incremental (increment: a minute increase in quantity) goals, but rather by a spontaneous flash of brilliance and intense creativity. another exception to this approach is the case of the ultra-successful actor, model, or even socialite who might suddenly leap-frog to his or her goal through serendipity. third, for those who have already achieved great things, taking baby (much smaller than the usual *baby carrots* *a baby flattop* *take two baby steps*) steps toward the next goal would only frustrate them and slow them down. suppose, for example, a recent gold medalist in the olympic games’ 100-meter sprint wishes to become a member the football franchise that won last year’s super bowl. what small, incremental accomplishments are needed to achieve his goal? none, aside from a phone call by his agent to the front office (n. 全體決策人員: the policy-making officials of an organization) of the team. admittedly, these are exceptional cases: yet they do exist.

in conclusion, setting modest but increasingly higher goals is generally good advice. yet this approach may be inappropriate or inadequate under certain exceptional circumstances.

53. “the term ‘user-friendly’ is usually applied to the trouble-free way that computer software moves people from screen to screen, function to function. however, the term can also refer to a government office, a library, public transportation, or anything designed to provide information or services in an easy, friendly way. just as all societies have many striking examples of user-friendly services, so do they abound in examples of user-unfriendly systems.” identify a system or service that you have found to be either “user-friendly” or “user-unfriendly.”discuss, from the user’s perspective, in what way the system either is or is not easy to use and explain the consequences or effect of such a system.

if one focuses on systems such as financial services and telecommunications, where emerging technologies have the greatest impact, one sees increasing user-friendliness. however, in other systems—public and private alike—inefficiencies, roadblocks (n. 路障,問題,困難,難題,難關,障礙), and other “unfriendly” features still abound. one such example is the u.s. health-care delivery system.

to a large extent, the user-unfriendly nature of health-care delivery stems from its close tie to the insurance industry. service providers and suppliers inflate prices, knowing that insurance companies can well afford to pay by passing on inflated costs to the insured. hospital patients are often discharged prematurely merely because insurance fails to cover in-patient care beyond a certain amount or duration. in the extreme (極端,非常,達於極點), patients are sometimes falsely informed that they are well or cured, just so that the facility can make room for insured patients. insurance providers reject claims and coverage intentionally and in bad faith (不誠實;不打算履行諾言的) when the insured has suffered or is statistically likely to suffer from a terminal (approaching or close to death: being in the final stages of a fatal disease *a terminal patient*) or other long-term—and costly—illness. insurance companies also impose extreme coverage exceptions for pre-existing conditions. both tactics are designed, of course, to maximize insurance company profits at the expense of the system’s user. finally, new medical technologies that provide more effective diagnosis and treatment are often accessible only to the select (adj.精選的:exclusively or fastidiously chosen often with regard to social, economic, or cultural characteristics) few who can afford the most comprehensive insurance coverage.

the consequences of these user-unfriendly features can be grave indeed for the individual, since this system relates directly to a person’s physical wellbeing and very life. for example, when a claim or coverage is wrongfully denied, lacking financial resources to enforce their rights, an individual customer has little practical recourse. the end result is to render health care inaccessible to the very individuals who need it most. these user-unfriendly features can be deleterious on a societal scale as well. an unhealthy populace is an unproductive one. also, increased health-care costs place an undue burden on bread-winning (breadwinner: 養家活口的人, 負擔家計的人) adults who feel the squeeze (a financial pressure caused by narrowing margins or by shortages) of caring for aging parents and for children. finally, these features foster a pervasive distrust of government, big business, and bureaucracy.

in sum, today’s “point-and-click (點擊)” paradigm inaccurately portrays the actual functionality of many systems, including our health-care delivery system, which is well-entrenched in self-interest and insensitivity to the needs of its users.

54. “popular entertainment is overly influenced by commercial interests. superficiality, obscenity, and violence characterize films and television today because those qualities are commercially successful.”discuss the extent to which you agree or disagree with this opinion. to support your position, use reasons and/or examples from your reading, your observations, or your experiences as a consumer of popular entertainment.

clearly, most popular films and television shows are superficial and/or include a certain amount of violence or obscenity. just as clearly, popularity leads to commercial success. but can we conclude that these productions are overly influenced by commercial interests? perhaps not, since some popular films and television shows are neither superficial, obscene, nor violent. closer scrutiny, however, reveals that most such productions actually support, not disprove, the thesis that commercial interests dictate movie and television content. (哪有必要作這樣的二次轉折)

one would-be (自稱自許的) threat to the thesis can be found in lower-budget independent films, which tend to focus more on character development and topical social issues than on sensationalism. recently, a few such films have supplanted hollywood’s major studio productions as top box-office (adj. 票房的) hits. does this mean that profit potential no longer dictates the content of films? no; it simply suggests that the tastes and preferences of the movie-going public are shifting.

a second ostensible challenge to the thesis can be found in companies such as disney, whose productions continue to achieve great popularity and commercial success, without resort to an appeal to baser interests. yet it is because these productions are commercially successful that they proliferate.

the only cogent challenge to the thesis is found in perennial television favorites such as “nova,” a public television show that is neither commercially supported nor influenced. however, such shows are more in the nature of education than entertainment, and for every one program like “nova” there are several equally popular—and highly superficial—programs.

with few exceptions, then, commercial success of certain films and television shows is no accidental byproduct of popularity; it is the intentional result of producers’ efforts to maximize profits.

55. “never tell people how to do things. tell them what to do, and they will surprise you with their ingenuity.”to what extent do you agree or disagree with the opinion expressed above? explain your point of view by giving reasons and/or examples from your own experience, observations, or reading.

i agree that supervisors should under most circumstances merely tell subordinates what to do, but not necessarily how to do it. of course, employees need adequate training in order to do a job. but beyond that, trusting employees to discover and develop their own methods for meeting a supervisor’s expectations can produce surprising rewards that outweigh any pitfalls of such an approach.

first of all, restraint in directing the how-to (adj. 解釋作法的, 指引的giving practical instruction and advice [as on a craft]) aspect of a project signals the supervisor’s confidence in an employee’s intelligence and abilities. sensing this confidence, the subordinate will often respond with his or her best work. this phenomenon lends truth to the adage that people rise to the level of what others expect from them.

secondly, by allowing a subordinate to decide how best to attain an objective, a supervisor imparts a larger share of responsibility for the project to the subordinate. this alleviates some of the burden from the supervisor, who may have more time for other tasks as a result. at the same time, when the subordinate shares in the responsibility, he or she will probably feel more accountable for how the job turns out. the result is likely to be better job performance.

thirdly, directing every step of a project often blocks a worker’s own creativity, as well as creating animosity. except in the training of a new worker with little or no experience, it would be naive and arrogant for any supervisor to assume there is one and only one best way—the supervisor’s own way—to get a job done. a bright, competent subordinate is likely to resent being led by the hand like a child. allowing employees to choose their own means and methods will spark their ingenuity in ways that enhance productivity now and in the future, and will foster goodwill and mutual respect in the workplace.

in sum, telling a subordinate how to do a job is rarely the best management approach. instead, supervisors should assign tasks without directing each step. when employees are left to choose methods for completing work, they will be bolstered by the supervisor’s trust, motivated to greater creativity and inclined to feel accountable for outcomes.

56. “the secret of business is to know something that nobody else knows.”explain what you think the above quotation means and discuss the extent to which you agree or disagree with it. support your position with relevant reasons and/or examples from your own experience, observations, or reading.

this statement is ambiguous. it could mean, literally, that business success depends on knowing more than anyone else about one’s operations, products and markets. or it could be a subtle recommendation to acquire privileged information, by whatever means, to use for one’s own advantage. i agree with the statement in the first sense. however, i strongly disagree with many implications of the second possible meaning.

it goes without saying that competitive edge in business is a function of knowledge. it is crucial to fully understand the technology and uses of one’s products; and it is prudent to micromanage (to manage with great or excessive control or attention to details) operations, knowing as much as possible about the small details that can add up to a significant economic difference. it is also prudent, and legitimate, to take every measure to protect that knowledge as trade secrets, since they often play a pivotal role in a firm’s competitiveness.

but the advice to know something that nobody else does could easily become distorted. if taken another way, the advice could recommend that one dig up (find, unearth) dirt in order to damage or discredit a rival. it could also be taken to recommend stealing trade secrets (商業秘密,行業秘密) or other inside information from a competitor in order to gain an unfair business advantage. all of these tactics are some also violate civil and criminal laws. moreover, the recommendation to find and use any information, even unfairly or illegally, can backfire. people who follow such advice risk civil liability, criminal prosecution, and the loss of an important business asset—their good reputations.

in sum, i agree with the statement up to the point that it validates detailed and even proprietary knowledge as a key to competitiveness. insofar as the statement sanctions unfair practices, however, following it would be unethical, bad for business, and damaging to the character and reputation of the perpetrator.

57. “everywhere, it seems, there are clear and positive signs that people are becoming more respectful of one another’s differences.”in your opinion, how accurate is the view expressed above? use reasons and/or examples from your own experience, observations, or reading to develop your position.

in determining whether we are becoming more respectful of one another’s differences, one must examine both overt (open to view: manifest) actions and underlying motives, as well as examining whether our differences are increasing or decreasing. the issue, therefore, is quite complex, and the answer is unclear.

disrespect for one another’s differences manifests itself in various forms of prejudice and discrimination. since the civil rights and feminist movements of the 60s and 70s, it would seem that we have made significant progress toward eliminating racial and sexual discrimination. anti-discriminatory laws in the areas of employment, housing (供給住宅,住房供給dwellings provided for people), and education, now protect all significant minority groups racial minorities and women, the physically challenged (adj. having a disability or deficiency) and, more recently, homosexuals. movies and television shows, which for better or worse (adv. 不論好壞) have become the cynosure of our cultural attention, now tout the rights of minorities, encouraging acceptance of and respect for others.

however, much of this progress is forced upon us legislative. without title 10 and its progenies (a body of followers, disciples, or successors), would we voluntarily refrain from the discriminatory behavior that the laws prevent? perhaps not. moreover, signs of disrespect are all around us today. extreme factions still rally around bigoted demagogues; the number of “hate crimes” is increasing alarmingly; and school-age children seem to flaunt a disrespect toward adults as never before. finally, what appears to be respect for one another’s differences may in fact be an increasing global homogeneity—that is, we are becoming more and more alike.

in sum, on a societal level it is difficult to distinguish between genuine respect for one another’s differences on the one hand and legislated morality and increasing homogeneity on the other. accordingly, the claim that we are becoming more respectful of one another’s differences is somewhat dubious.

58. “what is the final objective of business? it is to make the obtaining of a living—the obtaining of food, clothing, shelter, and a minimum of luxuries—so mechanical and so little time-consuming that people shall have time for other things.”— a business leader, circa 1930

explain what you think the quotation above means and discuss the extent to which you agree or disagree with the view of business it expresses. support your views with reasons and/or examples from your own experience, observations, or reading.

this quotation suggests that the ultimate purpose of business is to streamline and mechanize work, thereby minimizing it, so that people can make a living but still have time for other things in life. the assumptions behind this view of business are that the value of work is entirely instrumental, and that our work lives are distinct from the rest of our lives. i disagree with both assumptions.

admittedly, work is to a large extent instrumental in that we engage in it to provide for our needs while leaving time and resources for other activities—raising families, participating in civic life, traveling, pursuing hobbies, and so forth. and these activities normally take place away from the workplace and are distinct from our work. however, for most people, work is far more than a means to these ends. it can also be engaging, enjoyable and fulfilling in itself. and it can provide a context for expressing an important part of one’s self. however, work will be less of all these to the extent that it is streamlined and mechanized for quick disposal, as the quotation recommends. instead, our jobs will become monotonous and tedious, the work of drones. and we might become drone-like in the process.

in addition, work can to some extent be integrated with the rest of our lives. more and more companies are installing on-site daycare facilities and workout rooms. they are giving greater attention to the ambiance of the break room, and they are sponsoring family events, excursions and athletic activities for employees as never before (adv. 以前從來沒有). the notion behind this trend is that when a company provides employees with ways to fulfill outside needs and desires, employees will do better work. i think this idea has merit.

in conclusion, i admit that there is more to life than work, and that work is to some extent a means to provide a livelihood. but to suggest that this is the sole purpose of business is an oversimplification that ignores the self-actualizing significance of work, as well as the ways it can be integrated with other aspects of our lives

59. “juvenile crime is a serious social problem, and businesses must become more involved in helping to prevent it.”discuss the extent to which you agree or disagree with the opinion expressed above. support your point of view with reasons and/or examples from your own experience, observations, or reading.

juvenile delinquency is clearly a serious social problem. whether businesses must become more involved in helping to prevent the problem depends, however, on the specific business—whether it is culpable in creating the problem and whether its owners’ collective conscience calls for such involvement.

although parents and schools have the most direct influence on children, businesses nonetheless exert a strong, and often negative, influence on juveniles by way of their advertisements and of the goods they choose to produce. for example, cigarette advertisements aimed at young people, music and clothing that legitimize “gang (a group of persons working to unlawful or antisocial ends; especially: a band of antisocial adolescents)” sub-culture (次級文化), and toys depicting violence, all sanction (to give effective or authoritative approval or consent to) juvenile delinquency. in such cases perhaps the business should be obligated to mitigate its own harmful actions—for example, by sponsoring community youth organizations or by producing public-interest ads.

in other cases, however, imposing on a business a duty to help solve juvenile delinquency or any other social problem seems impractical and unfair. some would argue that because business success depends on community support, businesses have an ethical duty to give back to the community—by donating money, facilities, or services to social programs. many successful businesses—such as mrs. field’s, ben & jerry’s, and timberland—have embraced this philosophy. but how far should such a duty extend, and is it fair to impose a special duty on businesses to help prevent one specific problem, such as juvenile delinquency? moreover, businesses already serve their communities by enhancing the local tax base and by providing jobs, goods and services.

in the final analysis, while businesses are clearly in a position to influence young people, whether they should help solve juvenile delinquency is perhaps a decision best left to the collective conscience of each business.

60. “employers should have no right to obtain information about their employees’ health or other aspects of their personal lives without the employees’ permission.”discuss the extent to which you agree or disagree with the opinion stated above. support your views with reasons and/or examples from your own experience, observations, or reading.

determining whether employers should have access to personal information about employees requires that the interests of businesses in ensuring productivity and stability be weighed against concerns about equity and privacy interests. on balance (adv. 總而言之), my view is that employers should not have the right to obtain personal information about current employees without their consent.

a business’ interest in maintaining a stable, productive workforce clearly justifies right of access to certain personal information about prospective employees. job applicants can easily conceal personal information that might adversely affect job performance, thereby damaging the employer in terms of low productivity and high turnover. during employment, however, the employee’s interests are far more compelling than those of the employer, for three reasons.

first, the employer has every opportunity to monitor ongoing job performance and to replace workers who fail to meet standards, regardless of the reason for that failure. second, allowing free access to personal information about employees might open the floodgates to discriminatory promotions and salary adjustments. current federal laws—which protect employees from unfair treatment based on gender, race, and marital status, may not adequately guard against an employer’s searching for an excuse to treat certain employees unfairly. third, access to personal information without consent raises serious privacy concerns, especially where multiple individuals have access to the information. heightening this concern is the ease of access to information which our burgeoning electronic intranets make possible.

in sum, ready access to certain personal information about prospective employees is necessary to protect businesses; however, once hired, an employee’s interest in equitable treatment and privacy far outweighs the employer’s interest in ensuring a productive and stable workforce.

61. “even at its best, a government is a tremendous burden to business, though a necessary one.”discuss the extent to which you agree or disagree with the opinion expressed above. support your point of view with reasons and/or examples from your own experience, observations, or reading.

i agree with the statement insofar as government systems of taxation and regulation are, in general, a great burden to business, and i agree that government constraints are needed to prevent serious harms that would result if business were left free in the singular pursuit of profit. however, i think the speaker states the obvious and begs (a: evade, sidestep *begged the real problems* b: to pass over or ignore by assuming to be established or settled *beg the question*) the more relevant question.

is government “at best” a “tremendous burden” on business, as the speaker claims? i think one would be hard-pressed to find any small business owner or corporate ceo who would disagree. businesses today are mired in the burdens that government has imposed on them: consumer and environmental protection laws, the double-tiered (tier層, 等級) tax structure for c-corporations, federal and state securities regulations, affirmative action requirements, anti-trust laws, and so on. in focusing solely on these burdens, one might well adopt a strict laissez faire view that if business is left free to pursue profit the so-called invisible hand of competition will guide it to produce the greatest social benefit, and therefore that the proper nexus between business and government is no nexus at all.

is government, nevertheless, a “necessary” burden on business, as the speaker also claims? yes. laissez faire (n. 放任, 自由主義) is an extreme view that fails to consider the serious harms that business would do—to other businesses and to the society—if left to its own devices. and the harms may very well exceed the benefits. in fact, history has shown that left entirely to themselves, corporations can be expected not only to harm the society by making unsafe products and by polluting the environment, but also to cheat one another, exploit workers, and fix prices—all for profit’s sake. thus, i agree that government constraints on business are necessary burdens.

ideally, the government should regulate against harmful practices but not interfere with the beneficial ones. but achieving this balance is not a simple matter. for instance, i know of a business that was forced by government regulation of toxic effluents to spend over 120,000 to clean up an area outside of its plant where employees had regularly washed their hands. the ‘toxin’ in this case was nothing more than biodegradable soap. this example suggests that perhaps the real issue here is not whether government is a necessary burden on business—for it clearly is—but rather how best to ensure that its burdens don’t outweigh its benefits.

in sum, the speaker’s two assertions are palpable (明顯可知的) ones that are amply supported by the evidence. the more intriguing question is how to strike the best balance.

62. “what education fails to teach us is to see the human community as one. rather than focus on the unique differences that separate one nation from another, education should focus on the similarities among all people and places on earth.”what do you think of the view of education expressed above? explain, using reasons and/or specific examples from your own experience, observations, or reading.

this view of education seems to recommend that schools stress the unity of all people instead of their diversity. while i agree that education should include teaching students about characteristics that we all share, doing so need not necessarily entail shifting focus away from our differences. education can and should include both.

on the one hand, we are in the midst of an evolving global community where it is increasingly important for people to recognize our common humanity, as well as specific hopes and goals we all share. people universally prefer health to disease, being nourished to starving, safe communities to crime-riddled ones, and peace to war. focusing on our unity will help us realize these hopes and goals. moreover, in our pluralistic democracy it is crucial to find ways to unify citizens from diverse backgrounds. otherwise, we risk being reduced to ethnic, religious or political factions at war with one another, as witnessed recently in the former yugoslavia (南斯拉夫). our own diverse society can forestall such horrors only if citizens are educated about the democratic ideals, heritage, rights and obligations we all have in common.

on the other hand, our schools should not attempt to erase, ignore, or even play down (v. 降低, 貶低, 減少) religious, ethnic or cultural diversity. first of all, schools have the obligation to teach the democratic ideal of tolerance, and the best way to teach tolerance is to educate people about different religions, cultures and so on. moreover, educating people about diversity might even produce a unifying effect—by promoting understanding and appreciation among people from all backgrounds.

in conclusion, while it may appear paradoxical to recommend that education stress both unity and diversity, it is not. understanding our common humanity will help us achieve a better, more peaceful world. toward the same end, we need to understand our differences in order to better tolerate them, and perhaps even appreciate them. our schools can and should promote both kinds of understanding by way of a balanced approach.

63. “as government bureaucracy increases, citizens become more and more separated from their government.”discuss the extent to which you agree or disagree with the opinion expressed above. support your point of view with reasons and/or examples from your own experience, observations, or reading.

at first glance, it would seem that increased bureaucracy creates obstacles between the citizens and those who govern, thereby separating the two groups. closer examination reveals, however, that in many ways government bureaucracy actually bridges this gap, and that new technologies now allow for ways around the gap.

first of all, many government bureaucracies are established as a response to the needs of the citizenry. in a sense, they manifest a nexus between citizens and government, providing a means of communication and redress (n.矯正) for grievances that would not otherwise be available. for example, does the fda (food and drug administration[美國]食品及藥物管理局), by virtue of its ensuring the safety of our food and drugs, separate us from the government? or does the fha (federal housing administration聯邦住宅管理局), by helping to make home ownership more viable to ordinary citizens, thereby increase the gap between citizens and the government? no; these agencies serve our interests and enhance the accessibility of government resources to citizens.

admittedly, agencies such as these are necessary proxies for direct participation in government, since our societal problems are too large and complex for individuals to solve. however, technology is coming forward to bridge some of the larger gaps. for example, we can now communicate directly with our legislators by e-mail, visit our lawmakers on the web, and engage in electronic town hall meetings. in addition, the fact that government bureaucracies are the largest employers of citizens should not be overlooked. in this sense, bureaucracies bridge the gap by enabling more citizens to become part of the government.

in the final analysis, one can view bureaucracies as surrogates for individual participation in government; however, they are more accurately viewed as a manifestation of the symbiotic relationship between citizens and the government.

64. “the goal of business should not be to make as big a profit as possible. instead, business should also concern itself with the wellbeing (n. 福利) of the public.”discuss the extent to which you agree or disagree with the opinion expressed above. support your point of view with reasons and/or examples from your own experience, observations, or reading.

i agree that business has some obligation to the community and society in which it operates. as it stands, however, the statement permits one to conclude that this obligation should take precedence over the profit objective. by allowing for this interpretation, the speaker fails to appreciate the problems associated with shouldering business with an affirmative duty to ensure the public’s well being.

the primary reason why i agree business should have a duty to the public is that society would be worse off by exonerating business from social responsibility. left entirely to their own self-interest, businesses pollute the environment, withhold important product information from consumers, pay employees substandard wages, and misrepresent their financial condition to current and potential shareholders. admittedly, in its pursuit of profit business can benefit the society as well—by way of more and better-paying jobs, economic growth, and better yet lower-priced products. however, this point ignores the harsh consequences—such as those listed earlier—of imposing no affirmative social duty on business.

another reason why i agree business should have a duly to the public is that business owes such a duty. a business enters into an implied contract with the community in which it operates, under which the community agrees to permit a corporation to do business while the business implicitly promises to benefit, and not harm, the community. this understanding gives rise to a number of social obligations on the part of the business—to promote consumer safety, to not harm the environmental, to treat employees and competitors fairly, and so on.

although i agree that business should have a duty to serve the pubic, i disagree that this should be the primarily objective of business. imposing affirmative social duties on business opens a pandora’s box of problems—for example, how to determine. (1) what the public interest is in the first place, (2) which public interests are most important, (3) what actions are in the public interest, and (4) how business’ duty to the public might be monitored and enforced. government regulation is the only practical way to deal with these issues, yet government is notoriously inefficient and corrupt; the only way to limit these problems is to limit the duty of business to serve the public interest.

in sum, i agree that the duty of business should extend beyond the simple profit motive. however, its affirmative obligations to society should be tempered against the pubic benefits of the profit motive and against the practical problems associates

65. “the rise of multinational corporations is leading to global homogeneity*. because people everywhere are beginning to want the same products and services, regional differences are rapidly disappearing.”* homogeneity: sameness, similarity

discuss the extent to which you agree or disagree with the opinion expressed above. support your point of view with reasons and/or examples from your own experience, observations, or reading.

although global homogeneity in a broader sense may not be as inexorable as the speaker here suggests, i agree that multinational corporations are indeed creating global sameness in consumer preferences. this homogeneity is manifested in two concurrent megatrends (n. 大趨勢): (1) the embracing of american popular culture throughout the world, and (2) a synthesis of cultures, as reflected in consumer preferences.

the first trend is toward americanization (美國化) of popular culture throughout the world. in food and fashion, once a nation’s denizens “fall into the gap” or get a taste of a coke or big mac, their preferences are forever westernized. the ubiquitous nike “swoosh,” which nearly every soccer player in the world will soon don, epitomizes this phenomenon. in media, the cultural agendas of giants such as time-warner (華納兄弟娛樂公司) now drive the world’s entertainment preferences. the rolling stones and the stars of america’s prime-time television shows are revered among young people worldwide, while mozart’s music, shakespeare’s prose, and ghandi’s ideology are largely ignored.

a second megatrend is toward a synthesis of cultures into a homogenous stew. the popularity of “world music” and of the “new age” health care and leisure-time activities aptly illustrate this blending of eastern, western and third-world cultures. perhaps nowhere is the cultural-stew paradigm more striking, and more bland (blander), than at the international “food courts” now featured in malls throughout the developed world.

these trends appear inexorable. counter-attacks, such as ebonies, rap music, and bilingual education, promote the distinct culture of minority groups, but not of nations. further homogenization of consumer preferences is all but (adv. 簡直是,幾乎跟..一樣) ensured by failing trade barriers, coupled with the global billboard that satellite communications and the internet provide.

in sum, american multinationals have indeed instigated a homogeneous global, yet american-style, consumerism—one which in all likelihood will grow in extent along with free-market capitalism and global connectivity.

66. “manufacturers are responsible for ensuring that their products are safe. if a product injures someone, for whatever reason, the manufacturer should be held legally and financially accountable for the injury.”discuss the extent to which you agree or disagree with the opinion expressed above. support your point of view with reasons and/or examples from your own experience, observations, or reading.

in determining whether manufacturers should be accountable for all injuries resulting from the use of their products, one must weigh the interests of consumers against those of manufacturers. on balance, holding manufacturers strictly liable for such injuries is unjustifiable.

admittedly, protecting consumers from defective and dangerous products is an important and worthwhile goal. no doubt nearly all of us would agree that health and safety should rank highly as an objective of public policy. also, compelling a high level of safety forces manufacturers to become more innovative in design, use of materials, and so forth. consumers and manufacturers alike benefit, of course, from innovation.

however, the arguments against a strict-liability standard are more compelling. first, the standard is costly. it forces manufacturers to incur undue expenses for overbuilding, excessive safety testing, and defending liability law suits. consumers are then damaged by ultimately bearing these costs in the form of higher prices. second, the standard can be unfair. it can assign fault to the wrong party; where a product is distributed through a wholesaler and/or retailer, one of these parties may have actually caused, or at least contributed to, the injury. the standard can also misplace fault where the injured party is not the original consumer. manufacturers cannot ensure that second-hand users receive safe products or adequate instructions and warnings. finally, where the injured consumer uses the product for a purpose or in a manner other than the intended one, or where there were patent dangers that the user should have been aware of, it seems the user, not the manufacturer, should assume the risk of injury.

in sum, despite compelling interests in consumer safety and product innovation, holding manufacturers accountable for all injuries caused by their products is unjustifiably costly to society and unfair to manufacturers.

67. “work greatly influences people’s personal lives—their special interests, their leisure activities, even their appearance away from the workplace.”discuss the extent to which you agree or disagree with the opinion expressed above. support your point of view with reasons and/or examples from your own experience, observations, or reading.

the speaker claims that our jobs greatly influence our personal interests, recreational activities and even appearance. while i agree that the personal lives of some people are largely determined by their work, in my view it would be a mistake to draw this conclusion generally. in my observation, the extent to which occupation influences personal life depends on (1) the nature of the work, and (2) how central the work is to one’s sense of self.

on the one hand, consider my friends steve and william. steve works as a gardener, but after work he creates oil paintings of quality and poignancy. his leisure time is spent alternately at the sea, in the wilderness, and in dark cafes. william paints houses for a living, but on his own time he collects fine art and books in first edition, as well as reading voraciously in the area of american history. their outside activities and appearance speak little about what steve or william do for a living, because these men view their jobs as little more than a means of subsidizing the activities that manifest their true selves. at the same time, they have chosen jobs that need not spill over into their personal lives, so the nature of their jobs permits them to maintain a distinctive identity apart from their work.

on the other hand, consider my friend shana—a business executive who lives and breathes her work. after work hours you can invariably find her at a restaurant or bar with colleagues, discussing work. shana’s wardrobe is primarily red—right off the dress-for-success page of a woman’s magazine. for shana, her job is clearly an expression of her self-concept. also, by its nature it demands shana’s attention and time away from the workplace.

what has determined the influence of work on personal lives in these cases is the extent to which each person sees himself or herself in terms of work. clearly, work is at the center of shana’s life, but not of either steve’s or william’s. my sample is small; still, common sense and intuition tell me that the influence of work on one’s personal life depends both on the nature of the work and on the extent to which the work serves as a manifestation of one’s self-concept (自我概念).

68. “since the physical work environment affects employee productivity and morale, the employees themselves should have the right to decide how their workplace is designed.”discuss the extent to which you agree or disagree with the opinion stated above. support your views with reasons and/or examples from your own experience, observations, or reading.

i agree that physical workspace can affect morale and productivity and that, as a result, employees should have a significant voice in how their work areas are designed. however, the speaker suggests that each employee should have full autonomy over his or her immediate workspace, i think this view is too extreme, for it ignores two important problems that allowing too much freedom over workspace (工作區;工作空間) can create.

on the one hand, i agree that some aspects of workspace design are best left to the individual preferences of each worker. location of personal tools and materials, style and size of desk chair, and even desk lighting and decorative desk items, can each play an important role in a worker’s comfort, psychological wellbeing, concentration, and efficiency. moreover, these features involve highly subjective preferences, so it would be inappropriate for anyone but the worker to make such choices.

on the other hand, control over one’s immediate workspace should not go unchecked, for two reasons. first, one employee’s workspace design may inconvenience (v. to subject to inconvenience: put to trouble), annoy, or even offend nearby coworkers. for example, pornographic pinups ([釘在牆上的]偶像,偶像照片[例如裸體女人或影星、歌星的照片]) may distract some coworkers and offend others, thereby impeding productivity, fostering ill-will and resentment, and increasing attrition—all to the detriment of the company. admittedly, the consequences of most workspace choices would not be so far-reaching. still, in my observation many people adhere, consciously or not, to the adage that one person’s rights extend only so far as the next person’s nose (or ears. or eyes). a second problem with affording too much workspace autonomy occurs when workspaces are not clearly delineated—by walls and doors—or when workers share an immediate workspace. in such cases, giving all workers concurrent authority would perpetuate conflict and undermine productivity.

in conclusion, although employees should have the freedom to arrange their work areas, this freedom is not absolute. managers would be well-advised to arbitrate workspace disputes and, if needed, assume authority to make final decisions about workspace design.

69. “the most important quality in an employee is not specific knowledge or technical competence. instead, it is the ability to work well with other employees.”discuss the extent to which you agree or disagree with the opinion expressed above. support your point of view with reasons and/or examples from your own experience, observations, or reading.

whether the ability to work with others is more important than specific knowledge and technical competence depends on the specific job as well as the complexity of the job’s technical aspects. in general, however, social skills are more critical than technical competence to the ultimate success of an organizational unit.

admittedly, some level of technical competence and specific knowledge is needed to perform any job. without some knowledge of the systems, procedures, and vocabulary used in one’s department or division, an employee cannot communicate effectively with peers or contribute meaningfully to team goals. by the same token, however, nearly every job—even those in which technical ability would seem to be of paramount importance—calls for some skill in working with other employees. computer programmers, for example, work in teams to develop products according to agreed-upon specifications and timelines (time-line: a schedule of events and procedures: timetable). scientists and researchers must collaborate to establish common goals and to coordinate efforts. even teachers, who are autonomous in the classroom, must serve on committees and coordinate activities with administrators and other teachers.

moreover, employees can generally learn technical skills and gain specific knowledge through on-the-job (adj. 在職的) training and continuing education (depending on the complexity of the skills involved). social skills, on the other hand, are more innate and not easily learned. they are, therefore, requisite skills that employees must possess at the outset (adv. 起先,當初) if the organizational unit is to succeed.

in sum, specific knowledge does admittedly play a more critical role than social skills in some highly-technical jobs; nevertheless, the ability to work well with other employees is ultimately more important, since all jobs require this ability and since it is more difficult, to learn social skills on the job (在工作中的,在職的).

70. “so long as no laws are broken, there is nothing unethical about doing whatever you need to do to promote existing products or to create new products.”discuss the extent to which you agree or disagree with the opinion expressed above. support your point of view with reasons and/or examples from your own experience, observations, or reading.

the speaker asserts that in creating and marketing products, companies act ethically merely by not violating any laws. although the speaker’s position is not wholly insupportable, far more compelling arguments can be made for holding businesses to higher ethical standards than those required by the letter of the law (法律的字面意義, 法律條文).

on the one hand, two colorable arguments can be made for holding business only to legal standards of conduct. first, imposing a higher ethical duty can actual harm consumers in the long term. compliance with high ethical standards can be costly for business, thereby lowering profits and, in turn, impeding a company’s ability to create jobs (for consumers), keep prices low (for consumers), and so forth. second, limited accountability is consistent with the “buyer beware (小心, 謹防)” principle that permeates our laws of contracts and torts ([律]民事侵權行為), as well as our notion in civil procedure that plaintiffs carry the burden of proving damage. in other words, the onus should be on consumers to protect themselves, not on companies to protect consumers.

on the other hand, several convincing arguments can be made for holding business to a higher ethical standard. first, in many cases government regulations that protect consumers lag behind advances in technology. a new marketing technique made possible by internet technology may be unethical but nevertheless might not be proscribed by the letter of the laws which predated the internet. second, enforceability might not extend beyond geographic borders. consider, for example, the case of “dumping.” when products fail to comply with u.s. regulations, american companies frequently market—or “dump”—such products in third-world countries where consumer-protection laws are virtually nonexistent. third, moral principles form the basis of government regulation and are, therefore, more fundamental than the law.

in the final analysis, while overburdening businesses with obligations to consumers may not be a good idea in the extreme, our regulatory system is not as effective as it should be. therefore, businesses should adhere to a higher standard of ethics in creating and marketing products than what is required by the letter of the law.

71. “commercialism (重商主義) has become too widespread. it has even crept into schools and places of worship. every nation should place limits on what kinds of products, if any, can be sold at certain events or places.”discuss the extent to which you agree or disagree with the opinion expressed above. support your point of view with reasons and/or examples from your own experience, observations, or reading.

has commercialism become too widespread, particularly in schools, churches, and other places which traditionally have been safe havens from commercialism? if so, does the government have a responsibility to curb the problem? the answer to both questions, in my view, is no.

there is no evidence that commercialism is creeping into our churches. admittedly, some commercial activity is present in our schools. food service is increasing outsourced (outsourcing: the practice of subcontracting manufacturing work to outside and especially foreign or nonunion companies) to fast-food chains; a plethora of goods and services is sold in college bookstores and advertised in their school newspapers; and students serve as walking billboards (廣告牌) for the companies whose logos appear on clothing. however, this kind of commercialism does not interfere with school activities; to the contrary, in the first two cases they contribute to the efficient functioning of the organization. outsourcing food service, for example, is a cost-cutting measure which provides additional funding for teaching materials, facilities, and teacher salaries.

i do agree that, in general, commercialism is becoming more widespread, and that one of the byproducts may be a decline in the quality of our culture. electronic billboards now serve as backdrops for televised sporting events, and web sites must sell advertising space to justify maintenance costs. does this mean that government should step in and ban the sale of products in certain venues? no. this would require that government make ad hoc (adj.特別的), and possibly arbitrary, decisions as to which products may be sold or advertised at which places and events. these are value judgments that are best left to individual schools, churches, and other organizations. moreover, the expense of enforcing the regulations may well outweigh the cultural benefits, if any.

in sum, while commercialism is undeniably becoming more widespread, it is minimally intrusive and works to the net benefit of society. as a matter of public policy, therefore, government should not attempt to regulate the extent of commercialism.

72. “companies should not try to improve employees’ performance by giving incentives—for example, awards or gifts. these incentives encourage negative kinds of behavior instead of encouraging a genuine interest in doing the work well.”discuss the extent to which you agree or disagree with the opinion stated above. support your views with reasons and/or examples from your own experience, observations, or reading.

providing employee incentives can be a double-edged sword. on the one hand, the promise of bonuses or gifts can spur workers to higher achievement. on the other hand, incentives can create resentment and internal competitiveness that are damaging to morale and to the organization. even so, i think a carefully designed incentive program can operate to the net benefit (of) a company.

incentive programs are counterproductive when the distribution of rewards appears to be personally biased, when the program recognizes just one kind among many important jobs in the organization, or when there are too few rewards available. for example, if a manager regularly rewards an employee who is perceived to be a favorite, coworkers will be resentful. or if the company decides to recognize high sales, while ignoring an especially precise cost-assessment from the accounting department, the accountants may feel their work is not valued. finally, if rewards are too few, some employees will become overly competitive, while others may simply stop trying.

however, incentive programs can be designed to avoid such pitfalls. first, the company must determine that it can provide sufficient rewards to motivate all employees. then it must set, and follow, clear and non-arbitrary guidelines for achievement. finally, management should provide appropriate incentives throughout the organization, thereby sending the message that all work is valued. admittedly, even a thoughtfully designed incentive program cannot entirely prevent back-stabbing and unfair competitive tactics. but watchful management can quell much of this behavior, and the perpetrators usually show their true colors in time.

in sum, i think that the productivity inspired by thoughtful incentive programs will very likely outweigh any negative consequences. in the final analysis, then, i disagree with the speaker’s recommendation against their use.

73. people often give the following advice: “be yourself. follow your instincts and behave in a way that feels natural.”do you think that, in general, this is good advice? why or why not? develop your point of view by giving reasons and/or examples from your own experience, observations, or reading.

the advice to act naturally or follow one’s instincts can, admittedly, be helpful advice for someone torn between difficult career or (and) personal choices in life. in most situations, however, following this advice would neither be wise nor (be) sensible. following one’s own instincts should be tempered by codes of behavior appropriate to the situation at hand.

first of all, doing what comes naturally often amounts to impulsive overreaction and irrational behavior, based on emotion. everyone experiences impulses from time to time, such as hitting another person, quitting one’s job, having an extramarital affair, and so forth. people who act however they please or say whatever is on their mind without thinking about consequences, especially without regard to social situation, may offend and alienate others. at the workplace, engaging in petty gossip, sexual harassment (性騷擾), or back-stabbing might be considered “natural”; yet such behavior can be destructive for the individuals at the receiving end as well as for the company. and in dealings with foreign business associates, what an american might find natural or instinctive, even if socially acceptable here, might be deeply insulting or confusing to somebody from another culture.

second, doing what comes naturally is not necessarily in one’s own best interests. the various behaviors cited above would also tend to be counterproductive for the person engaging in them. “natural” behavior could prove deadly to one’s career, since people who give little thought before they act cannot be trusted in a job that requires effective relationships with important clients, colleagues, and others.

third, the speaker seems to suggest that you should be yourself, then act accordingly—in that order. but we define ourselves in large measure by our actions. young adults especially lack a clear sense of self. how can you be yourself if you don’t know who you are? even for mature adults, the process of evolving one’s concept of self is a perpetual one. in this respect, then, the speaker’s recommendation does not make much sense.

in sum, one should not follow the speaker’s advice universally or too literally. for unless a person’s instincts are to follow standard rules of social and business etiquette, natural behavior can harm others as well as constrain one’s own personal and professional growth.

74. “the people we remember best are the ones who broke the rules.”discuss the extent to which you agree or disagree with the opinion expressed above. support your point of view with reasons and/or examples from your own experience, observations, or reading.

i strongly agree that rule-breakers are the most memorable (難忘的) people. by departing from the status quo, iconoclasts call attention to themselves, some providing conspicuous mirrors for society, others serving as our primary catalysts for progress.

in politics, for example, rule-breakers mahatma ghandi and martin luther king secured prominent places in history by challenging the status quo through civil disobedience (不合作主義, 溫和抵抗). renegades such as ghengus khan, stalin, and hussein, broke all the human-rights “rules,” thereby leaving indelible marks in the historical record. and future generations will probably remember nixon and kennedy more clearly than carter or reagan, by way of their rule-breaking activities—specifically, nixon’s watergate debacle and kennedy’s extra-marital trysts.

in the arts, mavericks such as dali, picasso, and warhol, who break established rules of composition, ultimately emerge as the greatest artists, while the names of artists with superior technical skills are relegated to the footnotes of art-history textbooks. our most influential popular musicians are the flagrant rule breakers—for example, be-bop (bebop: 爵士音樂的一種) musicians such as charlie parker and thelonius monk, who broke all the harmonic rules, and folk musician-poet bob dylan, who broke the rules for lyrics.

in the sciences, innovation and progress can only result from challenging conventional theories—i.e., by breaking rules. newton and einstein, for example, both refused to blindly accept what were perceived at their time as certain “rules” of physics. as a result, both men redefined those rules, and both men emerged as two of the most memorable figures in the field of physics.

in conclusion, it appears that the deepest positive and negative impressions appear on either side of the same iconoclastic coin. those who leave the most memorable imprints in history do so by challenging norms, traditions, cherished values, and the general status quo—that is, by breaking the rules.

75. “there are essentially two forces that motivate people: self-interest and fear.”discuss the extent to which you agree or disagree with the opinion stated above. support your position with reasons and/or examples from your own experience, observations, or reading.

the speaker claims that people are motivated only by fear and self-interest. this claim relies on the belief that human beings are essentially selfish, or egoistic. in my view, the speaker oversimplifies human nature, ignoring the important motivating force of altruism.

on the one hand, i agree that most of our actions result in large part from self-interest and from our survival instincts, such as fear. for example, our educational and vocational lives are to a great extent motivated by our interest in ensuring our own livelihood, safety, health, and so on. we might perpetuate bad personal relationships because we are insecure—or afraid—of what will happen to us if we change course. even providing for our own children may to some extent be motivated by selfishness—satisfying a need for fulfillment or easing our fear that we will be alone in our old age.

on the other hand, to assert that all of our actions are essentially motivated by self-interest and fear is to overemphasize one aspect of human nature. humans are also altruistic—that is, we act to benefit others, even though doing so may not be in our own interest. the speaker might claim that altruistic acts are just egoistic ones in disguise—done to avoid unpleasant feelings of guilt, to give oneself pleasure, or to obligate another person. however, this counter argument suffers from three critical problems. first, some examples of altruism are difficult to describe in terms of self-interest alone. consider the soldier who falls on a grenade to save his companions. it would be nonsensical to assert that this soldier is acting selfishly when he knows his action will certainly result in his own immediate death. second, the argument offends (違反) our intuition that human motivation is far more complex. third, it relies on a poor assumption; just because we feel good about helping others, it does not follow that the only reason we help is in order to feel good.

in sum, the speaker oversimplifies human nature. all human motivation cannot be reduced to fear and self-interest. we can also be motivated by altruism, and the pleasure we might take in helping others is not necessarily an indication that our actions are selfish.

76. “for a leader there is nothing more difficult, and therefore more important, than to be able to make decisions.”discuss the extent to which you agree or disagree with the opinion expressed above. support your point of view with reasons and/or examples from your own experience, observations, or reading.

i agree that decisiveness is one clear mark of an effective leader. however, the speaker goes further to make the dual claim that decision-making is the most difficult and the most important aspect of a leader’s job. in my view, this additional claim amounts to an overstatement that fails to consider other aspects of a leader’s job that are either difficult or important.

first of all, decisiveness is not necessarily the most difficult aspect of a leader’s job. in fact, leaders rise to their positions typically because decisiveness comes easily or naturally to them. in this sense, the speaker’s claim runs contrary to actual experience. also, for some leaders the stress and the burden of their job pose more difficulties for them than the mere act of making decisions. for other leaders, balancing professional and personal life, or even time management in general, may be the most challenging aspect of the job, since leaders are typically very busy people.

secondly, decisiveness is not necessarily the key factor in determining the quality of leadership. decisiveness does not guarantee a good decision. an effective leader must also have wisdom, perspective (the capacity to view things in their true relations or relative importance *urge you to maintain your perspective and to view your own task in a larger framework*), clear vision, judgment, and courage. moreover, other factors such as trust and respect for others may be equally or more critical, since subordinates may not be willing to devote themselves to the plans and goals of a leader they mistrust or hold in low regard. even the best decision will be of little value without the commitment of others to carry it out. simply put, without someone to lead, a person cannot be a leader.

to sum up, i agree with the speaker only insofar as the ability to make decisions is a necessary ingredient of successful leadership. however, decision-making is not necessarily the most difficult aspect of every leader’s job; nor is it necessarily the most important factor in determining the effectiveness of a leader.

77. although “genius” is difficult to define, one of the qualities of genius is the ability to transcend traditional modes of thought and create new ones.explain what you think the above statement means and discuss the extent to which you agree or disagree with this definition of genius. in your discussion, be sure to include at least one example of someone who, in your opinion, exemplifies genius or a particular characteristic of genius. (attention!)

i strongly agree that true genius is the ability to see beyond conventional modes of thinking and to suggest new and better ones. this definition properly sets genius apart from lesser instances of critical acumen, inventiveness or creativity. under this definition, a true genius must successfully (1) challenge the assumptions underlying a current paradigm, and (2) supplant the old paradigm with a new, better, and more fruitful one.

this two-pronged (having a usually specified number of parts or approaches *a two-pronged strategy*) standard for true genius is aptly illustrated by examining the scientific contribution of the 15th-century astronomer copernicus (哥白尼). prior to copernicus, our view of the universe was governed by the ptolemaic (托勒密天動說的) paradigm of a geocentric (以地球為中心的) universe, according to which our earth was in a fixed position at the center of the universe, with other heavenly bodies revolving around it. copernicus challenged this paradigm and its key assumptions by introducing a distinction between real motion and motion that is merely apparent, in doing so, he satisfied the first requirement of a true genius.

had copernicus managed to show only that the old view and its assumptions were problematic, we would not consider him a genius today. copernicus went on, however, to develop a new paradigm; he claimed that the earth is rotating while hurtling (hurtle: v.急飛) rapidly through space, and that other heavenly bodies only appear to revolve around the earth. moreover, he reasoned that his view about the earth’s real motion could explain the apparent motion of the sun, stars and other planets around the earth. it turned out he was right; and his theories helped facilitate galileo’s empirical observations, kepler’s laws of planetary motion, and newton’s gravitational principle.

to sum up, i find the proposed definition of true genius incisive and accurate; and the example of copernicus aptly points up (v. 使尖銳, 強調) the two required elements of true genius required by the definition.

78. most people would agree that buildings represent a valuable record of any society’s past, but controversy arises when old buildings stand on ground that modern planners feel could be better used for modern purposes.in your opinion, which is more important—preserving historic buildings or encouraging modern development? explain your position, using reasons and examples based on your own experiences, observations, or reading.

the issue of whether to raze an old, historic building to make way for progress is a complex one, since it involves a conflict between our interest in preserving our culture, tradition, and history and a legitimate need to create practical facilities that serve current utilitarian purposes. in my view, the final judgment should depend on a case-by-case analysis of two key factors.

one key factor is the historic value of the building. an older building may be worth saving because it uniquely represents some bygone era. on the other hand, if several older buildings represent the era just as effectively, then the historic value of one building might be negligible. if the building figured centrally into the city’s history as a municipal structure, the home of a founding family or other significant historical figure, or the location of important events, then its historic value would be greater than if its history was an unremarkable one.

the other key factor involves the specific utilitarian needs of the community and the relative costs and benefits of each alternative in light of those needs. for example, if the need is mainly for more office space, then an architecturally appropriate add-on (外接式附屬檔案;[追加]) or annex might serve just as well as a new building. on the other hand, an expensive retrofit (更新,改進;[修整]) may not be worthwhile if no amount of retrofitting would permit it to serve the desired function. moreover, retrofitting might undermine the historic value of the old building by altering its aesthetic or architectural integrity.

in sum, neither modernization for its own sake nor indiscriminate preservation of old buildings should guide decisions in the controversies at issue. instead, decisions should be made on a case-by-case basis, considering historic value, community need, and the comparative costs and benefits of each alternative.

79. “the ability to deal with people is as purchasable a commodity as sugar or coffee, and it is worth more than any other commodity under the sun.”explain what you think the above quotation means and discuss the extent to which you agree or disagree with it. support your position with relevant reasons and/or examples from your own experience, observations, or reading.

this first part of this statement means that interpersonal—or social—skills can be marketed as part of a bundle of assets that one might tout to a prospective client, customer, or especially employer. presumably, the extent and value of these skills can be gauged by one’s previous experience with clients and customers or at jobs requiring a significant amount of teamwork and cooperation among workers—as measured by factors such as one’s tenure in such a job and letters of reference (letter of reference: 調查信,查詢信,證明信,保證書) from supervisors. while this claim seems plausible in the abstract (adv. 抽象地, 理論上), it ignores critical valuation problems. furthermore, the claim that the ability to deal with people exceeds the value of all other commodities is an overgeneralization (n. 超廣義化), since relative values depend on particular circumstances.

the first problem with this claim is that it is far more difficult to quantify the value of interpersonal skills, or other human qualities, than the value of commodities such as coffee or sugar, which can be measured, weighed, or otherwise examined prior to purchase. to a large extent, the ability to work with people is a quality whose true value can be determined only after it is purchased, then tried and tested for a period of time. additionally, its value may vary depending on the idiosyncrasies of the job. for example, a technically-oriented programmer or researcher might function well with a team of like-minded (志趣相投的) workers, yet have trouble dealing with management or marketing personnel.

the second problem with this claim is that it overgeneralizes in asserting that the ability to work with people is “worth more than any other commodity.” the relative value of this ability depends on the peculiarities of the job. in some jobs, especially sales, ambition and tenacity are more valuable. in other areas, such as research and development, technical skills and specific knowledge are paramount. moreover, in some businesses, such as mining or oil-drilling, the value of raw materials and capital equipment might be far more important a commodity than the social skills, or most other skills, of employees—depending on the economic circumstances.

in sum, the ability to deal with people is purchasable only to a limited extent, since its full value cannot be determined prior to purchase. moreover, its full value depends on the organizational unit as well as the nature of the business.

80. “as individuals, people save too little and borrow too much.”from your perspective, how accurate is the view expressed above? in your discussion, be sure to consider the conditions under which it is appropriate to save money and the conditions under which it is appropriate to borrow. develop your position using reasons and/or examples from your own experience, observations, or reading.

whether an individual saves too little or borrows too much depends on the purpose and extent of either activity. while appropriate and prudent in some circumstances, either can be irresponsible in excess. the evidence suggests that, on balance, people today tend to borrow irresponsibly and are on the brink of saving irresponsibly as well.

traditionally, saving is viewed as a virtue, while borrowing is considered a vice. however, just the opposite may be true under certain circumstances. foregoing saving in favor of immediate spending may at times be well justified. a serious hobbyist (沉溺於某種癖好者), for example, may be justified in foregoing saving to spend money on a hobby that provides great joy and fulfillment—whether or not it also generates income. a relatively expensive automobile is justifiable if the additional expense provides added safety for the owner and his family. and foregoing saving is appropriate, and often necessary, for “rainy day (窮困時期, 雨天)” medical emergencies or unanticipated periods of unemployment. borrowing can also be prudent—if the loan is affordable and applied toward a sound long-term investment.

were saving and borrowing limited to these types of scenarios (a sequence of events especially when imagined; especially: an account or synopsis of a possible course of action or events *his scenario for a settlement envisages… reunification*), i would aver that people today save and borrow responsibly. however, the evidence suggests otherwise. americans now purchase on credit far more expensive automobiles, relative to income, than ever before (adv. 以往任何時候)—vehicles that are far more than what is needed for safe transportation. excessive credit-card debt, another type of unjustifiable borrowing, is at record levels—and rising—among american households. does the baby-boomers’ current penchant for retirement investing compensate for these excesses? probably not. this trend is fueled by unrealistic expectations of future returns; it may therefore, escalate to speculation and, at its height, widespread leveraging—i.e., borrowing. such speculation is more suited to highly sophisticated investors who can well afford to lose their entire investment than to average americans and their nest eggs (n. 為養老, 應變等用的儲蓄).

in conclusion, while people seem to be saving aggressively today, their investment choices and concomitant high spending and borrowing levels call into question the assertion that we are indeed a “nation of savers.”

81. “no one can possibly achieve any real and lasting success or ‘get rich’ in business by conforming to conventional practices or ways of thinking.”discuss the extent to which you agree or disagree with the opinion stated above. support your views with reasons and/or examples from your own experience, observations, or reading.

whether a conformist can achieve lasting success or “get rich” in business depends primarily on the type of business involved. iconoclasts rise to the top in newer industries and in those where consumer demand is in constant flux. conformists ultimately prevail, however, in traditional service industries ensconced in systems and regulations.

in consumer-driven industries, innovation, product differentiation, and creativity are crucial to lasting success, in the retail and media sectors, for example, unconventional products and advertising are necessary to catch the attention of consumers and to keep up with the vagaries of consumer tastes. those who take an iconoclastic approach tend to recognize emerging trends and to rise above their peers. for example, ted turner’s departure from the traditional format of the other television networks, and the responsiveness of amazon.com to burgeoning internet commerce, propelled these two giants to leadership positions in their industries. and in technology, where there are no conventional practices or ways of thinking to begin with, companies that fail to break away from last year’s paradigm are soon left behind by the competition.

however, in traditional service industries—such as finance, accounting, insurance, legal services, and health care—lasting success and riches (pl. 財富, 財產) come not to nonconformists but rather to those who can deliver services most effectively within the confines of established practices, policies, and regulations. of course, a clever idea for structuring a deal, or a creative legal maneuver, may play a role in winning smaller battles along the way. but such tactics are those of conformists who are playing by the same ground rules (程式, [運動]體育場館的運動章程) as their peers; winners are just better at the game.

in conclusion, while non-conformists tend to be the wildly successful players in technology-driven and consumer-driven industries, traditionalists are the winners in system-driven industries pervaded by policy, regulation, and bureaucracy.

82. “business and government must do more, much more, to meet the needs and goals of women in the workplace.”what do you think of the opinion expressed above? in your discussion, be sure to use reasons and/or examples from your own experience, observations, or reading.

the issue here is whether business and government are doing enough to help meet the needs and goals of women in the workplace. i agree with the speaker insofar as many employers can do more to accommodate the special needs of women in their role as mothers. however, it seems to me that business and government are doing their fair share otherwise for women in the workplace.

women differ fundamentally from men in their child-bearing ability. related to this ability is the maternal instinct (n. 母性本能)—a desire to nurture that is far stronger for women than for men, generally speaking. at a minimum, then, businesses should acknowledge these fundamental differences and accommodate them so that a female employee’s job and career are not jeopardized merely for fulfilling her instinctive role as a female. more and more businesses are providing maternal leave with full benefits, day-care facilities, and job-sharing programs to accommodate these special needs of women. in my observation, however, many businesses can do more in these respects.

however, beyond accommodating these fundamental differences, neither business nor government has a special duty to improve the status of women at the workplace. the government already has an obligation to enact and enforce anti-discrimination laws, and to provide legal means for seeking redress in cases of discrimination. moreover, business and government both have a legal duty to abide by (v. 堅持, 遵守) those laws by way of their hiring, salary, and job-promotion policies. discharging (discharge: 履行職責;履行諾言) this duty should, in my view, suffice to serve the special interests of women in the workplace. while many would argue that de facto double standards still run rampant and largely unchecked, this claim raises subjective perceptions about fairness that can neither be confirmed nor dispelled with certainty.

in sum, business and government can always do more to accommodate women in their special role as mothers. otherwise, insofar as they are adhering to our current anti-discrimination laws, business and government are discharging their duty to help meet the needs and goals of women at the workplace.

83. “we shape our buildings and afterwards our buildings shape us.”explain what you think this statement means and discuss the extent to which you do or do not agree with it. support your views with reasons and/or specific examples from your experience, observations, or reading.

i believe this statement should be interpreted broadly—to mean that we are influenced by the exterior shape of buildings, as well as by the arrangement of multiple buildings and by a building’s various architectural and aesthetic elements. while i doubt that buildings determine our character or basic personality traits, i agree that they can greatly influence our attitudes, moods, and even life styles.

on the structural and multi-structural scales, the arrangement of numerous buildings can shape us in profound ways. high-density commercial districts with numerous skyscrapers might result in stressful commuting, short tempers, a feeling of dehumanization, and so on. a “campus” arrangement of smaller, scattered buildings can promote health, wellbeing, and stress reduction by requiring frequent brisk outdoor jaunts. buildings with multiple floors can also “shape” us, literally, by requiring exercise up and down stairs.

as for floor plans and internal space, physical arrangement of workspaces can shape workers’ attitudes toward work and toward one another. sitting in small, gray cubicles lined up in militaristic (adj. 軍國主義的) rows is demoralizing, leaving workers with the feeling that they are little more than impersonal cogs of some office machine. but creative design of workspaces in varied arrangements can create feelings of uniqueness and importance in each employee. workspace relationships that suggest some sort of hierarchy may breed competitiveness among coworkers, and may encourage a more bureaucratic approach to work.

finally, as for aesthetic elements, the amount of light and location of windows in a building can shape us in significant psychological ways. for most people, daily tasks are more enjoyable in settings with plenty of natural light and at least some natural scenery. choice of colors can influence our mood, concentration, and efficiency. numerous psychological studies show that different colors influence behavior, attitudes, and emotions in distinctly different ways. yellow enhances appetite, blue has a tranquilizing effect, and gray is the color of choice for companies who want their workers to be subservient.

in sum, our buildings, the space around them and the space within them, can affect us in important ways that influence our outlook on life, relationships with coworkers, and even physical health and wellbeing.

84. “a business should not be held responsible for providing customers with complete information about its products or services; customers should have the responsibility of gathering information about the products or services they may want to buy.”discuss the extent to which you agree or disagree with the opinion stated above. support your views with reasons and/or examples from your own experience, observations, or reading.

requiring businesses to provide complete product information to customers promotes various consumer interests, but at the same time imposes burdens on businesses, government, and taxpayers. on balance, the burdens outweigh the benefits, at least in most cases.

a threshold (a level, point, or value above which something is true or will take place and below which it is not or will not) problem with disclosure requirements is that of determining what constitutes “complete” information. admittedly, legislating disclosure requirements clarifies the duties of business and the rights of consumers. yet determining what requirements are fair in all cases is problematic. should it suffice to list ingredients, instructions, and intended uses, or should customers also be informed of precise specifications, potential risks, and results of tests measuring a product’s effectiveness vis-a-vis (與…相對;與…比較) competing products? a closely related problem is that determining and enforcing disclosure standards necessarily involves government regulation, thereby adding to the ultimate cost to the consumer by way of higher taxes. finally, failure to comply may result in regulatory fines, a cost that may either have a chilling effect on product innovation or be passed on to the customers in the form of higher prices. either result operates to the detriment of the consumer, the very party whom the regulations are designed to protect.

these burdens must be weighed against the interest in protecting consumers against fraud and undue health and safety hazards. to assume that businesses will voluntarily disclose negative product information ignores the fact that businesses are motivated by profit, not by public interest concerns. however, consumers today have ready access to many consumer-protection resources, and may not need the protection of government regulation. although health and safety concerns are especially compelling in the case of products that are inherently dangerous—power tools, recreational equipment, and the like—or new and relatively untested products, especially pharmaceuticals, narrow exceptions can always be carved out for these products.

in conclusion, while stringent disclosure requirements may be appropriate for certain products, businesses and consumers alike are generally better off without the burdens imposed by requiring that businesses provide complete product information to all customers.

85. “advertising is the most influential and therefore the most important artistic achievement of the twentieth century.”discuss the extent to which you agree or disagree with the opinion expressed above. support your point of view with reasons and/or examples from your own experience, observations, or reading.

advertising is dearly the most influential art form in this century. it is therefore tempting to think it is also the most important. however, great artistic achievement is determined by criteria beyond mere influence. and when examined against these criteria, the genre of advertising does not measure up (v.合格, 符合標準) as truly important.

to begin with, great art inspires us to look at the human situation from new perspectives. for example, early impressionist paintings challenge our thinking about visual perception and about the nature of the reality we assume we see. other works, like rodin’s “the thinker,” capture for our reflection the essential value of human rationality. in stark contrast, advertising encourages people not to think or reflect at all, but simply to spend.

in addition, the significance of great artistic achievement transcends time, even when it reflects a particular age. yet advertising, by its very nature, is transient; in an eye-blink, today’s hot image or slogan is yesterday’s news. of course, the timelessness of a work cannot be determined in its own time. still, it’s hard to imagine even the most powerful advertisement living beyond its current ad campaign.

admittedly, one ad—andy warhol’s painting of the campbell soup can—has achieved timelessness. but notice the irony; the packaging or advertising image was banal until it was elevated above mere graphic design to high art. the lesson here is that advertising, in itself, probably will not achieve great importance as art. but taken up by the artist as content in a larger commentary on society, it can become transcendent.

in sum, artists will no doubt continue to comment on advertising and on the materialistic values it reflects and promotes. but the ads themselves, however influential in marketing terms, fail to fulfill all the criteria for important art.

86. “whether promoting a product, an event, or a person, an advertising campaign is most effective when it appeals to emotion rather than to reason.”discuss the extent to which you agree or disagree with the opinion expressed above. support your point of view with reasons and/or examples from your own experience, observations, or reading

there are two traditional advertising tactics for promoting a product, event, candidate, or point of view. one is to provide reasons; the other is to bypass reasons altogether and appeal strictly to emotion. considered in isolation, emotional appeals are far more effective. but many of the most influential ads combine slim reasons with powerful appeals to emotion.

to appreciate the power of emotional appeals we need only consider the promotion of sodas, beer, cigarettes, cosmetics and so on. this advertising is the most successful in the industry; and it trades almost exclusively on the manipulation of our desires, fears and senses of humor. in fact, it wouldn’t make sense to offer up (v. 貢獻) arguments, because there really aren’t any good reasons for consuming such products.

even so, some of these products are advertised with at least superficial reasoning. for instance, in the promotion of facial moisturizers it has become popular to use the image of a youthful woman with fresh, unlined skin along with the claim that the product “can reduce the signs of aging.” this is indeed a reason, but a carefully couched one that never really states that product users will look younger. still, countless middle-aged women will pay twice as much for products that add this claim to the expected image of youthfulness that trades on (v. 利用) their fears of growing old.

one of the most clever and ironic combined uses of reason and emotion is seen in the old volvo slogan, “volvo, the car for people who think.” the suggested reason for buying the car is obvious: it is the intelligent choice. but the emotional snare (something deceptively attractive) is equally clear; the ad appeals to one’s desire to be included in the group of intelligent, thoughtful people.

in conclusion, i agree that appeals to emotion are more powerful tools than arguments or reasoning for promoting products. it is no coincidence that advertising agencies hire professional psychologists, but not logicians (邏輯學家). still, in my view the most influential advertisements mix in a bit of reasoning as well.

87. “as technologies and the demand for certain services change, many workers will lose their jobs. the responsibility for those people to adjust to such change should belong to the individual worker, not to government or to business.”discuss the extent to which you agree or disagree with the opinion stated above. support your position with specific reasons and/or examples drawn from your reading, your observations, or your own experience.

as technology and changing social needs render more and more jobs obsolete, who is responsible for helping displaced workers adjust? while individuals have primary responsibility for learning new skills and finding work, both industry and government have some obligation to provide them the means of doing so.

l agree that individuals must assume primary responsibility for adjusting to job obsolescence, especially since our educational system has been preparing us for it. for decades, our schools have been counseling young people to expect and prepare for numerous major career changes during their lives. and concerned educators have recognized and responded to this eventuality (不測事件) with a broader base of practical and theoretical coursework that affords students the flexibility to move from one career to another.

however, industry should bear some of the responsibility as well. it is industry, after all, that determines the particular directions technological progress and subsequent social change will take. and since industry is mainly responsible for worker displacement, it has a duty to help displaced workers adjust—through such means as on-site training programs and stipends for further education.

government should also assume some of the responsibility, since it is partly government money that fuels technological progress in industry. moreover, government should help because it can help—for example, by ensuring that grants and federally insured student loans are available to those who must retool (to reequip with tools) in order to find new work. government can also help by observing and recording trends in worker displacement and in job opportunities, and by providing this information to individuals so that they can make prudent decisions about their own further education and job searches.

in conclusion, while individuals should be prepared for future job changes, both government and industry shoulder obligations to provide training programs, funding and information that will help displaced workers successfully retool and find new employment.

88. “each generation must accept blame not only for the hateful words and actions of some of its members but also for the failure of other members to speak out (v. 大膽地說, 大聲地說) against those words and actions.”discuss the extent to which you agree or disagree with the opinion expressed above. support your point of view with reasons and/or examples from your own experience, observations, or reading.

the issue at hand is whether each generation is blameworthy for the hateful words and actions of some of its members, and for the failure of others to denounce those hateful words and actions. in my view, it does not make clear sense to hold a vague abstraction like a generation responsible for anything. nevertheless, each person has a duty to resist hateful words and actions, and to speak out against them.

admittedly, up to a point we have no legal obligation to resist hateful words. given our first amendment right of free speech, we are entitled to say whatever hateful things we wish, as long as our words do not harass, slander, libel, incite to riot, or otherwise cause significant harm. even so, this legal entitlement does not absolve us of deeper moral duties. for example, all persons are morally bound not to harm others, and to be helpful where it is important and within our capacity. the rhetoric of hate violates both these duties by promoting attitudes and social climates in which those who are hated are refused help and often harmed.

not so clear is the issue of whether we also have a moral duty to denounce the hateful rhetoric and conduct of others. i believe we do, for silence is perceived as tacit approval or at least indifference. seen this way, silence helps foster hateful attitudes and related harm. in other words, not speaking out is just another way to fail in our obligations to be helpful and not harmful. moreover, as individuals we are able to speak out against hateful words and actions, in a variety of ways. by teaching tolerance to our children, for example, we can help them understand and appreciate differences among people, and therefore understand that hate-based responses to difference are simply wrong.

in sum, while it makes no sense to hold a generation responsible for anything as a group, i agree that every individual bears responsibility for speaking out against hateful words and behavior, as well as for resisting them.’

89. “the study of history is largely a waste of time because it prevents us from focusing on the challenges of the present.”discuss the extent to which you agree or disagree with the opinion expressed above. support your point of view with reasons and/or examples from your own experience, observations, or reading

the speaker suggests that studying history is a waste of time because it distracts us from current challenges. posed this way, the question carries the assumption that the study of history has no bearing on present problems or their possible solutions. on the contrary, history can provide examples, perspectives and insights that are directly relevant to contemporary challenges.

one way that studying history can help us face new challenges is by showing us inspirational (給予靈感的) examples of success. for instance, we can learn from the experience of the great inventor thomas edison that sometimes a series of apparent failures is really a precursor to success. also consider the journey of lewis and clark into the northwest territory. understanding the motivations needed to overcome adversities they faced can help to inspire modern-day explorers and scientists.

studying history can also help us avoid repeating mistakes. for instance, we can learn from the failure of prohibition during the 1930s that it can be a mistake to legislate morality. and future generations might learn from the 1997 indictment of the tobacco industry that it is bad policy to trade off the wellbeing of consumers in order to secure profits.

finally, the study of history is important because we cannot fully appreciate our present challenges without understanding their historical antecedents. consider the issue of whether california should be officially bilingual. the treaty that transferred california from mexico to the united states stipulated that california must embrace both spanish and english as official languages. those who view the current bilingual debate as purely a contemporary issue might bring to the debate a more enlightened viewpoint by appreciating this historical fact and the events that led to the treaty.

in sum, though the past might seem distant, it is far from irrelevant. studying history can inspire us to achievement, help us avoid costly mistakes, and help us simply appreciate that in most cases we’ve been down this road before.

90. “people often complain that products are not made to last. they feel that making products that wear out fairly quickly wastes both natural and human resources. what they fail to see, however, is that such manufacturing practices keep costs down for the consumer and stimulate demand.”which do you find more compelling: the complaint about products that do not last or the response to it? explain your position using relevant reasons and/or examples drawn from your own experience, observations, or reading.

sample essay 1:

this topic raises the issue of whether, on balance, consumers are damaged or benefited by quality-cutting production methods. indisputably, many consumer products today are not made to last. nevertheless, consumers themselves sanction (認可) this practice, and they are its ultimate beneficiaries—in terms of lower prices, more choices, and a stronger economy.

common sense tells us that sacrificing quality results in a net benefit to consumers and to the overall economy. cutting production corners not only allows a business to reduce a product’s retail price, it compels the business to do so, since its competitors will find innovative ways of capturing its market share otherwise. lower prices stimulate sales, which in turn generate healthy economic activity. observation also strongly supports this claim. one need only look at successful budget retail stores such as walmart as evidence that many—and perhaps most—consumers indeed tend to value price over quality.

do low-quality products waste natural resources? on balance, probably not. admittedly, to the extent that a product wears out sooner, more materials are needed for replacement units. yet cheaper materials are often synthetics, which conserve natural resources, as in the case of synthetic clothing, dyes and inks, and wood substitutes and composites. moreover, many synthetics and composites are now actually safer and more durable than their natural counterparts—especially in the area of construction materials.

do lower-quality products waste human resources? if by “waste” we mean “use up unnecessarily,” the answer is no. many lower-quality products are machine-made ones that conserve, not waste, human labor—for example, machine-stitched or dyed clothing and machine-tooled furniture. moreover, other machine-made products are actually higher in quality than their man-made counterparts, such as those requiring a precision and consistency that only machines can provide. finally, many cheaply made products are manufactured and assembled by the lower-cost asian and central american labor force—a legion for whom the alternative is unemployment and poverty. in these cases, producing lower-quality products does not “waste” human resources; to the contrary, it creates productive jobs.

in the final analysis, cost-cutting production methods benefit consumers, both in the short-term through lower prices and in the long run by way of economic vitality and increased competition. the claim that producing low-quality products wastes natural and human resources is specious at best.

sample essay 2 (6):

many people feel that products are not made to last, and correspondingly, many natural and human resources are wasted. on the other hand, it can be noted that such manufacturing practices keep costs down and hence stimulate demand. in this discussion, i shall present arguments favoring the former statement and refuting the latter statement.

products that are not made to last waste a great deal of natural and human resources. the exact amount of wasted natural resources depends on the specific product. for example, in the automobile industry, the yugo is the classic example of an underpriced vehicle that was not made to last. considering that the average yugo had (not “has” since they are no longer produced!) a life expectancy of two years and 25,000 miles, it was a terrible waste.

automobile industry standards today create vehicles that are warranted for about five years and 50,000 miles. by producing cheap yugos that last less than half as long as most cars are warranted, the yugo producer is wasting valuable natural resources. these same resources could be used by ford or toyota (豐田汽車) to produce as escort or tercel that will last twice as long, thereby reducing the usage of natural resources by a factor of two.

human resources in this example are also wasteful. on the production side, manufacturers of a poor quality automobile, like the yugo, get no personal or professional satisfaction from the fact that their product is the worst automobile in the united states. this knowledge adversely affects the productivity of the yugo workers.

conversely, the workers at the saturn plants constantly receive positive feedback on their successful products. saturn prides itself in its reputation for quality and innovation as is seen in its recent massive recall to fix defect. this recall was handled so well that saturn’s image was actually bolstered. had a recall occurred at a yugo plant, the bad situation would have become even worse.

another factor in the human resources area is the reaction by the consumer. a great deal of human resources have been wasted by yugo owners waiting for the dreaded tow truck to show up to haul away the yugo carcass. any vehicle owner who is uncertain of his/her vehicle’s performance at 7 am as he/she is about to drive to work, senses a great deal of despair. this is great waste of human resources for the consumer.

91. “government should establish regulations to reduce or eliminate any suspected health hazards in the environment, even when the scientific studies of these health hazards are incomplete or contradictory.”discuss the extent to which you agree or disagree with the opinion stated above. support your position with reasons and/or examples from your own experience, observations, or reading.

the stated opinion is that government should regulate any suspected environmental health hazards, even if relevant scientific evidence is conflicting or incomplete. while i agree that it’s often wiser to err on the side of caution, i think the speaker’s blanket (1: covering all members of a group or class *a blanket wage increase* 2: effective or applicable in all instances) assertion goes too far. government reaction to specific cases should be decided on the basis of two considerations: (1) the degree of evidential uncertainty, and (2) the seriousness of the risk involved.

the greatest uncertainty arises from contradictory evidence. consider an analogy taken from medical research, where one study links caffeine to increased risk of heart disease, while another claims there is no correlation between the two. provided that both studies used sufficiently large and random samples, and the results were statistically significant in each case, it is difficult to decide whether to give up coffee. if the effect in question were a little sleep disturbance, then it might be reasonable to sustain moderate intake of caffeine. but with a risk as serious as cancer, it would be reasonable to abstain, pending more conclusive evidence.

lesser degrees of uncertainty stem from incomplete evidence. one highly publicized case involved early studies suggesting that chloroflourocarbon emissions accelerate ozone depletion in our atmosphere. some scientists were unsure whether the models were correct; cfc-producing businesses took their case against regulation to congress and the public, arguing that the scientific evidence was inconclusive. but of course, waiting for conclusive evidence could mean the eventual destruction of life on our planet. the u.s. government wisely decided first to limit, and then prohibit most cfc production. the risks of being wrong in this case are enormous; today most of the international community is working toward the virtual elimination of chloroflourocarbons.

in sum, i believe it is unreasonable to give blanket prescriptions concerning government reaction to health hazards in the environment. where uncertainty is greatest, and risks are relatively small, it would be wise to wait for more scientific evidence. but when the risks are great, government should regulate against environmental health hazards, even in the face of uncertainty.

92. “employees should show loyalty to their company by fully supporting the company’s managers and policies, even when the employees believe that the managers and policies are misguided.”discuss the extent to which you agree or disagree with the opinion stated above. support your position with reasons and/or examples from your own experience, observations, or reading.

the issue is whether employees should show loyalty to their firms by unequivocally supporting company managers and policies. i agree that employees have a binding duty to be loyal to their employers. however, loyalty does not always mean mindless (沒頭腦的, 不用腦子的) support of superiors and their polices. moreover, in extreme circumstances, the duty to be loyal may be overridden by a more important duty.

employee loyalty is best understood as a commitment to seek the interests of the firm. in plain terms, the interests of the firm are to increase stockholder wealth. most of the time, and for most employees, this will mean following the orders and policies of those in charge.

sometimes, however, executives or managers may set counterproductive policies. an employee who clearly sees this might better serve the firm’s interests—and be loyal—by questioning the misguided policies rather than silently obeying them. for this reason, many companies will endure the presence of an occasional iconoclast among the ranks. although such independent thinkers are annoying from a managerial standpoint, they often put up (提供) the creative idea that saves the bottom line.

and, on occasion, company policy might be plainly unjust or harmful to society. consider the well-known example of roger boisjoly, the morton-thiokol engineer who had early concerns about the ill-fated launch of the space shuttle challenger. in such situations, conscience may require an employee to disregard ordinary loyalty and dispute the decisions made by superiors. and, if speaking out to company insiders is ineffective, the employee might recognize an overriding duty to go public, and blow the whistle on the firm.

in conclusion, employees have an important duty to be loyal to their employers. this duty, however, is not rightly construed as simple obedience; its most important feature is commitment to promote the employer’s interest in making a profit. occasionally, loyalty in this regard can require an employee to challenge unproductive company policies. and at times the duty to be loyal might itself be outweighed by obligations of conscience.

93. “to be successful, companies should trust their workers and give them as much freedom as possible. any company that tries to control employees’ behavior through a strict system of rewards and punishments will soon find that such controls have a negative effect on employee morale and, consequently, on the company’s success.”discuss the extent to which you agree or disagree with the opinion stated above. support your views with reasons and/or examples from your own experience, observations, or reading.

the stated opinion is that success in business is promoted by giving employees the greatest possible freedom, and avoiding strict systems of punishment and reward. although i agree with the speaker’s viewpoint, i would qualify (限制, 限定) it somewhat. employee freedom must be balanced against sound systems of managerial control and accountability. and certain rewards are appropriate, and are effective incentives to work harder and better.

first of all, current research suggests that employee freedom is good for business. for example, employees who are give the freedom to develop their own methods for completing tasks. in addition, employees with a larger role in company decision-making processes experience a sense of greater investment in their work and, in turn, become more productive. even so, employee freedom cannot be unlimited. to keep projects successfully on track, some system of managerial control is needed.

secondly, employers who motivate worker with rigid systems of reward and punishment are finding that this method often backfires. for one thing, people resent and resist being driven by the whip, so to speak (adv. 可謂,可以這么說). for another, employees who focus on the promise of an external reward tend to be less personally committed to the task at hand. the reason is obvious: the reward becomes more important than the work. in both cases, quality and productivity are likely to suffer.

nonetheless, employees who hope to be retained or promoted should expect to be held accountable for their job performance. furthermore, there should be special compensation for work done creatively, or especially well. for example, an unexpected bonus at the end of a successful project is a fitting reward that provides an incentive for future effort without risking the pitfalls of a stricter system.

in sum, it is better for business to avoid controlling employees by harsh and inflexible methods, including strict punishments and reward. people work more creatively and productively when given a measure of freedom on the job. still, this does not mean that organizations should abandon systems of accountability, or managerial control over projects.

94. “if parents want to prepare their children to succeed in life, teaching the children self-discipline is more important than teaching them self-esteem.”discuss the extent to which you agree or disagree with the opinion stated above. support your views with reasons and/or examples from your own experience, observations, or reading.

the speaker claims that teaching children self-discipline is more important to their future success than helping them develop self-esteem. i think the formula ought to be reversed. granted, self-discipline is a critical element of success. still, a person’s estimation of self forms the basis from which all one’s life choices, for good or ill (好歹), are made.

first of all, it is important to understand the concept of self-esteem. many psychologists recommend nurturing self-esteem in children above all else. detractors (詆毀者,貶低者) of this viewpoint denounce the prescription, arguing that it encourages children to grow up self-absorbed, overbearing, insolent and worse. but this criticism misconstrues self-esteem, which should not be confused with egotism or arrogance. instead, self-esteem begins with one’s positive assessment of self-worth (n. [=self-esteem]自尊, 自尊心,自大, 自負), and sustains personal characteristics like confidence, competence, and even caring.

given this understanding, it is difficult to overrate the connection between self-esteem and personal success. a child who grows up believing she is worthwhile, strong and able is more likely to be self-assured and well-adjusted. and this, in turn, will dispose her to attempt challenging projects and nurture positive associations with others.

admittedly, success is rare for those who procrastinate or cannot stay focused on the task at hand. however, there are many examples where self-discipline in the absence of healthy self-esteem has lead to undesirable and even tragic outcomes. take the menial worker who is meticulous on the job, but cannot envision herself capable of greater achievement. or, in the extreme case, consider the scooters at columbine high school, whose plans were exacting and carefully executed. they did not lack self-discipline, though they reportedly suffered very low self-esteem.

in conclusion, self-esteem is the most fundamental feature of personal accomplishment. for this reason, it is vital that parents nurture it in their children. it is also important to teach children self-discipline; however, it is incorrect to place its significance above that of self-esteem.

95. “companies are never justified in employing young children, even if the child’s family would benefit from the income.”discuss the extent to which you agree or disagree with the opinion stated above. support your views with reasons and/or examples from your own experience, observations, or reading.

the speaker claims that it is wrong for a company to employ young children, even if the child’s family needs the income. i strongly agree with this position. there are many reasons in favor of it, and few that would justify even the most exceptional cases of child employment.

to begin with, young children are easy targets or exploitation and abuse. employment is basically a contractual relationship between a firm and its employees. this relationship imposes duties of loyalty and standards of work on employees; at a minimum, it also obligates employers to compensate workers fairly and provide them with reasonably safe work environments. because of their age, young children are not yet fully able to grasp their rights in this kind of arrangement. therefore, they are not likely to recognize exploitive (adj. =exploitative) treatment by employers, such as overly long work periods, unfair wages, unsafe working conditions, and so on.

in addition, early employment can seriously harm children in other ways. first, it robs them of their most valuable commodity—childhood and the important schooling (學校教育) and play that normally comes with it. this, in turn, can diminish their potential to become well-adjusted and accomplished adults. even in the privileged circumstances of hollywood stardom, successful young actors face huge obstacles in their development toward adulthood. biographies of child stars are replete with stories of early substance abuse and psychological problems that stem from too much success, too soon. the recent tragic death of dana plato illustrates this point clearly.

moreover, family need is hardly a justification for employing young children. in our culture as well as many others, public programs are available to assist those in dire financial need. thus there is little rationale for thinking that economic need outweighs the palpable dangers of childhood employment.

in sum, companies should not employ young children. early work seriously jeopardizes the wellbeing and future prospects of children. moreover, financial need is not a compelling reason for child labor when alternatives are available.

96. “in order to understand a society, we must examine the contents of its museums and the subjects of its memorials. what a society chooses to preserve, display, and commemorate is the truest indicator of what the society values.”discuss the extent to which you agree or disagree with the opinion stated above. support your position with reasons and/or examples from your own experience, observations, or reading.

the speaker alleges that understanding a society requires examining its memorials and museums, since their subject matter and contents are the truest indicators of what a society values. i agree that we can learn a great deal about a society through its museums and memorials. however, these may not be the “truest” indicators of social values. to discover other values that shape a society, we also need to examine it popular forms of expression.

museums and memorials offer important evidence of what a society holds dear. for instance, early war memorials symbolize abstract virtues such as courage and honor, as well as combat victory. later memorials honoring the fallen in vietnam tend to emphasize individual sacrifice that will present its history in ways that underscore its hardships, achievements and ideals. when museums present evidence of a social failing, such as racism, the intent is to stress a cherished ideal, such as equality, that has been violated.

however, museums and memorials reflect a society’s official values, not necessarily its most pervasive or influential ones. to discover the common values that significantly affect daily life, we must explore a society’s popular media, its forms of entertainment and its advertising. from the media we learn about people’s interests and viewpoints. looking at popular entertainment tells us whether people enjoy representations of sex and violence, or stories of courage and valor. and advertising appeals to basic material values; it shows us what individuals are willing to buy, and why. in examining these popular forms of expression, we can see that the everyday preferences and values that shape a society are frequently at odds with its official ideals.

in conclusion, to develop a complete picture of what a society is like, we need to understand its officially sanctioned values as well as its popular ones. for this reason, it is important to examine popular forms of expression, as well as the content of a society’s museums or the subjects of its memorials.

97. “in business, more than in any other social arena, men and women have learned how to share power effectively.”discuss the extent to which you agree or disagree with the opinion stated above. support your views with reasons and/or examples from your own experience, observations, or reading.

the stated opinion is vague, with no clear meaning for “sharing power effectively.” but if this phrase is intended to convey the idea that roughly equal numbers of men and women occupy prestige and high-paying positions in business, then i believe the claim at issue is mistaken for two reasons.

first of all, it is not the case that women are demonstrably more successful at attaining powerful positions in business than in other social arenas. admittedly, the percentage of women earning degrees and entering the business world is significantly greater than in other prestigious professions such as medicine, engineering or science. however, the ratio of women in graduate business programs and in business management positions is about the same as in law schools and firms, or in ph.d. programs and in teaching positions in higher education. in business, law and higher education, the proportion of professional women is around 40 percent.

secondly, very few women achieve the highest-level positions in business. recent studies indicate that women occupy just under 3 percent of corporate executive positions from the vice president level on up. and more importantly, this percentage has not changed significantly during the past ten or fifteen years, a period during which the number of women in management careers in record numbers, they are setting into lower level jobs while their male counterparts are achieving the more powerful ones.

there is considerable controversy about the reasons why women tend to crowd around the bottom of the business career ladder. some blame the proverbial glass ceiling, said to be held firmly in place by an “old boy network.” others claim that women are naturally held back as they struggle to fulfill the dual roes of professional and family caregiver. in any case, women are not any more successful in achieving powerful positions in business than in some other high prestige careers; and they do not share power effectively with men and within the business world itself.

98. “in order to accommodate the increasing number of undergraduate students, college and universities should offer most courses through distance learning, such as videotaped instruction that can be accessed through the internet or cable television. requiring students to appear at a designated time and place is no longer an effective or efficient way of teaching most undergraduate courses.”discuss the extent to which you agree or disagree with the opinion stated above. support your position with reasons and/or examples from your own experience, observations, or reading.

in response to the challenge of providing education for an expanding undergraduate population, one tempting solution is to replace university teachers and classrooms with distance learning technologies like the internet or cable television. however, i believe that these technologies are best suited as valuable support resources, not as replacements for the traditional face-to-face classroom experience.

admittedly, the internet or cable television may be more cost-effective than traditional classrooms as means for quickly transmitting information to a large number of students. and, computer-generated standardized tests are a cheap way to assess information acquisition. however, there is much more to teaching than conveying information. likewise, there is more to learning than a demonstrated ability to pass standardized tests.

teaching just begins with the delivery of information. after that, teaching involves the complex and often spontaneous process of dialogical reasoning about the information at hand. this process includes clarifying, analyzing, evaluating, criticizing and synthesizing information and points of view, as well as creatively and logically exploring alternatives, solutions and new design possibilities. done well, teaching further provides effective models of rationality and moral responsibility. it is difficult to see how flat technology can replace the human element in these essential aspects of the teaching craft.

in the same way, absorbing information is just the starting point of learning. to learn is also to develop habits of careful, critical and creative thinking about information and to acquire a hunger for learning more. moreover, learning is fundamental to a person’s emerging rational autonomy and sense of moral responsibility to others. these dispositional aspects of learning are difficult to foster in technical packages or to assess; nonetheless they are at the heart of what learning is supposed to produce: educated persons.

in conclusion, i believe that distance learning technologies are best used as efficient supplements to teaching and learning. we cannot think that technology will make a good substitute for the classroom without relying on the unlikely assumption that students are effective autodidacts (n. 自學者, 自學成功的人), and can develop educated dispositions and habits of mind in the absence of teachers and mentors. this assumption, i fear, would effectively reduce education to unreflective training.

99. “if a nation is to ensure its own economic success, it must maintain a highly competitive educational system in which students compete among themselves and against students from other countries.”discuss the extent to which you agree or disagree with the opinion stated above. support your position with reasons and/or examples from your own experience, observations, or reading.

i don’t think it is a good idea to design an educational system that focuses mainly on competition. for although a little competition might produce desired results, in the long run too much competition will be destructive. instead, i believe that our national economic success will be better promoted by an educational system that encourages cooperative learning among students, and with students from other countries.

granted, competitiveness is an important aspect of human nature. and, properly directed, it can motivate us to reach higher and produce more, not to mention meet deadlines. but being competitive fixes our focus externally, on marking and beating the progress of others with whom we compete. such external motivation can direct our attention away from creative solutions to our problems, and away from important human values like cooperation and fair play (公平比賽, 公平條件). indeed, a highly competitive environment can foster cheating and ruthless back-stabbing within an organization, and ill-will and mistrust among nations. in the extreme case, competition between nations becomes war.

on the other hand, an environment of cooperation encourages us to discover our common goals and the best ways to achieve them. at the national and international levels, our main interests are in economic wellbeing and peace. in fact, economic success means little without the security of peace. thus, global peace becomes a powerful incentive for developing educational models of cooperative learning, and implementing exchange programs and shared research projects among universities from different countries.

moreover, research suggests that cooperative settings foster greater creativity and productivity than competitive ones. this has been shown to be the case both in institutions of higher learning and in business organizations. if true, it seems reasonable to argue that national economic success would be similarly tied to cooperative rather than competitive effort.

in conclusion, competition can provide an effective stimulus to achievement and reward. even so, i believe it would be unwise to make competition the centerpiece of our educational system. we stand to reap greater benefits, including economic ones, by encouraging cooperative learning.

100. “in order to force companies to improve policies and practices considered unethical or harmful, society should rely primarily on consumer action—such as refusal to buy products—rather than legislative action.”discuss the extent to which you agree or disagree with the opinion stated above. support your position with reasons and/or examples from your own experience, observations, or reading.

consumer action is an important tool for responding to harmful or unethical business practices. however, it is not always an especially effective one. thus i disagree with the stated opinion, and would argue that legislative action is an equally important means for constraining the activities of business.

ideally, consumer action would be the best free market response to harmful or unethical business activity. consider widespread public support for the united farm workers’ table grape boycott, called by late union leader cesar chavez in protest of injustices against field workers. the boycott worked; eventually loss of revenues forced california grape growers to agree to fairer working conditions. this is what adam smith had in mind when he argued that business, left free to pursue profit, would be guided by the so-called invisible hand of competition to produce the greatest social benefit.

however, history has shown the hand of competition sometimes to be inept as well as invisible in guiding the modern corporation. limited solely by the forces of competition, corporations have been known to steal from one another, exploit workers, inflate prices, market harmful products, lie in advertising and pollute the environment. moreover, consumer response has had little if any effect on many such practices.

indeed, it is increasingly difficult to direct an effective consumer response. in a time of large conglomerates and multi-tiered international corporations, consumers may not know which company is responsible for a given harm or injustice. additionally, those harmed by a business may not be its consumers. for instance, if a clothing chain subcontracts to foreign garment companies that seriously exploit their workers, few of the store’s customers would stop shopping there out of sympathy for faceless (不知名的) workers thousands of miles away. finally, consumer action takes time; it took years for the ufw boycott to succeed. sometimes the harm in question is simply too great to wait for the invisible hand.

in conclusion, consumer action should not be the primary means of reacting to undesirable business practices. some legislation is necessary to curb business activities that create serious harm or injustice.

101. “the automobile has caused more problems than it has solved. most societies would probably be much better off if the automobile had never been invented.”discuss the extent to which you agree or disagree with the opinion stated above. support your views with reasons and/or examples from your own experience, observations, or reading.

the speaker claims that most societies would be better off without the invention of the automobile. granted, automotive transportation has imposed tremendous costs on society. nevertheless, i disagree with the stated opinion, for the automobile has produced even greater benefits.

admittedly, automobiles create serious social problems. highway vehicles are responsible for a large share of the carbon monoxide, nitrogen oxide, sulfur dioxide and other toxic emissions that pollute our air. moreover, chemical gasoline additives designed to reduce air pollution have resulted in unexpected ground and water contamination. debilitating and often fatal pollution-related respiratory diseases are causing health care costs to skyrocket. similarly rising are the costs of auto accident-related injuries and deaths, which emergency room personnel claim amount to an epidemic. in addition, crowded urban areas now experience the psychological phenomenon of road rage, which often leads to assault injuries and deaths. and, out-of-use vehicles are unsightly litter in front yards, junkyards and the countryside.

however, automobile use is not the sole factor leading to these problems and their associated costs. overpopulation is another contributing cause; if there were fewer people, and fewer people with cars, the problems mentioned above would be less severe as well. but give the current global population explosion, it is difficult to imagine very many things in society functioning at all without automotive transportation.

the most general benefit of the automobile is increased mobility, which in turn provides for the efficient delivery of emergency services, medical supplies, housing materials, fresh food and other important goods to large numbers of people. moreover, the automobile contributes to the employment picture in two ways: the industry provides many jobs, and people are more likely to get jobs if they own and drive automobiles. the automobile also provides other benefits, like recreation and convenience, that are not quite in category of basic need. but their importance should not be underestimated. recreation contributes significantly to quality of life and wellbeing. and sheer convenience opens up possibilities for the realization of many additional goods, like helping those in need.

in conclusion, the invention of the automobile was indeed a mixed blessing. but the benefits of automotive technology outweigh its harms, particularly in a heavily populated world.

102. “an advanced degree may help someone get a particular job. once a person begins working, however, the advanced degree and the formal education it represents are rarely relevant to success on the job.”discuss the extent to which you agree or disagree with the opinion stated above. support your views with reasons and/or examples from your own experience, observations, or reading.

i strongly disagree with the speaker’s opinion that earning an advanced degree is rarely relevant to success on the job. granted there are many technical disciplines that experience rapid “information turnover,” leaving specific information acquired in one’s graduate or professional program quickly outdated. nonetheless, there are many features of advanced formal education that will contribute to success both on and off the job throughout one’s lifetime.

to begin with, even in such areas as computer science, where it is estimated that much of what a student learns today will be obsolete in just five years, an advanced degree and the education it represents will continue to be of benefit. unlike undergraduate education, which is divided into a number of small educational units every term, post-baccalaureate (<美>學士學位) education is concentrated on deeper learning in fewer areas. moreover, most graduate education requires extended, critical focus on complex issues or research projects. and even if the technical information relevant to completing a graduate-level project becomes outmoded, the learned critical methods and problem-solving approaches will not.

this is because such methods and approaches require ability in creative and logical thinking. it takes significant interpretative and analytical skill to successfully learn a body of complex material or research a difficult issue. moreover, developing a sophisticated research project or solving a complex technical problem require those skills plus the abilities to creatively envision alternatives, and logically rule out all but the best ones.

finally, it takes discipline and persistence to complete an advanced degree. because of the sheer scope and complexity of the material, issues and problems studied at the graduate level, it is not easy to bring a thesis or research project to completion. giving up is a constant temptation. those who succeed demonstrate traits of character that will serve them for a lifetime.

in conclusion, earning an advanced degree will be an element in most people’s success on the job, even though the information they learn in graduate or professional school may become obsolete. advanced study fosters important reasoning and problem-solving skills, as well as character traits like discipline and persistence. these skills and traits cannot help but contribute to personal success.

103. “most people today place too much emphasis on satisfying their immediate desires. the overall quality of life would be greatly improved if we all focused instead on meeting our long-term needs.”discuss the extent to which you agree or disagree with the opinion stated above. support your position with reasons and/or examples from your own experience, observations, or reading.

the speaker claims that the overall quality of life would be better if people would resist satisfying immediate desires and focus on long-term needs. i tend to agree. common examples related to health and financial security serve as an apt basis for my agreement.

to begin with, long-term good health is necessary to optimal quality of life. even so, there are those who use tobacco or abuse substances, fully understanding the dire health consequences of such behavior. in addition, many people have existing health problems that can be alleviated or eliminated with proper nutrition and exercise, yet persist in enjoyable but unhealthy habits. less dramatically, many of us lack the discipline to invest one half hour, three times a week, in sustained aerobic exercise that can slow the aging process and contribute to improved health.

secondly, a good quality of life requires long-term financial security. yet, in our consumerist society, far too many people spend money they should be saving or investing, just to own the latest technological toy or go on a dream vacation. worse yet, others will use credit, spending money they don’t even have, to gratify immediate desires for consumer products, recreation or entertainment. in all cases, they are literally stealing from their own future wealth and security in order to seize short-term satisfaction. financial worry, delayed retirement and even impoverished old age are common outcomes in such scenarios.

finally, immediate gratification on the part of some can diminish the quality of life for others. family and friends suffer from a loved one’s ill-health or financial instability. and careful, disciplined people end up giving away some of their wealth, through taxation, in order to subsidize public programs for those who have traded immediate gratification for poverty or ill-health.

in conclusion, the stated opinion is correct, we would all be better off in the long run if more people chose to forego satisfaction of current desires. of course, not every detriment to quality of life is an outcome of personal choice. but a great many problems do result from undisciplined seeking of instant gratification.

104. “the value of any nation should be measured more by its scientific and artistic achievements than by its business successes.”discuss the extent to which you agree or disagree with the opinion stated above. support your views with reasons and/or examples from your own experience, observations, or reading.

it is tempting to concede the speaker’s opinion that achievements in art and science are more significant to the worth of a nation than its “mere” business successes. still, i disagree with the view because it overlooks the several ways that commercial success supports artistic accomplishment and scientific progress. it also ignores the extent to which commercial success renders a nation more secure, and better able to promote human rights and political stability throughout the world.

first of all, art and science have always depended upon the patronage of successful business. consider italy’s powerful banking clan, the medici family, and their support for artists such as michelangelo (米開朗基羅[1475-1564,義大利文藝復興時期成就卓著的科學家,藝術家]) and raphael. substantial scientific and artistic support now comes from foundations established by extraordinarily successful business families like the rockefellers, gettys, carnegies and mellons. in addition to their private-sector foundation funding, successful businesses also pay taxes that are returned to university art and science programs. and, many businesses form partnerships with universities to further scientific discovery and related technological progress. finally, both art and science are highly successful business enterprises in their own right.

secondly, the value of a nation is related to its ability to defend itself, and to help promote peace and justice among other nations. a nation is more capable in both regards when its businesses enjoy greater success. for example, the current return of kosovars to their homeland was facilitated by a cost-intensive military effort that required funding from, among other sources, tax revenues from business. in addition, a wealthy nation can, by means of its trade agreements, use its economic strength to encourage other countries to extend greater human rights to their citizens. finally, a wealthy nation can depend on an expansive weapons industry in the private sector; and it can fund an effective military to carry out national defense.

in sum, it is simplistic to suppose that achievement in art and science is more important to the value of a nation than the success of its businesses. the latter is essential to national and world security. and, business success is deeply connected to accomplishment in art and science.

105. “all archeological treasures should remain in the country in which they were originally discovered. these works should not be exported, even if museums in other parts of the world are better able to preserve and display them.”discuss the extent to which you agree or disagree with the opinion stated above. support your views with reasons and/or examples from your own experience, observations, or reading.

whether archaeological treasures should remain in the countries where they are found is a complex and controversial issue. i sympathize with the view that antiquities should remain in the country of their discovery. but given real-word considerations, it is sometimes best to place archaeological treasures wherever they will be safe and well-preserved.

recent antiquities laws throughout the world reflect my point of view that the ancient treasures of a place should remain there. it seems outrageous that greeks or egyptians must visit the british museum to see the best remnants of their distant past; and this link is grounds for a vague but justified claim to ownership.

however, cultural ownership is only one consideration. historically, ancient treasures have been most interesting to two groups: scholars and robbers. admittedly, the two are sometimes indistinguishable, as when schliemann stole out of turkey with an immense trove (收藏的物件;被發現的東西) of what he mistakenly thought was king priam’s treasure. schliemann eventually placed his collection in the relatively safe hands of national museums, where it took the vicissitudes of war to destroy part of it. but none of schliemann’s find would be available to the turkish people or the world if plunderers had got there first.

often, the plunderers do get there first. when carter found the tomb of tutankhamen, tomb-robbers, largely egyptian, had carried off the treasures from bombs of other pharaohs ([古埃及的]法老). the fact that the world, including the egyptians, have the exhaustively cataloged and well-preserved wonders of the tutanhkamen find is owing to carter and his associates. this, then, becomes the only argument for exporting ancient treasures to safer locations: it is a lesser evil than not having the treasures at all.

in sum, it is usually best to leave archaeological treasures within the country of their discovery. even so, it is sometimes necessary to relocate them. this, however, leaves open two important and related issues: which specific situations justify relocation; and, whether there is ever an obligation to restore collections to the country where they were found.

106. “the most effective way for managers to assign work is to divide complex tasks into their simpler component parts. this way, each worker completes a small portion of the task but contributes to the whole.”discuss the extent to which you agree or disagree with the opinion stated above. support your views with reasons and/or examples from your own experience, observations, or reading.

the stated opinion is that the most effective way for managers to assign work is to divide complex tasks into their simpler component parts. this strategy is probably cost-effective (節省成本的) in many situations. however, i think that it works against important organizational values over time.

distinct divisions of labor are efficient for a number of reasons. first of all, workers with few highly specific skills are usually cheaper to hire than those with broader education and experience. secondly, it is less expensive to train employees in narrow areas. finally, strict compartmentalization (n.區分, 劃分) of tasks makes it easier for managers to control employees, and, therefore, to control and increase productivity. but however profitable this strategy might be in the short run, it can ultimately work against the organization.

to begin with, fragmenting work into small units leads to employee alienation. those responsible for only a detailed component of a project can easily lose sight of larger organizational goals and their own importance in achieving them. research indicates that they then become less committed to their work, and less productive. of course, unproductive employees can be replaced. but replacement is costly; and high employee turnover is bad for organizational morale.

in addition, compartmentalizing tasks can stifle creativity, as well as undermine self-motivation and pride in one’s work. with little collaboration or even communication between discrete work units, larger creative insights are lost. and, cooperative efforts usually foster a series of common purpose and pride in accomplishment.

of course, diversifying jobs and increasing worker participation in larger projects could lead to lower productivity. but the experience of large manufacturing corporations like general motors shows just the opposite. when gm facilities implemented these and other strategies to improve work-place quality, they reported that productivity increased.

in conclusion, i believe that organizing work into discrete tasks will compromise important organizational values like creativity, self-motivation, commitment and pride in accomplishment. so, although there are times when small divisions of labor will be necessary, generally work should be diversified, and workers should have greater involvement in projects overall.

107. “people are overwhelmed by the increasing amount of information available on the computer. therefore, the immediate goal of the information technology industry should be to help people learn how to obtain the information they need efficiently and wisely.”discuss the extent to which you agree or disagree with the opinion stated above. support your views with reasons and/or examples from your own experience, observations, or reading.

the speaker asserts that because of the vast amount of information available on the internet and its accessibility to anyone with a computer and an internet connection ([計]internet聯接), it is increasingly important that people be able to access their target information efficiently, without confronting great amounts of irrelevant information along the way. i agree with this view. additionally, individuals need to be able to separate the wheat from the chaff on the internet, since with increasing information comes increasing misinformation (n. 誤報, 誤傳).

it is possible today to sit down at one’s computer with a specific question in mind, one that can be safely presumed to have an answer somewhere in cyberspace (n. 電腦空間). but there is no guarantee that the questioner will have an easy time of finding that answer, and there are several reasons for this unhappy fact. for one thing, there is no comprehensive “directory” or “index” to the information stored on the net. another problem is that people do not always describe a subject the same way. so if one person searches for information under a certain label, only if whoever provides the desired information used the same label will the first person find the information easily.

a different kind of problem is the fact that, as commercial enterprises, the proprietors (所有者;經營者) of commercial search engines are subject to financial pressures and thus tend to favor some candidates for search result lists over others. what this means is that, if you do a search on a particular topic, the company that owns the search engine is likely to display most prominently those items whose producers pay the most for such display.

finally, we must remember that anyone with modest equipment and expertise can establish a website. the result is that unless one knows from whom one is obtaining information, one must be circumspect about the integrity of the information as well as the motives of the information provider.

108. “employees should not have full access to their own personnel files. if, for example, employees were allowed to see certain confidential materials, the people supplying that information would not be likely to express their opinions candidly.”discuss the extent to which you agree or disagree with the opinion stated above. support your views with reasons and/or examples from your own experience, observations, or reading.

the issue is whether employees should have full access to their own personnel files. the speaker claims that they should not, pointing out that such access could diminish the condor of those supplying information. to some extent, i agree with this viewpoint. although employees are entitled to be accurately informed about the substance of performance reviews or complaints in their files, at times there will be good reason not to identify information sources.

first of all, employers have a right to control some information pertinent to (adj. 與..有關的) their business success. unproductive or uncooperative workers can seriously harm an organization; for this reason, employers need to have accurate information about employee performance. but when employees have full access to their own personnel files, co-workers and even supervisors will often find it difficult to give frank criticism of underachievers (n. 成就過小者,在學業方面未發揮潛力的學生) or to report troublemakers. so although employees have legitimate claims to know what has been said about them, they are not always entitled to know who said it.

secondly, employers are obligated to control some information when their employees are accursed of unlawful conduct. since employers are responsible for wrongdoing (不正當行為;壞事) at the workplace, they must investigate charges of, for example, drug activity, possession of firearms, or harassment. but again, without assurances of anonymity, accusers may be less forthright. furthermore, they may be in jeopardy of retaliation by the accused. so while workers under investigation may be generally informed about complaints or reports, they should not know who filed them. even so, employers do not enjoy an unlimited right to gather and keep confidential information about employees. for example, it would be unjust to investigate an employee’s political viewpoints, religious preference, or sexual orientation. such invasions of privacy are not warranted by an employer’s right to performance-related information, or duty to protect the workplace from criminal wrongdoing.

in conclusion, limiting employee access to personnel files is sometimes warranted to encourage candor and prevent retaliation against information sources. at the same time, employers have no right to solicit or secure information about the private lives of their workers.

109. “all personnel evaluations at a company should be multi-directional — that is, people at every level of the organization should review not only those working ‘under’ them but also those working ‘over’ them.”discuss the extent to which you agree or disagree with the opinion stated above. support your views with reasons and/or examples from your own experience, observations, or reading.

the speaker asserts that employees at all levels in an organization should review one another, including those working “over” them as well as “under” them. i agree in part. often companies will conduct two different kinds of periodic review, one to justify decisions regarding promotion and pay, and another to increase overall efficiency by assessing employee performance. multi-directional evaluation should never be part of the first kind of review; however, it can be valuable in the second kind and, therefore, should be used there.

on the one hand, lower-level employees have too much organizational power if their evaluations are used in decisions about the pay or promotions of their superiors. employees can intimidate superiors with the threat of bad review. also employees can use the review process to retaliate against those at higher levels. in either case, the authority of a manager or an executive can be seriously compromised, and productivity is lost in the process.

on the other hand, the most revealing criticisms of a superior’s style often come from those subject to it. in a process of review that isn’t connected to promotion or pay, employees at all levels can be more comfortable and forthright about sharing concerns. in turn, every employee is more likely to get accurate feedback, including constructive criticism, that will help each nurture strengths and improve areas of weakness. in this way (adv. 這樣), multi-directional evaluation can greatly enhance organizational efficiency.

furthermore, multi-directional evaluation in this context helps prevent worker alienation and subsequent lowered productivity. widening the performance review process will very likely foster a greater sense of personal involvement in one’s work, especially among lower-level employees. recent studies have shown that people who feel more invested (invest: b: to furnish with power or authority; c: to grant someone control or authority over: vest) in their jobs tend to work more cooperatively and productively.

in conclusion, there is an important role for multi-directional personnel evaluation in the workplace. while it should be clearly separate from issues of promotion and pay, as part of the performance review process it can encourage better employee relations and higher productivity.

110. “the most effective business leaders are those who maintain the highest ethical standards.”discuss the extent to which you agree or disagree with the opinion stated above. support your views with reasons and/or examples from your own experience, observations, or reading.

the assertion at issue is that business people who uphold the highest ethical standards are the most effective leader. i strongly agree with this statement. for a while (adv. 暫時), unethical behavior might seem effective. but a few examples from the investment banking industry keenly illustrate how dishonesty and corruption in leadership can bring a business to its knees, shattering the trust of its employees and ruining its reputation with clients.

consider the cases of michael milken, former head of junk bond trading at drexel burnham lambert, and paul mozer, formerly in charge of salomon brothers’ government bonds trading. each of these men engaged in double-dealing (口是心非,欺騙) and other illegal acts, reaping tremendous profits for their companies, and winning the admiration of subordinates and superiors alike. however, their successes were relatively short-lived (短命的;持續不久的;曇花一現的). securities exchange commission (sec) investigations in each case revealed massive wrongdoing. as a result, drexel went out of business. and salomon brothers barely recovered, after suffering the forced resignations of its top executives, a financially devastating loss of reputation, and the exodus of many valued employees.

moreover, salomon’s survival is probably owing entirely to its subsequent leadership under warren buffett. buffett, who was on the salomon brother board of directors at the time of the scandal, was brought in to (bring in to: vt. 把..告訴,讓..知道,讓..參與) save the beleaguered company. his success in keeping it afloat at all can be directly tied to his sterling (conforming to the highest standard *sterling character*) ethical reputation in the international business community at the time. buffett’s reputation restored at least some lost confidence among clients and investors, and probably prompted some employees to reconsider their decisions to leave the company.

while not every case of unethical leadership is quite so public or devastating as these, they do illustrate an important point. in any business, once corruption at the top becomes known, the predictable outcome will be damaged reputation, lower worker morale, and, along with them, lost productivity.

in conclusion, unethical conduct at the leadership level in a company might go unnoticed and serve one’s interests in the short-term. however, in the long run it will work against one’s effectiveness and may even prove ruinous.

111. “because of recent advancements in business and technology, the overall quality of life in most societies has never been better than at the present time.”discuss the extent to which you agree or disagree with the opinion stated above. support your views with reasons and/or examples from your own experience, observations, or reading.

the stated opinion is that recent advancements in business and technology have made overall quality of life better than ever. i disagree somewhat with the speaker’s viewpoint. for although such advancements have improved our lives in many respects, they have also diminished our quality of life in other ways.

clearly, progress in business and technology has produced many benefits. for example, we can research problems and their solutions in minutes on the internet; productivity is at an all-time (adj. 空前的, 創記錄的) high. and we can get more done in less time, leaving more time for hobbies, entertainment or other leisure activities. we can even mix a little work into our leisure time, by taking our laptops (攜帶型電腦) and cell phones on vacation (在度假中). this way, we can stay one step ahead on projects at work, anticipating deadlines and staying in touch with co-workers and important clients.

in addition, leisure time has itself been enhanced by business and technology. never before have we had so many spectacular diversions available, or so many leisure- and entertainment-related businesses vying for our attention. moreover, we can obtain everything form airline tickets to a language course and holiday wardrobe (行頭a collection of wearing apparel (as of one person or for one activity) *a summer wardrobe*) via the internet, in the convenience of our own homes.

nevertheless, advances in business and technology have compromised our quality of life as well. for all the wonders of computers, they have spawned their own special illnesses and ailments, like severe eyestrain (眼睛疲勞), back and neck problems, and carpal tunnel syndrome (n. 腕管綜合徵: a condition caused by compression of a nerve where it passes through the wrist into the hand and characterized especially by weakness, pain, and disturbances of sensation in the hand). and though we now have a world of information available in a keystroke (按鍵,按一下鍵), some of this information—like pornography, hate group diatribes and bomb-building instructions—are harming our society, especially our children. even apparently harmless material, like direct mail advertising and telephone soliciting, is endlessly annoying. finally, family life is sometimes a casualty (a person or thing injured, lost, or destroyed : victim *the ex-senator was a casualty of the last election*) of all this progress, with parents and children spending more time transfixed (使呆住) before their computer monitors and less time together.

in conclusion, advances in business and technology are a mixed blessing. for while we enjoy many benefits of this so-called progress, in many ways it has changed our lives for the worse.

112. “in most fields—including education, politics, and business—the prevailing philosophy never stays in place very long. this pattern of constantly shifting from one theoretical position to another is an inevitable reflection of human nature: people soon tire of the status quo.”discuss the extent to which you agree or disagree with the opinion stated above. support your views with reasons and/or examples from your own experience, observations, or reading.

the speaker claims that the prevalent philosophies in many areas change frequently, a result of people tiring of the statue quo. i disagree with this claim because, first, philosophical perspectives do not change frequently; and, second, it is not mere tiresomeness that lies at the root of such changes.

consider first the field of higher education, where the longtime objectives of broadly educating and civilizing students now clash with the business-oriented goal of producing efficient workhorses (吃苦耐勞者) for the world of commerce (world of industry). with the power of money behind it for ammunition, the latter just might win. but, this change will hardly be due to anyone’s boredom with the status quo. instead, it will be an outcome of evolving social and economic forces.

secondly, consider changes in political philosophy. a traditional liberal philosophy is that government should provide for the underprivileged through various public assistance programs. one outcome of a plain welfare program, however, was that recipients lingered for years on its rolls. the liberal point of view is now evolving to one that endorses some public stipend, but requires a return to the workforce. i think this change has come about as an attempt to improve ideology on the basis of past failures, not simply because people grew tired of the existing order.

finally, consider the transition in business from models of rigid management control to the cooperative workplace. whereas traditionally workers complied with orders dictated from the top and were motivated by strict reward/punishment systems, many employees today participate in the decisions affecting their work, and are motivated by more cooperative projects. this shift, moreover, is the result of extensive research that shows stricter models result in lower productivity.

in conclusion, philosophical changes in the areas of education, politics and business are not frequent, but slowly evolve. furthermore, the transitions in question are the outcomes of socioeconomic change, research, and lessons from past failures, not tedium associated with the status quo.

113. “it is essential that the nations of the world increase spending on the building of space stations and on the exploration of other planets, even if that means spending less on other government programs.”discuss the extent to which you agree or disagree with the opinion stated above. support your position with reasons and/or examples from your own experience, observations, or reading.

the author believes that increased spending on space stations and the exploration of other planets is essential. i disagree with this view, and i especially disagree with the claim that such spending should take place at the expense of other government programs.

increased funding for the space program would be justified only if it could be shown that money spent on space programs would improve the lives of people on earth more than money spent in other areas that directly affect the wellbeing of human. but there is no evidence that space programs can benefit people to a greater extent than, for example, medical care, medical research, education or environmental protection.

admittedly, the space program has produced a great many (adj.很多) “spinoff (n. 副產品,伴隨的結果)” results that make life better in ways that have nothing to do with space. but if the nations of the world were to make as substantial an investment in medical research—or environmental protection, or marine exploration—many unforeseen but useful byproducts would certainly result. and it seems unwise to argue that we should invest huge sums of money in a project, hoping it will produce virtually unforeseen good results, particularly when the alternative is to invest the same money in projects that are certain to produce substantial benefits.

it remains to consider whether the avowed goals of the space programs can justify increased spending. but those goals are unclear. we explore the universe out of scientific curiosity, which might warrant spending at the current level. but we are producing as much information now as the scientific community can reasonably assess. without convincing arguments that some currently unstated goal will be served by more spending, there can be no justification for taking money from other programs.

therefore, it is not essential to increase funding for the space program, or to sacrifice other programs on its behalf. space exploration does not yield the obvious benefits to humans that other government programs do. and neither the promise of spinoff benefits nor the current goals of the program justify increased spending.

114. “technology ultimately separates and alienates people more than it serves to bring them together.”discuss the extent to which you agree or disagree with the opinion stated above. support your posit ion with reasons and/or examples from your own experience, observations, or reading.

i believe there is some truth to the speaker’s claim that technology separates and alienates people. however, there is certainly at least as much evidence that technology serves best to bring people together.

the most obvious way that technology separates and alienates people from one another is symbolized by the computer nerd (nerd: n. 討厭的人, 卑微的人an unstylish, unattractive, or socially inept person; especially: one slavishly devoted to intellectual or academic pursuits *computer nerds*) sitting glazed-eyed (adj. 面無表情的, 目光呆滯的) before his computer screen in a basement, attic, bedroom, or office cubicle. while this scene is a caricature, of course, it’s true that practically everybody who uses email or surfs (transitive senses: to scan the offerings of [as television or the internet] for something that is interesting or fills a need) the internet does so alone, with only his or her computer for company (adv. 陪著). and, to the extent that computer use increases the amount of time we collectively spend in solitary activities, it increases the amount of time we spend separated from our fellow humans.

on the other hand, technology has been a wonderful aid in bringing people together, or, in many cases, back together. speaking for myself, i can say that i have become connected with quite a number of people via email with whom i might never have spoken otherwise. these include old friends with whom i had fallen out of (fall out of: v. 放棄[習慣等]) the habit of writing regular letters but with whom i now correspond regularly because of the ease with which email can be sent and delivered.

a second way in which the new technology has brought people together is by allowing individuals who have common interests to make contact with one another. it is possible to find people who share one’s interest in nearly anything, from aardvarks (n. [動]土豚) to zippers. such contacts may be ephemeral, but they can be a great source of information and amusement as well. i would hazard (venture, risk *hazard a guess as to the outcome*) a guess that for each person who sits neurotically (neurotic: of, relating to, constituting, or affected with neurosis) at home, eschewing personal contacts with others in favor of an exclusive relationship with his computer, there are hundreds of others who have parleyed their email capacity and their access to the web into a continuous succession of new acquaintances.

in sum, it seems clear to me that technology has done more to bring people together than to isolate them.

134. “although many people object to advertisements and solicitations (the practice or act or an instance of soliciting; especially: entreaty, importunity) that intrude into their lives through such means as the telephone, the internet, and television, companies and organizations must have the right to contact potential customers and donors whenever and however they wish.”discuss the extent to which you agree or disagree with the opinion stated above. support your views with reasons and/or examples from your own experience, observations, or reading.

the speaker’s opinion is that government should not restrict advertising or soliciting though the mail, telephone, internet, or television. up to a point, i agree with this viewpoint. however, i strongly disagree with the reasoning that businesses and organizations enjoy a right to advertise or solicit, at any time and through any means.

it is doubtful that such a sweeping (a: extensive *sweeping reforms*; b: marked by wholesale and indiscriminate inclusion *sweeping generalities*) right can be defended by the usual appeals to freedom of speech or free market economics. first of all, public expression is justifiably limited when it is obscene or causes significant harm; federal communication guidelines that apply to advertising and soliciting reflect this. secondly, free markets must satisfy certain criteria, including that full information about competing products be held by everyone, and that competition not be unfairly thwarted. under these conditions, it would be hard to defend any but purely informational advertising; in contrast, most actual advertising is designed to manipulate people’s appetites, desires or sense of loyalty.

even so, we should be wary of government restrictions on advertising or soliciting. government involvement in our free pursuits is justified only to prevent substantial harm to society. when advertisements or solicitations are clearly harmful, as is obvious with much pornography on the internet, then government should intervene to restrict such messages, particularly those directed at children. but although endless sales pitches (pitch竭力推銷) and pleas for charity are certainly annoying, most of them are not all that damaging.

moreover, consumers can limit the number of ads and solicitations they receive. for instance, consumer-protection organizations provide information about how to remove oneself from mailing lists. most credit card companies offer customers the choice to receive direct mail or telephone advertising. software companies and internet servers provide programs and other means for restricting information received online. and telephone solicitors will, upon request, place consumers on their “no call” list.

in conclusion, government generally should not restrict advertising or soliciting through the media, mail, telephone or internet. this is best left to consumers, who have means available to them. however, with respect to ads or solicitations that create serious harm to society, especially to children, government restrictions are justified.